board vitals part II

¡Supera tus tareas y exámenes ahora con Quizwiz!

When performing a triple arthrodesis on a foot with a significant hindfoot valgus deformity particular forefoot deformities must be addressed as part of the complete foot correction? (Select all that apply) A. Forefoot varus B. Forefoot abduction C. Forefoot valgus D. Forefoot adduction

B - Forefoot abduction,A - Forefoot varus. Correct answer: (A) (B) Forefoot varus and forefoot abduction. Explanation: A foot that has significant hindfoot valgus will have a compensatory forefoot varus and an abducted forefoot. Therefore when performing a triple arthrodesis and placing the hindfoot in a rectus to 5° valgus position the forefoot positions must be addressed. The forefoot varus stems from the midtarsal joint supination that must occur as a result of the ground reactive force pushing on the arch as the hindfoot seeks a valgus potential. The forefoot abduction as the subtalar joint pronates and the forefoot lateralizing about the talar head. A foot that has significant hindfoot valgus will have a compensatory forefoot varus and an abducted forefoot. Therefore when performing a triple arthrodesis and placing the hindfoot in a rectus to 5° valgus position the forefoot positions must be addressed. The forefoot abduction as the subtalar joint pronates and the forefoot lateralizing about the talar head. (C) A compensatory forefoot valgus does not occur with a hindfoot valgus. (D) Forefoot adduction does not occur with a valgus hindfoot.

39 yo female presents with pain in her right distal hallux. Patient has history of psoriasis in her skin/nails. Upon taking an xray it is revealed that the patient has erosions at the central joint as well as whiskering, periostitis and an "ivory phalanx". Which of the following is most likely diagnosis? A. Rheumatoid Arthritis B. Psoriatic Arthritis C. Ankylosing Spondylitis D. Reiter's Syndrome

B. Psoriatic Arthritis

Obtaining a pre-operative EKG is recommended for which 2 of the following patients? Please choose the 2 best answers: A. 52 yo M with no significant medical history B. 65 yo F with history of insulin-dependent diabetes C. 45 yo F with history of compensated CHF D. 58 yo M with history of hypothyroidism E. 55 yo F with depression, well-controlled with medication

C - 45 yo F with history of compensated CHF,B - 65 yo F with history of insulin-dependent diabetes. Correct answer: (B) (C) 65 yo F with history of insulin-dependent diabetes and 45 yo F with history of compensated CHF. Explanation: A pre-operative EKG is not recommended for all podiatric surgical cases. Pre- operative EKGs are recommended for intermediate risk surgical procedures when a patient has at least one known risk factor. Many podiatric surgeries will fall into the low-risk category if they are ambulatory or superficial procedures, however larger cases will likely be considered intermediate-risk. For patients undergoing intermediate-risk surgery with one or more risk factors, a preoperative EKG is recommended. Such risk factors include cerebrovascular disease, congestive heart failure, creatinine levels >2.0 mg/dL, insulin-dependent diabetes mellitus, or ischemic cardiac disease.

Radiographic evaluation of the cavus foot deformity on the lateral foot projection demonstrated a Meary's angle of 14° and a calcaneal inclination angle of 20°. Where is possible the apex of deformity for this cavus foot? (Select all that apply) A. Lisfranc's Joint B. Naviculocuneiform Joint C. Chopart's Joint D. Proximal to Chopart's Joint

C - Chopart's Joint,A - Lisfranc's Joint. Correct answer: (A) (C) Lisfranc's Joint and Chopart's Joint. Explanation: A Meary's angle of 14° and a calcaneal inclination angle of 20° is indicative of an anterior cavus deformity whose apex can be in Lisfranc's joint. A anterior cavus foot is characterized on the lateral projection by a Meary's angle greater than 10° and a calcaneal inclination angle less than 30°. A Meary's angle of 14° and a calcaneal inclination angle of 20° is indicative of an anterior cavus deformity whose apex can be in Chopart's joint. A anterior cavus foot is characterized on the lateral projection by a Meary's angle greater than 10° and a calcaneal inclination angle less than 30°. (B) The naviculocuneiform joint is not a possible apex of deformity for the cavus foot deformity. (D) When the apex of deformity in a cavus foot is located proximal to the Chopart's joint this describes a posterior cavus deformity. The posterior cavus deformity is characterized on the lateral projection by a Meary's angle less than 10° and a calcaneal inclination angle greater than 30°.

Which of the following is a possible cause of metatarsus adductus? (select all that apply) A. Contraction of the posterior tibialis tendon B. Abnormal insertion of the abductor halluces C. Exaggerated plantar insertion of the tibialis anterior tendon D. Abnormal insertion of the peroneus brevis tendon

C - Exaggerated plantar insertion of the tibialis anterior tendon,B - Abnormal insertion of the abductor halluces,A - Contraction of the posterior tibialis tendon. Correct answer: (A) (B) (C) Contraction of the posterior tibialis tendon, abnormal insertion of the abductor halluces and exaggerated plantar insertion of the tibialis tendon. Explanation: Metatarsus adductus is one of the most common foot deformities. It is a deformity of the transverse plane in which the metatarsus is in an adducted position relative to the longitudinal axis of the lesser tarsus. Some suggested anatomic causes include exaggerated insertion of the tibialis anterior tendon, contraction of the posterior tibialis tendon, and abnormal insertion of the abductor halluces.

IDSA guidelines of diabetic foot infections require urgent surgical interventions with: (Select all that apply) A. An abscess B. Gas in the deeper tissues C. Necrotizing fasciitis D. Substantial nonviable tissue E. Extensive bone involvement

C - Necrotizing fasciitis,B - Gas in the deeper tissues,A - An abscess. Correct answer: (A) (B) (C) An abscess, gas in the deeper tissues and necrotizing fasciitis. Explanation: Wounds with extensive non-viable tissue or osteomyelitis still need debridement, but are less urgent scenarios.

The weight-bearing status following a Lapidus fusion should be: (Select all that apply) Six weeks NWB with a SLC, if three solid cortical screws are used. Immediate and full WB in a fracture boot, if an interfragmentary screw and medial locking plate is used. WB at four weeks, with crossed cannulated screws and the addition of bone marrow aspirate. Four weeks NWB with a SLC, if a dorsal locking plate is used.

C - WB at four weeks, with crossed cannulated screws and the addition of bone marrow aspirate.,B - Immediate and full WB in a fracture boot, if an interfragmentary screw and medial locking plate is used.. Correct answer: (B) (C) Immediate and full WB in a fracture boot, if an interfragmentary screw and medial locking plate is used and WB at four weeks, with crossed cannulated screws and the addition of bone marrow aspirate. Explanation: Weight-bearing following fusions, be it a toe or an ankle, is evolving, and it is important for a board candidate to realize the changes when answering an oral board question. In the ":old days", the standard was 6-8 weeks NWB in a short leg cast (SLC). But those were the days of crossed wires or pins. Now, WB status is dictated largely, but not entirely, by the type of fixation utilized. Things like patient compliance and BMI are also considerations. Two examples from McGlamry are, one, NWB in a short leg cast for two weeks followed by 4-6 weeks full WB in a fracture boot if three solid cortical screws are used. Second, if an interfragmentary compression screw and a medial locking plate are used, immediate and full WB in a fracture boot is used. In Clinics in Podiatric Medicine and Surgery, several examples are given for earlier WB. Examples include protective WB at two weeks with two or three crossed screws without plating; WB at two weeks with locking plate fixation; partial WB at five days, full WB at 2-3 weeks, and out of a protective boot at five weeks with crossed cannulated screws and a locking plate system, and bone marrow aspirate.

When applying a neutralization plate: A position screw is utilized to protect the plate from shear, torsion, and bending forces. A plate is utilized to protect the position screw from shear, torsion, and bending forces. A lagged screw is utilized to protect the plate from shear, torsion, and bending forces. A plate is utilized to protect the lagged screw from shear, torsion, and bending forces.

Correct Answer: D. A plate is utilized to protect the lagged screw from shear, torsion, and bending forces. Explanation: To begin, is the screw protecting the plate, or vice versa? The plate is protecting the screw from shearing, torsion, or bending. The screw is applied using the lag technique, but is unable to withstand much loading. In order to follow AO concepts, specifically early ROM followed by as early as possible protected WB ( advocated by Ilizarov), a plate is applied to "neutralize" the forces. A lag screw can also be inserted through the plate. A primary indication for a neutralization plate is tibial fractures, with the exception of transverse fractures. If an associated butterfly fragment is present, lag screws first fix the butterfly fragment, followed by the plate, which conducts stresses from one main fragment to the other.

Which inhaled anesthetic has the following characteristics? - MAC = 6.0 - requires a special vaporizer - low solubility - pungent odor - causes airway irritation A. sevoflurane B. halothane C. isoflurane D. desflurane

Correct Answer: D. Desflurane. Explanation: Desflurane is a volatile halogenated agent with a minimum alveolar concentration of 6.0. It is used as a maintenance inhaled anesthetic. Desflurane is a liquid at room temperature. With that said it requires a special vaporizer which heats it to a consistent temperature which allows desflurane gas to be available at a constant pressure. It has the most rapid onset and most rapid offset of all the inhalation anesthetics due to its low solubility. Some negatives of desflurane include a pungent odor, low potency and high cost. Additionally it may cause airway obstruction or tachycardia if given at too strong a concentration. Due to the airway irritability it is seldom used as an induction inhalation agent.

The sagittal axis at the first metatarsophalangeal joint allows movement in this plane? A. Transverse plane B. Sagittal plane C. Transverse and sagittal plane D. Frontal plane E. Frontal plane and sagittal plane

Correct answer: (A) Transverse plane. Explanation: The first metatarsophalangeal joint has two distinct axes of motion or degrees of freedom. The transverse or horizontal axis provides pure sagittal plane motion, and the vertical axis provides pure transverse plane motion. The transverse axis of the first metatarsal phalangeal joint is an axis whose position moves with sagittal plane motion of the joint.

This is the most common hyperdactyly variant in the foot. A. Central polydactyly B. Preaxial polydactyly C. Central syndactyly D. Postaxial polydactyly

Correct Answer: D. Postaxial polydactyly. Explanation: Polydactyly is a common congenital deformity defined as excess of five digits in humans. It has racial and geographic predilections and may occur as an isolated deformity or less commonly, in association with certain congenitally inherited syndromes. Postaxial polydactyly is the most common hyperdactyly variant and denotes lateral digital duplication, whereas preaxial is applied to medial digital duplication. Central polydactyly is the least common and involves duplication of the innermost digits most often the second toe.

There are a total of five metatarsals, which are numbered from I to V in a medial to lateral fashion. Each metatarsal is a long bone presenting with a base, shaft, and head. More commonly, one refers to the protrusion of a metatarsal in the articulated foot in relative length, what is the relative length of the metatarsals in the articulated foot? A. 2>3>1>4>5 B. 1>2>3>4>5 C. 2>3>4>5>1 D. 3>2>1>4>5

Correct answer: (A) 2>3>1>4>5. Explanation: The formula most commonly cited referring to the relative length of the metatarsals is 2 (longest) >3>1>4>5 (shortest). There are a total of five metatarsals, which are numbered from I to V in a medial to lateral fashion. Each metatarsal is a long bone presenting with a base, shaft, and head. The metatarsals extend anteriorly from the foot and are nearly parallel to each other (0°- 8°). The first metatarsal is the shortest in absolute length while the second is the longest. Straus showed that the first metatarsal is approximately 83 percent of the length of the second metatarsal. More commonly, one refers to the protrusion of a metatarsal in the articulated foot (relative length). For example, the second metatarsal protrudes the most distal and is considered to be the longest. Because the fifth metatarsal does not protrude as far as any of the other metatarsals, in the articulated skeleton, it is considered the shortest. The formula most commonly cited referring to the relative length of the metatarsals is 2 (longest) >3>1>4>5 (shortest).

How many degrees does the hip flex in contact phase? A. 30 degrees B. 25 degrees C. 45 degrees D. 35 degrees

Correct answer: (A) 30 degrees. Explanation: The hip is flexed about 30 degrees and experiences a flexor torque at contact, which is resisted by the gluteus maximus and adductor magnus. Extension is produced as the knee and ankle plantarflex, which advances the femur forward faster than the pelvis, resulting in a relative extension. Motion at the pelvis is minimal.

There are many components of a shoe. Which aspect is the anterior portion of the upper that covers the toes and forefoot? A. Vamp B. Quarter C. Toe box D. Shank

Correct answer: (A) Vamp. Explanation: The vamp is the anterior portion of the upper that covers the toes and forefoot. (B) The quarter is the posterior aspect of the upper. (C) The toe box is the most distal dorsal aspect, which gives stiffness and structure around the toes. (D) The shank is a component of the outersole, which extends from the anterior aspect of the heel to the ball of the shoe. The shank provides longitudinal stability and support.

Osteoarthritis is: A. A hypertrophic joint disease B. An atrophic joint disease C. A destructive joint disease D. Primarily an inflammatory condition

Correct answer: (A) A hypertrophic joint disease. Explanation: One of the ways joint disorders can be distinguished from one another is based on radiographic features of the joint spaces. Osteoarthritis features include bone overgrowth and enlargement, subchondral sclerosis and osteophyte formation at the joint margin. These findings are important in distinguishing the condition from rheumatoid arthritis or other joint disease processes, which make the joint space smaller. Osteoarthritis is not a destructive joint disease: destructive joint diseases refer to disease processes that are lytic and acute in nature. Osteoarthritis is degenerative and evidence of the condition on x-ray examination takes years to develop. Finally, osteoarthritis is not primarily inflammatory in nature (pg. 360, Christman). If there is inflammation present it is usually mild and not a primary pathologic feature.

What is the most common type of fifth metatarsal fracture, based on anatomic location? A. Avulsion fracture of the fifth metatarsal base B. Jones fracture C. Zone 3 fifth metatarsal base fracture D. Transverse fifth metatarsal shaft fracture E. Dancers fracture (spiral oblique shaft fracture)

Correct answer: (A) Avulsion fracture of the fifth metatarsal base. Explanation: Avulsion fifth metatarsal base fracture, also called "pseudo-Jones fracture", accounts for 51.2% of fifth metatarsal fractures. (B) The true Jones fracture of the proximal diaphysis accounts for 13.6% of fifth metatarsal fractures. (C) Zone 3 metatarsal base fractures account for 5.7% of fifth metatarsal fractures. (D) Transverse shaft fractures at the metatarsal neck account for 16.8% of fifth metatarsal fractures. (E) Dancers fractures account for 11.1% of fifth metatarsal fractures.

This agent's mechanism of action is depression of the reticular activating system leading to loss of consciousness. The duration of action of this highly lipid-soluble agent is determined by redistribution rather than elimination by the liver or kidneys. What intravenous anesthetic agent is being described? A. Barbiturates B. Benzodiazepines C. Etomidate D. Ketamine

Correct answer: (A) Barbiturates. Explanation: The barbiturate intravenous anesthetics most commonly used are thiopental and methohexital. They are derivatives of barbituric acid. The work by depressing the reticular activating system which results in loss of consciousness. The duration of action of barbiturates, which are highly lipid-soluble, is determined by redistribution as opposed to elimination. In general, barbiturates are central nervous system depressants, and can therefore produce a wide spectrum of effects, from mild sedation to total anesthesia. Barbiturates can also effective as anxiolytics, hypnotics, and anticonvulsants. While barbiturates do have analgesic effects, these effects are relatively weak, therefore preventing them from being used in surgery without the addition of other analgesics such as opioids.

A 43-year-old male with Addison's disease of 20 years duration is on a maintenance dose of 30 mg. prednisone per day. Prior to surgery, how should this patient be treated? A. Be supplemented with additional hydrocortisone. B. Discontinue hydrocortisone. C. Be supplemented with thyroxin. D. Maintain his daily hydrocortisone dosage.

Correct answer: (A) Be supplemented with additional hydrocortisone. Explanation: Patients receiving long term prednisone therapy of 20-30 mg per day for more than 5 days are assumed to have suppressed adrenal function. There is a decrease in endogenous production of steroid from adrenals and stress from surgery requires an increased dose. Most podiatric procedures are classified as minimal to moderate surgical stress operations. For steroid- dependent patients who are having forefoot surgery (minimal stress), a regimen of 100 mg of hydrocortisone intramuscularly or intravenously, or double the usual dose the day of the surgical procedure, may suffice for supplementation. After the day of surgery, patients resume their regular steroid regimen. Hydrocortisone should not be discontinued, because the increased amount of stress on the body can cause the patient to have a hypotensive crisis. Hydrocortisone should also not just be maintained, but an increased dose should be given secondary to increased stress from surgery. Thyroxin is a thyroid drug and does not help with increasing steroid production.

The best option for a 45 y/o patient with a failed first MPJ total implant arthroplasty secondary to infection is: A. Bone block first MPJ arthrodesis B. Orthotic device C. Re-insertion of a second implant D. Leaving the joint as an excision arthroplasty (Mayo-Heuter)

Correct answer: (A) Bone block first MPJ arthrodesis. Explanation: The patient is young, and requires a procedure which will stand the test of time. Probably an implant was not the best choice to begin with. A two-stage surgery is indicated. The first stage involves removal of the implant, cultures taken with the patient off all antibiotics prior to surgery, insertion of an antibiotic-impregnated cement block, and fusion with structural grafting six weeks later. Allograft can be used, but with revision surgery, the iliac crest graft with its inherent stem cells is probably best, despite the problems associated with crest grafting. The proximal tibial metaphysis can also be used. (B) Again, the patient is young. Any type of accommodative orthotic device usually meets with a very low satisfaction rate with patients. And, as a tip, you are taking a surgical board exam; they are probably looking for a surgical solution. (C) Debate continues as to whether re-implantation should be attempted in the presence of infection. Old school thought was NEVER; newer thinking has insertion of a new implant acceptable after the infection is resolved; the question remains however-when is the infection resolved? And, was the infection causing the pain, or was it something else inherent to the implant itself and with re-insertion, will the same problem occur again. In Orthopedics, infected hips are re-implanted after the quadrangle of clinical evaluation, serological investigation, microbiological determination, and diagnostic imaging determine the infection is resolved. Re- implantation can occur anywhere from six weeks to three months. So, although stated as "incorrect" in this question, consideration can be given to this option in the right scenario. (D) A Mayo-Heuter joint resection would leave the patient with a short, non-functional toe, and would only be acceptable in perhaps a geriatric patient with an apropulsive, shuffling gait.

A 11 year old patient presents to the office complaining of foot pain. After physical examination you diagnoses the patient with a tarsal coaltion. Which of the following is the most common tarsal coalition present in at the age of 11? A. Calcalno navicular bar B. Talonavicular coalition C. Calcaneocuboid coalition D. Talocalcaneal coalition

Correct answer: (A) Calcalno navicular bar. Explanation: Calcaneonavicular bar is the most common during the age group of 8-12. Talonavicular coalition is more common in the age group of 3-5 and talocalcaneal from age 12- 16. Calcaneonavicular bar is the most common colition.

What term in a research study is used to refer to all the people who enter a study regardless if they are treated, exposed, develop the disease, or drop out of the study before its conclusion? A. Source population B. Inclusion criteria C. Diseased population D. Study population

Correct answer: (D) Study Population. Explanation: The study population, also known as patient population, is used to refer all the people who enter a study regardless if they are treated, exposed, develop the disease, or drop out of the study before its conclusion. The study population typically originates in a source population, which is narrowed via exclusion criteria.

What is the closed kinetic chain supination at the subtalar joint? A. Calcaneal inversion with talar abduction and dorsiflexion B. Calcaneal inversion, with talar adduction and plantarflexion C. Calcaneal eversion, with talar dorsiflexion D. Calcaneal plantarflexion, inversion and adduction

Correct answer: (A) Calcaneal inversion with talar abduction and dorsiflexion. Explanation: Closed kinetic chain supination consists of calcaneal inversion with talar abduction and dorsiflexion. This appears clinically as a foot facing inward, with a "stacking" of the talus over the calcaneus producing a "higher, thinner" appearance to the foot when viewed in the frontal plane. Conversely, closed kinetic chain pronation consists of calcaneal eversion along with talar adduction and plantarflexion. This appears clinically as a foot facing outward, with a "lowering" of the talus from the calcaneus producing a "shorter, wider" appearance of the foot when viewed in the frontal plane.

Which of the following is the most correct sequence of events in the stance phase of the normal gait cycle? Calcaneus inversion, foot passes through neutral position, calcaneal eversion, foot passes through neutral postion. Foot passes through neutral position, calcaneal eversion, calcaneal inversion, foot passes through neutral position. Calcaneal inversion, foot passes through neutral position, calcaneal inversion, calcaneal eversion. Calcaneal eversion, foot passes through neutral position, calcaneal inversion, foot passes through neutral position.

Correct answer: (A) Calcaneus inversion, foot passes through neutral position, calcaneal eversion, foot passes through neutral postion. Explanation: In the normal gait cycle stance phase, the heel initially hits the ground inverted (usually 2-3 degrees). At this time internal rotation of the leg occurs and the foot passes through subtalar joint neutral position for the first time. The foot then pronates to approximately 4 degrees everted. The foot is maximally pronated at the end of the contact phase. There is then an external rotation of the weight-bearing limb which initiates supination. The foot is then moving in the direction of supination and passes through neutral position for the second time and finally re-supinates and gets ready for the propulsive phase. Any deviation in the above is usually considered a pathomechanical abnormality.

This method of osseous fixation is described as encircling of part with a loop; it involves the circular application of malleable wire to stabilize fracture/osteotomy not amenable to other internal fixation methods. B. Transfixation C. Percutaneous pinning D. Tight rope E. Vassal's wire technique

Correct answer: (A) Cerclage wire. Explanation: Defined as the encircling of a part with a ring or loop. Cerclage involves the circular application of various diameters of malleable wire to stabilize fractures not amenable to other forms of internal fixation. Cerclage can be useful when exposure to a fracture is limited, yet interfragmentary compression is required for healing. Cerclage may be the only option for stabilization of fractures or osteotomies in patients with osteoporotic bone, in which screw purchase is inadequate and intramedullary fixation may not be indicated. A modification of the cerclage technique involves the transosseous or intraosseous passage of malleable wire. This technique allows for interfragmentary compression to be generated across transverse osteotomies or fractures that are situated in a juxtaarticular fashion.

Tension band fixation is one of the technique used in internal fixation. One of the following fractures is least likely to be fixated and stabilized by a tension band fixation: A. Comminuted fracture B. Oblique fracture C. Spiral fracture D. Non-union fracture

Correct answer: (A) Comminuted fracture. Explanation: This is borrowed from the engineering principle of tension band fixation, its application in the internal fixation of eccentrically loaded bone. In order to restore the load- bearing capacity of an eccentrically loaded fracture bone and minimize the forces born by the fixation device it is necessary to absorb the tensile forces, the result of the bending moment, and convert them into compressive forces. A tension band is therefore a device which will exert a force equal in magnitude but opposite in direction to the bending force. The tension must be made of a material which resists tensile forces and which can be prestressed. The bone must be able to withstand compression. This means that the bone must not be comminuted either under or on the opposite side of the tension band.

This pediatric foot and ankle deformity presents with moderate to severe talar deformity and all its relationships are abnormal with subluxation of the talocalcaneal joint, underdevelopment of soft tissue structures to the medial side of the foot, underdevelopment of the calf muscle, ligaments of the forefoot are normal. A. Congenital clubfoot B. Metatarsus adductus C. Calcaneovalgus D. Tibial torsion

Correct answer: (A) Congenital clubfoot. Explanation: Also known as equinovarus congenita, congenital talipes equinovarus, talipes congenital, talipes valgus, talipes varus congenital deformity frequently includes the foot and ankle, leg deformity and contractures. This includes deformities of the foot and ankle. The forefoot is adducted and plantarflexed at the Lisfranc's and/or midtarsal joints. The Rearfoot is inverted at the subtalar joint. And the ankle joint is plantarflexed. (B) Metatarsus adductus is defined as a transverse-plane deformity occurring at Lisfranc's (tarsometatarsal) joint in which the metatarsals are deviated toward the midline (medially) of the body. Often, it is associated with internal tibial torsion. (C) Calcaneovalgus is defined as a congenital flexible flatfoot deformity usually present at birth. It affects females more frequently than males, can be unilateral or bilateral, and is present in 1 in 10 live births. The deformity consists mainly of extreme dorsiflexion of the foot and calcaneal valgus. It is one of the most common foot deformities. Fortunately, it is generally flexible and has an excellent prognosis if treated early and appropriately. (D) Internal tibial torsion is an abnormal increase in tibial version in an internal direction. It is a transverse-plane deformity caused by a fixed structural abnormality occurring in the tibia. This results in an abnormal angle between the knee and ankle axes for a given patient age. Essentially, there is a medial rotation of the distal tibia on the proximal tibia.

Which of the following is true of a patient with clubfoot deformity? A. Decreased talocalcaneal angle B. Calcaneus is dorsiflexed C. Navicular is lateral to the head of the talus D. Abduction of the metatarsals can be present

Correct answer: (A) Decreased talocalcaneal angle. Explanation: You see a decrease in the talocalcaneal angle as there is almost full articulation of the talus and navicular. (B) With this deformity you tend to see a plantarflexion of the foot as a whole since you see the foot and ankle in equinus. Thus the calcaneus is plantarflexed and not dorsiflexed. (C) The navicular is medial to the head of the talus. The navicular tuberosity is so far medial that sometimes it can touch the medial malleolus in severe cases. (D) You will not see abduction of the metatarsals. You do sometimes see an adduction of the metatarsals.

A patient presents for hardware removal of the right second digit. The patient had an arthroplasty with silicone implant a decade ago and now complains of pain to the second MTPJ. You elect to remove the silicone implant. During surgery you notice: A. Detritic synovitis B. Osteomyelitis of the right second digit C. Metallosis D. Intact silicone implant

Correct answer: (A) Detritic synovitis. Explanation: Historically, the most common material producing continued inflammation in the foot and ankle surgery are the silicone elastomers. Small particles of this material generated from prostheses have been implicated in inflammatory reaction to adjacent synovium. Further spread of these particles to the lymphatic vessels have been claimed to be associated with lymphadenopathy, fever, and even lymphoma. Detritic synovitis is a phenomenon believed to be a foreign body reaction to shards of silicone materials. During the early use of silicone implants in metatarsophalangeal joint surgery, several studies on detritic synovitis showed that silicone particles are found in bone, synovium, and regional lymph nodes with migration of silicone through the vascular and lymphatic channels. Histologic changes included hyperplasia of the synovial membrane with silicone granulomas and aggregate of inflammatory cells composed of lymphocytes, histiocytes, plasma cells, and foreign body giant cells. Further histologic examination revealed intracortical lysis and fracturing of the phalangeal cortex with varying degrees of osseous destruction including areas of lysis as well as multiple cyst formation in adjoining bones, all of which significantly increased over time. Histopathological studies by Schilero and McCarthy et al. have demonstrated that fissuring, fracturing, microfragmentation and subluxation of the implants had occurred after they had been present for 1 year. This process freed shards of silicone elastomer causing gross reaction within the synovium leading to fibrous hyperplasia. In addition, Schilero reported that subsequent bone cyst formation occurred and a local immune response involving T cell lymphocytes and foreign body giant cells was seen.

In order to derotate the aductovarus fifth digit below what orientation should the lenticular incision be made? A. Distal medial to proximal lateral B. Distal lateral to proximal medial C. Lateral to medial D. Medial to lateral

Correct answer: (A) Distal medial to proximal lateral. Explanation: The distal medial to proximal lateral elliptical incision when lose will promote a valgusization of the digit and derotate the deformity. The opposite orientation, distal lateral to proximal medial, would provide a further varus deformity. The lateral to medial and medial to lateral are one in the same and would simple provide some correction in the sagittal plane.

What is the open kinetic chain pronation at the subtalar joint? A. Dorsiflexion, eversion, abduction B. Dorsiflexion, inversion and plantarflexion C. Plantarflexion, inversion and adduction D. Plantarflexion and adduction

Correct answer: (A) Dorsiflexion, eversion, abduction. Explanation: Open chain pronation consists of dorsiflexion, abduction, and eversion. The resultant position of an open chain pronated subtalar joint is a foot pointing up, away from the midline, and facing outward

These are the most common adverse events associated with intravenous administration of vitamin K during reversal of anticoagulation. A. Dyspnea and chest tightness B. Burning during infusion C. Diaphoresis D. Redman syndrome

Correct answer: (A) Dyspnea and chest tightness. Explanation: Intravenous vitamin K will provide a more rapid reversal than any other routes of administration. A retrospective study found that 1 mg of IV vitamin K reversed anticoagulation to an INR <1.4 in a median time of 27 hours. The most notable adverse events to IV vitamin K were dyspnea and chest tightness during infusion, which occurred in approximately 2% of patients. Vitamin K given orally is the most common method of administration and does not provide the same potential side effects as the intravenous vitamin K. Dosages of 1.0 mg to 2.5 mg of oral vitamin K will correct supratherapeutic INR >4.5 to a therapeutic range within 24 to 48 hours. Oral vitamin K is only available in 5-mmg tablets in the United States; these tablets are scored allowing for easy administration of 2.5 mg. Lower doses can be administered by having the patient drink the intravenous preparation.

What is the common denominator for acute compartment syndrome? A. Elevated interstitial fluid pressure causing vascular occlusion B. Tachycardia C. Increases BP D. Increased temperature out of proportion

Correct answer: (A) Elevated interstitial fluid pressure causing vascular occlusion. Explanation: The sequelae of a compartment syndrome may be devastating - Volkmann's contracture, neural deficit, and even gangrene. The common denominator of compartment syndromes is an elevated interstitial fluid pressure causing vascular occlusions in the compartments containing muscle. The remaining choices are not components of the common denominator of compartment syndrome.

What is the insertion of the extensor digitorum brevis? A. Fibrous expansions of the long extensor tendon at the level of the MTPJ B. Fibrous extensions of the long flexor tendon at the level of the MTPJ C. Fibrous expansions of the long extensor tendon at the level of the PIPJ D. Fibrous expansions of the long flexor tendon at the level of the PIPJ

Correct answer: (A) Fibrous expansions of the long extensor tendon at the level of the MTPJ. Explanation: The tendons of the extensor digitorum brevis are found on the lateral aspect of the extensor digitorum longus tendons 2, 3, 4. Within the extensor expansion, the muscle belly passes in a distomedial direction and is relatively short; the tendons then pass to the dorsolateral aspect of the respective toe. The extensor digitrum brevis has only three tendon slips, one each of the second, third, and fourth digits.

Which of the following radiographic features is unlikely to be seen with gout? A. Fibular deviation of digits B. Punched out bone lesions C. Overhanging sclerotic bone D. Tophi

Correct answer: (A) Fibular deviation of digits. Explanation: On radiographic examination the initial finding may be soft tissue swelling. Crystalline deposition or tophi may be noted and can cause the development of punched out erosions with elevated bony margins forming the overhanging edge sign. The articular space is often well maintained. In the subcortical bone, a lacy pattern of bone erosion is pathognomonic of this disease process. Fibular deviation is most commonly seen along with ulnar deviation in rheumatoid arthritis.

What is the most likely etiology of the hammertoes pictured below? A. Flexor Stabilization B. Extensor Substitution C. Flexor Substitution D. Extensor Stabilization

Correct answer: (A) Flexor Stabilization. Explanation: Flexor stabilization hammertoes occur when the long flexor tendons fire for a longer duration in the gait cycle to assist a failing tibialis posterior tendon in a pronated foot. The long flexors overpower the interossei. This then pulls the distal phalanx of the digit toward the sulcus of the foot therefore causing buckling of the digit. This is the most common cause of hammertoes. Extensor substitution hammertoes are commonly encountered in cavus and anterior cavus foot types. Due to the plantarflexed nature of the forefoot the long extensor tendons work for a longer period of time in the gait cycle to facilitate the clearance of the toes to the ground. This causes an uneven tug-o-war phenomenon and the long extensors overpower the long flexors. This deforms the toes and leaves very prominent extensor tendons tenting the skin on the dorsum of the foot. Flexor substitution hammertoes occur when the long flexors fire longer in the gait cycle to compensate for weak triceps surae. This again overpowers the long extensors and deforms the toe.

This motion doesn't exist at the first metatarsophalangeal joint? A. Frontal plane motion B. Sagittal plane motion C. Transverse plane motion D. Transverse and sagittal plane motions

Correct answer: (A) Frontal plane motion. Explanation: The first metatarsophalangeal joint has two distinct axes of motion or degrees of freedom. The transverse or horizontal axis provides pure sagittal plane motion, and the vertical axis provides pure transverse plane motion. The transverse axis of the first metatarsal phalangeal joint is an axis whose position moves with sagittal plane motion of the joint. As arthrodial motion of the joint begins at 20 degrees of dorsiflexion, the transverse axis shifts dorsally and proximally within the metatarsal head. The vertical axis lies in the sagittal and frontal planes, hence, only transverse plane motion will occur about it. No normal frontal plane motion of the first metatarsophalangeal joint is said to occur at this joint.

A 16-year-old girl is a Jehovah's Witness who states prior to a surgical procedure that she refuses any transfusions. Her parents are also Jehovah's Witnesses. The patient expresses an understanding of the potential consequences of her decision. The surgery is a low-risk elective procedure, but during surgery, she experiences a life-threatening hemorrhage. Which of the following statements is true? A. Give blood transfusion because the patient is a minor. B. Obtain court order for blood transfusion, since it is in the patient's best interest. C. Try repeatedly to obtain consent from parents. D. Do not perform transfusion. E. Perform transfusion because this situation was unexpected when the patient gave consent and refused treatment.

Correct answer: (A) Give blood transfusion because the patient is a minor. Explanation: Patient autonomy is the right of a competent and informed patient to determine his or her own healthcare decisions, even when doing so may be harmful. However, children under 18 years of age are minors and legally incompetent. Parents of a minor cannot withhold treatment to save life or limb from their children, and if there is an immediate emergency, the patient should be treated appropriately. If the danger is not immediate but still critical, a court intervention or referral to an ethics committee may be indicated.

A patient presents with rigid hammertoes, with flexion at the PIPJ and painful corns. The biomechanical exam demonstrates a flexor substitution type foot. The best digital procedure for this patient is: A. Higgs B. Post C. Girdlestone-Taylor D. Terrier

Correct answer: (A) Higgs. Explanation: A flexor substitution type of foot is a high arched foot secondary to a weak triceps surae and overpowering of the flexors against weak interossei muscles. The tibialis posterior, long flexors, and peroneus longus are now firing earlier and longer, to compensate for the weak triceps surae. The result is a high arched foot, with little rearfoot motion and a poor shock absorber. The goal with digital surgery in this type of foot is to create a rigid beam of the toe. This can only be accomplished with a digital fusion, not arthroplasty; the Higgs is one example, a peg-in-hole type of digital fusion. (B) The Post is a proximal phalangeal head resection; an arthroplasty. With the severe deforming forces of flexor substitution, the toe would not be able to become a rigid beam at propulsion. (C) The Girdlestone-Taylor procedure is a flexor-to-extensor tendon transfer. It can be used as a sole procedure with a flexible deformity; this deformity is rigid; the tendon transfer will not work in this scenario. (D) The Terrier is also an arthroplasty procedure involving the proximal phalangeal head and middle phalangeal base. Again, the rigid beam could not be produced with this cavus type foot. A flail toe is a distinct possibility.

What is the greatest risk for perioperative stroke? A. History of prior stroke B. Cardiac arrhythmias C. Hypertension D. History of diabetes

Correct answer: (A) History of prior stroke. Explanation: Patients with a history of stroke, they often have a history of hypertension, diabetes, dyslipidemia, and cardiac arrhythmias. Workup should include ECG, echocardiogram and stress tests. They often have vascular disease with significant end organ damage. A complete blood count and complete metabolic panel should be drawn up to evaluate for such injury. The greatest risk factor for operative stroke is a history of prior stroke. (B) (C) (D) The patients with cardiovascular disorders often have a substantial history of cardiac arrhythmias (B), hypertension (C), diabetes (D), dyslipidemia. Prior to surgery, these patients must have cardiac testing including ECG, echocardiography, and possibly cardiac stress testing. These may be risk factors for stroke, if left uncontrolled, but the greatest risk for stroke remains, a prior stroke.

Which one of the following is recommended as DVT therapy for an actively bleeding postoperative patient? A. Intermittent pneumatic compression B. Low-dose unfractionated heparin (LDUH) C. Lower molecular weight heparin (LMWH) D. Fondapirinux

Correct answer: (A) Intermittent pneumatic compression. Explanation: VTE prophylaxis options can be broadly classified into 2 categories: mechanical and pharmacologic. Mechanical choices include graded compression stockings, intermittent pneumatic compression devices, venous foot pumps, and even inferior vena cava filters. Mechanical options pose no risk of bleeding. They are an appropriate choice for postoperative surgical patients with a substantial risk of bleeding such as those with recent hemorrhage or neurosurgery patients. Intermittent pneumatic compressing devices and graded compression stockings have shown efficacy in VTE prevention in patients after general or orthopedic surgery systematic reviews. The latest American College of Chest physician's guidelines advocate mechanical prophylaxis in patients with contraindications to pharmacologic agents.

Please refer to the clinical picture below. Which of the following statements in regard to the clinical image is not correct? It is a result of unchecked proliferation of the fibrous tissue following injury to the skin and remains confined to the site of the original skin injury. It invades the surrounding normal skin and is usually worse in areas where the skin is under tension or is thickest. Borders are usually well demarcated, but may be irregular in outline. It shows an increased level of histamine which may explain why they are often associated with pruritus, hyperesthesia and pain.

Correct answer: (A) It is a result of unchecked proliferation of the fibrous tissue following injury to the skin and remains confined to the site of the original skin injury. Explanation: The clinical image above is a classic example of a keloid scar. The first statement describes a hypertrophic scar. Hypertrophic scars are a result of unchecked proliferation of the fibrous tissue following injury to the skin and remains confined to the site of the original skin injury. Keloid scars, on the other hand, invades the surrounding normal skin and is usually worse in areas where the skin is under tension or is thickest. A keloid's borders are usually well demarcated, but may be irregular in outline. It shows an increased level of histamine which may explain why they are often associated with pruritus, hyperesthesia and pain.

Which of the following statements is true regarding the Weil lesser metatarsal osteotomy? It is intra-articular, extending from distal dorsal aspect of articular cartilage proximally to plantar cortex, parallel to weightbearing surface. It is a chevron osteotomy of the metatarsal neck. It is extra-articular, extending from the metatarsal neck proximally to plantar cortex. It is a metatarsal base osteotomy. It is a dorsiflexory wedge osteotomy of the metatarsal neck.

Correct answer: (A) It is intra-articular, extending from distal dorsal aspect of articular cartilage proximally to plantar cortex, parallel to weightbearing surface. Explanation: The Weil osteotomy is an intra-articular osteotomy that is effective for lesser metatarsal pathology due to its inherent stability, robust blood supply at the metatarsal head, and its ability to treat triplane metatarsophalangeal joint deformity. Although this osteotomy does provide dorsiflexion, it is not technically a dorsiflexory wedge osteotomy.

A 12 year old female has a painful hallux valgus. X-rays show that the first metatarsal head is round. The inter metatarsal angle is 21 degrees and the hallux abductus angle is 40 degrees. The first ray is extremely hypermobile in the sagittal plane. Significant progression of the deformity over a 18 month period has been documented. Given this data, which of the following procedures is indicated? A. Lapidus procedure (first tarsometatarsal arthrodesis) B. Ludloff procedure (oblique first metatarsal osteotomy) C. Austin bunionectomy D. First metatarsal base epiphysiodesis

Correct answer: (A) Lapidus procedure. Explanation: The Lapidus procedure is indicated here. Transverse and sagittal stability are best provided by a metatarsocuneiform fusion. Intervention at this age can be justified by the docu- mented progression over a short period of time. (B) While the Ludloff procedure can accommodate the high IM angle, it does not deal with the excessive hypermobiility in the scenario above. (C) The Austin bunioneictomy cannot handle the high intermetatarsal angle and hallux abductus angle. (D) It is likely that there is not enough growth potential at age 12 to "grow out" this bunion deform-ity through an epiphysiodesis. Additionally the epiphysiodesis does not address the excessive hyper mobility.

A 12 year old female has a painful hallux valgus. X-rays show that the first metatarsal head is round. The inter metatarsal angle is 20 degrees and the hallux abductus angle is 38 degrees. The first ray has a high degree of hypermobility. Significant progression of the deformity over a 12 month period has been documented. From then options below what would be the best treatment option? A. Lapidus procedure B. McBride bunionectomy C. Austin bunionectomy D. Closing base wedge osteotomy

Correct answer: (A) Lapidus procedure. Explanation: The Lapidus procedure is the best treatment option listed above. The Lapidus can correct the high degree of deformity and provide stabilization of the hyper mobile first ray. This will provide a lasting correct and improved biomechanical performance of the foot. As documented significant progression of this deformity has been observed, surgical intervention is justifiable. (B) The McBride procedure, although not as invasive as the Lapidus procedure, is not indicated in the above scenario. It would be insufficient in correcting the high inter metatarsal angle and high hallux abductus angle. (C) The Austin bunionectomy, again not as invasive the Lapidus, is also not indicated for the above scenario. The Austin bunionectomy would not be able to provide sufficient correction for the high inter metatarsal angle and high hallux abductus angle. (D) The closing base wedge osteotomy is also not indicated due to the high degree of hyper mobility. The Lapidus procedure is indicated here. The McBride and Austin bunioneictomies cannot handle the high intermetatarsal angle and hallux abductus angle. The Keller procedure is joint destructive and will not allow a propulsive first ray in this young individual. Transverse and sagittal stability are best provided by a metatarsocuneiform fusion. Intervention at this age can be justified by the documented progression over a short period of time.

Which of the following is true regarding local anesthesia? Local anesthetics bind reversibly to sodium channels and increase the threshold for depolarization After giving local anesthesia, the sense of proprioception is lost prior to the sense of pain Adding epinephrine to a local anesthetic decreases duration of its effects Allergic reactions to amides are secondary to their metabolite PABA

Correct answer: (A) Local anesthetics bind reversibly to sodium channels and increase the threshold for depolarization. Explanation: Choice A describes the exact mechanism of how local anesthetics work by binding REVERSIBLY to the sodium channels increasing the threshold for depolarization. Choice B is incorrect as the first sense to be lost is pain then temperature then touch and then proprioception is the last to be lost. Choice C is incorrect because adding epinephrine to local anesthetics increases the duration of its effects. Choice D is incorrect because ester local anesthetics cause more allergies than amides and this is because the esters have PABA as their metabolite.

A patient presents to the podiatry office for a follow-up of right foot bunionectomy. The procedures completed were an Austin osteotomy and a lateral release. Patient is complaining of numbness in the dorso lateral aspect of the hallux. Which of the following nerves were most likely damaged during dissection? A. Medial terminal branch of deep peroneal nerve B. Medial dorsal cutaneous nerve C. Intermediate dorsal cutaneous nerve D. Saphenous Nerve

Correct answer: (A) Medial terminal branch of deep peroneal nerve. Explanation: The nerve innervation for the hallux is the medial plantar nerve both plantar medial and plantar lateral hallux. The dorsolateral aspect of the hallux is innervated by the medial terminal branch of the deep peroneal nerve and the dorsal medial aspect of the hallux is innervated by the medial dorsal cutaneous nerve. The saphenous nerve does not typically extend in to the distal toe and the intermediate dorsal cutaneous nerve is not involved in the innervation of the hallux.

Following arthrodesis of the 1st metatarsophalangeal joint, the patient presents with increased pain to the surgical site. A diagnosis of foreign body reaction was made due to the stainless steel fusion plate that was used. During a second surgery the soft tissue surrounding the implant was noted to be black and gray with evidence of flaking. This process is also known as: A. Metallosis B. Metallic shear C. Passivation D. Creep

Correct answer: (A) Metallosis. Explanation: Metallosis is the release of corrosion debris into the surrounding soft tissue. Passivation is production of corrosion resistance by a surface of reaction products. This layer is normally an oxide layer, which is impervious to gas and water. Creep is when a solid material slowly and permanently migrates or deforms under mechanical stresses that are below yield strength of the material.

Which of the following would be grounds to cancel elective foot surgery? A. Myocardial infarction 2 months prior to surgery B. Potassium levels of 4.3 mEq/L day before surgery C. Platelet level of 289,000 per microliter D. Blood Sugar level morning of surgery 131mg/dl

Correct answer: (A) Myocardial infarction 2 months prior to surgery. Explanation: Patients should not have elective surgery until > 6 months after having an MI as they are at a much higher cardiovascular risk at this time. Normal potassium levels range from 3.5 mEq/L to 5 mEq/L and in this case 4.3 is within normal limits and surgery should not be cancelled for that. Looking at platelet levels is important as well. A platelet level of 289,000 per microliter is within normal limits. Blood sugars the morning of surgery should be well controlled. Oral DM medications are stopped prior to the day of surgery but in most cases insulin can be continued at a reduced rate. A blood sugar of 131 mg/dl should not cancel elective surgery.

Peripheral nerves have the best chance for full recovery with which of the following types/class of injury? A. Neuropraxia B. Axonotmesis C. Neurotmesis D. Nerve avulsion followed by primary nerve repair

Correct answer: (A) Neuropraxia. Explanation: Neuropraxia is best described as the mildest type of focal nerve lesion that produces clinical deficits localized loss of conduction along a nerve without axon degeneration. It is caused by a focal lesion, usually demyelinating, but occasionally ischemic (when of shorter than a few hours' duration), followed by a complete recovery. Axonotmesis can best be described as interruption of the axons of a nerve that results in degeneration of its distal (peripheral) segment (wallerian degeneration) without appreciable damage to the supporting structures (endoneurium; perineurium; epineurium) of the nerve at the site of the injury. This allows for spontaneous and good regeneration potential. Neurotmesis is the most severe nerve injury is a partial or complete severance of a nerve, with disruption of the axon and its myelin sheath and the connective tissue elements.

Measuring the heel position in relation to the ground while the subtalar joint is in neutral position is known as: A. Neutral calcaneal stance position B. Resting calcaneal stance position C. Subtalar Joint Neutral Position in Stance D. Calculated Subtalar Joint Position

Correct answer: (A) Neutral calcaneal stance position. Explanation: Neutral calcaneal stance position (NCSP) is defined as the position of the calcaneal bisection relative to the ground with subtalar joint in neutral position. This should be measured with the patient standing in the patient's normal angle and base of gait. The Resting Calcaneal (RCSP), represents the frontal plane position of the rearfoot, in relation to the supporting surface, when the patient is standing in their angle and base of stance, in a completely relaxed position. Subtalar joint (STJ) neutral position is the position typically used by clinicians to obtain a cast representation of a patient's foot before fabrication of biomechanical functional orthosis.

Tension band principle suggests that an eccentric application of force through a segment produces areas of compression at the fracture site. On what side of the fracture should the plate be placed? A. On the tension side of the fracture B. On concave side of the fracture C. On the same side of the cortex D. On both concave and convex side of the fracture

Correct answer: (A) On the tension side of the fracture. Explanation: The tension application of force through a segment produces areas of compression and distraction at the fracture site. Application of a bone plate on the tension side converts the forces distracting the segments into forces of compression.

Which of the following statements regarding the debridement of infected bone in a diabetic ulcer with confirmed osteomyelitis is false? Once all the infected bone has been removed and wound closure has been obtained a minimum of six weeks of intravenous antibiotics is mandatory. When debriding cortical bone, the process is continued until punctate bleeding is visualized emanating from the cortical bone. This is called the Papineau sign. All soft, nonbleeding bone should be removed regardless of biomechanical consideration. Power tools such as a sagittal saw a cutting burr are preferable to handheld instruments such as chisels and rongeurs.

Correct answer: (A) Once all the infected bone has been removed and wound closure has been obtained a minimum of six weeks of intravenous antibiotics is mandatory. Explanation: When only healthy bone remains, only one week of appropriate antibiotics is necessary when wound closure has been obtained. This of course is predicated that the surgeon is confident that only healthy bone and soft tissue remain. Likewise wound closure would only be attempted if the surgeon was again confident that only healthy bone and soft tissue remained. Course of antibiotics longer than one week are not only unnecessary, but also excessive. It places the patient in further jeopardy to the complications of prolonged antibiotic therapy such as Clostridia difficilis, resistant organisms, allergic reactions, etc. When removing infected bone this act must take priority over biomechanic considerations. Post amputation biomechanics are irrelevant if infected bone remains as long term success and salvage is doomed. Using hand tools such as chisels and rongeus may crack and splinter the bone and that is rationale why power tools are advantageous in this situation. The Papineau sign or punctate cortical bleeding is the hallmark indicator reaching viable cortical bone.

What is the most common form of administration of vitamin K? A. Oral vitamin K B. Subcutaneous vitamin K C. Intravenous vitamin K D. Intradermal vitamin K

Correct answer: (A) Oral vitamin K. Explanation: Vitamin K given orally is the most common method of administration and does not provide the same potential side effects as the intravenous vitamin K. Dosages of 1.0 mg to 2.5 mg of oral vitamin K will correct supratherapeutic INR >4.5 to a therapeutic range within 24 to 48 hours. Oral vitamin K is only available in 5-mmg tablets in the United States; these tablets are scored allowing for easy administration of 2.5 mg. Lower doses can be administered by having the patient drink the intravenous preparation. Subcutaneous delivery of vitamin K used to be common place; it has been shown that absorption by this route is not predictable and thus no longer recommended. In emergent surgical procedure, reversal of anticoagulation is typically achieved with the administration of fresh frozen plasma or vitamin k. FFP has an immediate onset of action, but its effects are short-lived. And it does not cause resistance to anticoagulation with warfarin postoperatively. The INR should be monitored after initial administration of FFP and every few hours subsequently to determine if additional treatment is needed to achieve the INR goal. For less urgent situations like this case, in a surgery to be performed within 24 to 96 hours, vitamin K is the treatment of choice. Intravenous vitamin K will provide a more rapid reversal than any other routes of administration. A retrospective study found that 1 mg of IV vitamin K reversed anticoagulation to an INR <1.4 in a median time of 27 hours. The most notable adverse events to IV vitamin K were dyspnea and chest tightness during infusion, which occurred in approximately 2% of patients.

Which of the following is found along the inferior aspect of the hallux interphalangeal joint, is centrally located, and is a normal variant? A. Os interphalageus B. Os intermetatarseum C. Os vesalianum D. Os supratalare E. Os peroneum

Correct answer: (A) Os interphalageus. Explanation: Classically found along the inferior aspect of the hallux interphalangeal joint. It is rare to see it in the lesser toes. The position of the os interphalangeus is either central or eccentric, central being more common. These ossicles are round and oval. It can be identified in either the dorsoplantar or the lateral view of the hallux. It is superimposed on the proximal phalanx head and appears as fairly well defined oval of increased density. It is rarely bipartite.

The complications with stump neuroma resection are largely very similar to this of primary neuroma resection. Stump neuroma resection is most often performed from a plantar incisional approach. From the choices below what is a potential complication that is unique to stump neuroma resection? A. painful plantar scar B. metatarsalgia C. subluxation of adjacent metatarsophalangeal joint D. reflex sympathetic dystrophy

Correct answer: (A) Painful plantar scar. Explanation: The most common complication after stump neuroma excision is recurrence. Continued pain after stump removal resection has been reported to occur up to 47% of the time. Failure of stump neuroma resection shares many of the same factors as does failure of primary neuroma resection. The factors include the wrong diagnosis, incorrect webspace, unrealistic patient expectations and inadequate surgical excision. One complication that is unique to stump neuroma resection is the formation of a painful plantar scar. As mentioned in the stem of the question, it is generally accepted to perform stump neuroma resection from a plantar longitudinal incision. Plantar incisions that travel against the relaxed skin tension lines are highly susceptible to thick, painful scars and callus formation.

Name all the muscles found in the 4th layer of muscles in the foot: A. Plantar and dorsal interossei B. Adductor muscle, adductor digiti minimi quinti brevis, flexor hallucis brevis C. Abductor digiti minimi, abductor hallucis, flexor digitorum brevis D. Quadratus plantae and lumbricales

Correct answer: (A) Plantar and dorsal interossei. Explanation: The 3 plantar interosseous muscles are unipennate, originating from the plantar aspect of the bases and medial aspect of the 3rd, 4th, and 5th metatarsals. Each muscle inserts in the medial aspect of the base of the of the proximal phalanx and extensor expansion of the respective digit, the tendons pass deep to the extensor expansion. The four dorsal interosseous muscles are bipennate. The 1st dorsal interosseous attaches to the entire tibial surface of the second metatarsal bone and to the inferior surface of the fifth metatarsal phalangeal joint and the base of proximal phalanx of the fifth toe. This insertion is located between the attachment of the abductor tendon and the flexor tendon sheath.

Upon reviewing dorsoplantar and lateral foot projections, an anterior break in the cyma line is noted on both projections. A calcaneal inclination angle of 15o, and an obscured sinus tarsi are noted. What type of foot is this? A. Planus B. Rectus C. Cavus D. Equinovarus

Correct answer: (A) Planus. Explanation: Radiographically, the pes planus foot type is characterized by an anterior break in the cyma line in both the dorsoplantar and lateral weight-bearing projections. Additionally a low pitch of the calcaneus is noted. The area of the sinus tarsi is usually diminished or obliterated. Eversion of the calcaneus may result in the lateral calcaneal tubercle being slight elevated. The region of the sustentaculum tali will usually appear lowered and appear as a broader area of density instead of the thick dense line of the normal foot. This is a result of the pronated position and eversion of the calcaneus. Also because of the pronation, the tubercle of the cuboid will fail to exhibit the peroneal groove on the radiograph. An example is provided below.

For those patients with uncontrolled hyperthyroidism who are presenting for elective surgery: A. Postpone the surgery until they are stable on a medical regimen, and controlled B. Start the anti-thyroid medications the day of surgery to avoid a crisis C. It's incumbent to the anesthesiologist to start the anti-thyroid medications and have the Beta blockers ready the systemic effects of excess thyroid hormone D. Proceed with the surgery then start with the anti-thyroid medications

Correct answer: (A) Postponed the surgery until they are stable on a medical regimen, and controlled. Explanation: For patients with uncontrolled hyperthyroidism presenting for elective surgery, their surgical procedure should be postponed until they are on a stable medical regimen to reduce the risk of thyroid storm. For those presenting for urgent or emergent surgery, it is incumbent on the anesthesiologist to have ready access to drugs that block the systemic effects of excess thyroid hormone. Such drugs include Beta blockers, antithyroid medications, and iodine. Beta blockers not only directly inhibit sympathetic activation but also inhibit the peripheral conversion of T4 to T3. Thionamides, such as propylthouracil (PTU) and methimazole, are actively transported to the thyroid gland and inhibit further production of hormone. Furthermore, PTU inhibits peripheral conversion of T4 to T3. Finally inorganic iodide in excess will manifest an antithyroid action known as the Wolff-Chaikoff effect. Potassium iodide can also be given as either Lugol solution or saturated solution of potassium iodide and is usually administered preoperatively for thyoid surgery as it decreases the vascularity of the gland.

The effects of radiation on tissues depend on the following: Proliferative capacity and differentiation. No data are available to determine the effect of radiation of tissues. Among tissues, the central nervous system is always involved when tissues are subject to radiation. Tissues are rarely damaged by radiation.

Correct answer: (A) Proliferative capacity and differentiation. Explanation: According to the law of Bergonie and Tribondeau which states that the sensitivity of tissue to radiation depends on two factors: Proliferative capacity and differentiation. Tissues that are rapidly dividing are more sensitive to radiation than are tissues in which cells are dividing slowly or are dormant. Tissues that are fully differentiated are not as sensitive. And cell sensitivity is classified as high for lymphocytes, immature red blood cells, moderate for epithelial cells, low sensitivity for muscle and nerves. Radiosensitivity increases for cells which have a high rate of mitosis, and least specialized.

What local anesthetic property determines duration of the local anesthetic? A. protein binding B. pKa C. ionic potential D. lipid solubility

Correct answer: (A) Protein binding. Explanation: The physical and chemical properties that influence local anesthetic activity are lipid solubility, protein binding, and the physical-chemical parameter (pKa). The duration of the local anesthetic is determined by the degree of protein binding. The higher the degree of protein binding, the longer the duration of the local anesthetic. The protein binding in bupivicaine exceeds 90% while the protein binding of lidocaine is 64%. In clinical use this translates to bupivicaine being two to three times longer acting than lidocaine.

A patient is set to undergo foot and ankle surgery at your facility. His past medical history is significant for anklylosing spondylitis. Which of the following is correct? A. Pulmonary function testing is required B. The patient always presents with normal functional capacity C. Ankylosing spondylitis always spares the phrenic nerve D. Autonomic instability is uncommon

Correct answer: (A) Pulmonary function testing is required. Explanation: Severe spine deformities necessitate pulmonary testing to evaluate the extent of lung disease present. Scoliosis and ankylosing spondylitis are but two common examples of conditions which reduce functional residual capacity and vital capacity, and pulmonary function testing provides risk analysis information regarding the potential for extended ventilator support postoperatively. (B) Patients presents with severe spine diseases will present with reduced functional residual capacity and vital capacity. (C) Ankylosing spondylitis is a severe spine disease; in patients with this disease who are undergoing surgery, the phrenic nerve should be evaluated for proper assessment of the airway. (D) Autonomic instability is common with severe spine disease and is difficult to manage.

Which of the following procedures is best indicated for the cock-up fifth toe pictured below? A. Ruiz-Mora Procedure B. Thompson Procedure C. Post Procedure D. Lapidus Procedure

Correct answer: (A) Ruiz-Mora Procedure. Explanation: The Ruiz-Mora procedure is the only procedure listed above indicated for a cock- up fifth toe. It consists of an elliptical plantar incision, resection of the proximal phalanx and suturing of the incision to medially deviate and plantarflex the toe with closure. The Thompson procedure is indicated for an underlapping fifth toe. It consists of a Z-type incision dorsally over the proximal phalanx, the resection of the fifth proximal phalanx in its entirety, reefing of the capsule to fill the dead space and corrective closure of the Z incision. The Post procedure is for correction of the rectus hammertoe. It consists of resection of the head of the proximal phalanx through either a linear or transverse elliptical incision and reefing of the capsule with closure. The Lapidus procedure is indicated for severely overlapping fifth toes. It consists of a curvilinear incision over the dorm of the fifth tmp joint and proximal phalanx with a secondary more proximal incision over the extensor tendon. The extensor tendon is transacted proximally and bought through the distal incision. The dorsal, medial and lateral tmp capsule are released. The extensor tendon is wrapped around the phalanx from dorsal medial to plantar lateral and attached to periosteum of the head of the fifth metatarsal, soft tissue or muscle proximal to the fifth metatarsal head.

Besides scalpels, which surgical instrument is most commonly used for the dissection of tissues? A. Scissors B. Thumb forceps C. Hemostatic forceps D. Traction forceps

Correct answer: (A) Scissors. Explanation: Scissors are most commonly used for the dissection of tissues besides the scalpel. Scissors are designed to be used with the thumb and ring fingers in the finger rings, while the index finger stabilizes the hinge region of the instrument. The blunt tips of dissecting scissors are best used for spreading and cutting tissues. Scissor blades can be straight or curved. Straight blades are typically used near the wound surface, while curved blades are used in deep dissection. The curved design improves visualization of the cutting field, enhances directional mobility, and allows for tissue separation along smooth curves. Mayo scissors are designed to be used for cutting thick bands of fascia and ligament, while the lighter Metzenbaum scissors are designed for subcutaneous dissection and cutter lighter fascia and capsular structures. (B) Thumb forceps are used to grasp and pick-up tissues. They are made of two pieces of surgical steel attached at one end, with varying types of tips. (C) Hemostatic forceps are the primary instrument used for acquiring hemostasis during anatomic dissection. They are hinged instruments made up of a set of jaws, finger rings, and a locking mechanism. (D) Traction forceps are hinged instruments made up of a set of jaws for grasping tissues, finger rings, and a locking mechanism. The grasping aspect of the jaws are designed to provide traction without unnecessary pressure that may crush the object being grasped.

You are assigned to fix a transverse 1st metatarsal fracture. After proper reduction of the fracture, you decide to apply a self -compressing plate. It is a dynamic compression plate. You are asked by your attending to explain the theory behind the self- compressing plate, which one of the following is the correct theory? Self-compressing plates achieve axial compression by having screws inserted eccentrically through the oval holes, as far from the fracture as possible Self-compressing plates achieve axial compression by having screws inserted through oval holes, one screw proximal to the fracture site, and the other one distal to fracture site providing two points of fixation and stability Self-compressing plates achieve axial compression by screws inserted on the two most distal aspects of the fracture site, providing compression and stability Self-compression plates achieve axial compression by screws inserted one perpendicular to the fracture site and one perpendicular to the axis of the bone

Correct answer: (A) Self compressing plates achieve axial compression by having screws inserted eccentrically through the oval holes, far from the fracture as possible. Explanation: Self-compressing plates are plates which make it possible to achieve axial compression by combining screw hole geometry with screw insertion. The oval holes and eccentric placement of the screws accounted for its self-compressing property. To achieve axial compression by means of self-compressing plates, fractures have to be anatomically reduced. The screws are then inserted eccentrically through the oval holes with corresponding drill sleeves as far from the fracture as possible. This will result in axial compression of the fracture. Some examples of self-compressing plates are: the semi-tubular plate, the one-third tubular plate, the quarter-tubular plate. Lately the semi-tubular plate is being replaced by the DCP.

Which of the following measures of nutritional status can signify an increased risk for postoperative complications? A. Serum albumin less than 3.2g/dL B. Total lymphocyte count less than 6,000μL3 C. Weight loss of less than 5% in one month D. Elevated HDL cholesterol

Correct answer: (A) Serum albumin less than 3.2g/dL. Explanation: Malnutrition, as manifested by decreased serum albumin levels, is a powerful predictor of higher perioperative morbidity. A weight loss of more than 5% in one month or 10% over 6 months, a serum albumin less than 3.2g/dL and a total lymphocyte count of less than 3,000μL3 can signify an increased risk of postoperative complications. When HDL cholesterol is elevated this is usually a sign of good health and diet.

Which of the following statements are true regarding infected diabetic foot ulcers? A. Staphylococcus aureus is the most common pathogen B. Gram negative organisms are the most common pathogens C. Anaerobic organisms are the most common pathogens D. Streptococci are the most common pathogens E. Methacillin-resistant Staphylococcus aureus is the most common pathogen

Correct answer: (A) Staphylococcus aureus is the most common pathogen. Explanation: Staphylococcus aureus is the most common pathogen in infected diabetic ulcers, accounting for 79% of cases. Of these, 15-20% are methicillin-resistant (MRSA).

65 year old Hispanic male comes to the office for a pre-operative visit for a Lapidus procedure. The patient is currently taking Warfarin for his controlled Atrial Fibrillation. Patient has no history of any thromboembolic events. What is the recommended amount of time the patient should stop Warfarin prior to his surgery? A. Stop warfarin 5 days prior to surgery B. Stop warfarin 1 week prior to surgery C. Stop warfarin the day before surgery D. Stop warfarin the morning of surgery

Correct answer: (A) Stop warfarin 5 days prior to surgery. Explanation: It is recommended for a patient with a lower risk of thromboembolism to stop warfarin 5 days prior to surgery to decrease the risk of bleeding during the surgical procedure. If the patient were to be at a higher risk then bridging the patient with Heparin would be a stronger consideration. The other choices listed above (B, C, D) are not the proper time period for stopping Warfarin. Warfarin is usually restarted 24 hours after the surgical procedure with its regular dosing.

Following traumatic talar injury, there is concern for avascular necrosis of the talus. A radiographic finding suggestive that the bone has good healing potential is: A. Subchondral radiolucency in the body of the talus B. Subchondral opacity in the body of the talus C. Vertical fracture of the talar neck and subtalar joint discoloration D. Vertical fracture of the talar neck and subtalar joint and ankle joint dislocation

Correct answer: (A) Subchondral radiolucency in the body of the talus. Explanation: Subchondral radiolucency within the body of the talus is known as the Hawkins sign. This is a positive sign that represents adequate blood supply for the bone to heal without leading to avascular necrosis. (B) The Hawkins sign is characterized by subchondral radiolucency, not subchondral opacity. (C) A vertical displaced fracture of the talar neck with subtalar joint dislocation is Type II of the Hawkin's Classification. Type II, III and IV type injuries have increasing incidence of avascular necrosis. (D) A vertical displaced fracture of the talar neck with dislocation of the ankle joint and subtalar joint is Type III of the Hawkin's classification. Type II, III and IV type injuries have increasing incidence of avascular necrosis.

This anomaly is seen where there is fusion or presence of fleshy webbing between the digits of the feet. A. Syndactyly B. Amelia C. Phocomelia D. Sympodia E. Polydactyly

Correct answer: (A) Syndactyly. Explanation: Syndactyly is the fusion or fleshy webbing between the digits of the hands or feet. Hemimelia is the absence of all or part of the distal limb, the proximal limb is normal. Amelia is a congenital absence of a limb or limbs. Phocomelia is a congenital absence of the proximal limbs such that the feet or hands are attached to the trunk by an anomalous segment. Sympodia is the fusion of the lower extremities. Polydactyly is supernumerary digits on the hands or feet.

This ligament primarily limits the eversion of the subtalar joint: A. The interosseous ligament. B. The cervical ligament. C. The anterior talofibular ligament. D. The posterior talofibular ligament. E. The spring ligament.

Correct answer: (A) The interosseous ligament. Explanation: Ligamentous attachments include the interosseous ligaments which primarily limit eversion. The ligaments of the subtalar joint are considered short and powerful structures well- adapted to the severe stresses applied to them during weightbearing activities. The main ligament is the interosseous talocalcaneal ligament which consists of two bands, occupying the sinus tarsi. The anterior band runs from the calcaneus superiorly, anteriorly, laterally to insert on the inferior surface of the talar neck, just behind the edge of the articular facet of the head. The posterior band originates just superior to the anterior band obliquely, superiorly, posteriorly and laterally to insert just anterior facet of the talus. The ligament has been said to limit inversion and eversion of the subtalar joint.

What is the open kinetic chain position of the midtarsal joint? A. The oblique axis is pronated and the longitudinal axis is supinated in contact period. B. The oblique axis is supinated and the longitudinal axis is pronated in contact period. C. The oblique axis is pronated and the longitudinal axis is pronated during contact period. D. The oblique axis is supinated and the longitudinal axis is supinated during contact period.

Correct answer: (A) The oblique axis is pronated and the longitudinal axis is supinated in contact period. Explanation: Throughout the contact period of gait, the oblique axis of the midtarsal joint is pronated and the longitudinal axis is supinated. This represents an open kinetic chain position created by the pull of the muscles of the anterior compartment, which are eccentrically contacting to decelerate plantarflexion of the ankle after heel-strike. Contraction of the anterior tibial, which inserts medial to the long axis of the foot, will invert or supinate the midtarsal joint about the longitudinal axis. Contraction of the extensor digitorum longus and peroneus tertius, which insert lateral to the long axis of the foot, abduct and dorsiflex, or pronate, the midtarsal joint about the oblique axis. Throughout the contact period, with the subtalar joint pronating to absorb shock, the longitudinal axis remains inverted to maintain the foot in a plantigrade position. Forefoot loading through contact, beginning on the lateral side of the foot, enables ground reaction forces to maintain the midtarsal joint's oblique axis in its maximally pronated position.

A "Stochastic Event" in radiation is one in which _______________________. A. The probability/risk of occurrence increases with the radiation dose. B. The severity of the occurrence increases with the radiation dose. C. A threshold dose must be exceeded before the radiation effects can occur. D. The progeny born to the exposed individual will develop the radiation effects.

Correct answer: (A) The probability/risk of occurrence increases with the radiation dose. Explanation: In order to answer this question, you need to know the difference between "stochastic" and "nonstochastic". If a particular condition occurs/develops after a specific amount of radiation exposure, then this is called a nonstochastic event. Because there is a pre-determined value that has to be met and exceeded in order to develop the condition, we can say that there is no chance involved. We can repeatedly predict if and when the effects will develop. Nonstochastic events have a threshold dose, and the severity of the condition will increase as the dose increases. Therefore, answers (B) and (C) are talking about a "Nonstochastic Event". A Stochastic event is one that occurs randomly and is not predictable. The event/condition may occur with or without radiation exposure. There is no specific dosage (threshold dose) that must be met. Effects occur by chance and are not reproducible from person to person. With stochastic events, the severity of the condition/event is not affected by the radiation dosage. However the risk/chance/probability of developing the condition will increase as the magnitude of the dosage increases. Therefore the correct answer is (A). Answer (D) is referring to the genetic effects of radiation exposure and have nothing to do with chance.

While in the clinic, you would like to obtain an medial oblique view radiograph. The radiology technician is not available, and you would like to take the x-ray yourself. How will you position the patient? The tubehead at 0 degrees, the knee flexed then turned inward so that the medial side of the foot is closer to the cassette and the sole forms a 45 degrees angulation with the cassette. The central ray direction should be the second metatarsal cuneiform. 15 degrees from the vertical, directed posteriorly. The tubehead at 0 degrees, the knee flexed then turned outward so that the lateral side of the foot is closer to the cassette and the sole forms a 45 degrees angulation with the cassette.

Correct answer: (A) The tubehead at 0 degrees, the knee flexed then turned inward so that the medial side of the foot is closer to the cassette and the sole forms a 45 degrees angulation with the cassette. Explanation: Position the patient with the tubehead at 0 degrees, the knee flexed then turned inward so that the medial side of the foot is closer to the cassette and the sole forms a 45 degrees angulation with the cassette, with the central ray toward the third metatarsal cuneiform joint.

What provides the necessary ability to the AO/ASIF screw to purchase the bone? A. Threads B. Core C. The rake D. The pitch

Correct answer: (A) Threads. Explanation: Screw thread provides the means by which the screw is able to purchase bone. Thread is a thin ribbon of material encircling the core of the screw at a specific angle. The angle determines the rate at which the screw will advance through bone. Thread configuration patterns may be either buttress shaped or V shaped. There is no significant difference in holding power between buttress and V-threaded screws; however, the buttress design affords greater resistance to bending. Thread pitch refers to the distance between adjacent threads. Cortical screws have a smaller pitch (20 to 32 threads per inch) that allows for a greater number of threads to engage compact or dense bone. Cancellous screws have a larger pitch because cancellous bone is less dense and more porous. Increasing the pitch should provide a larger area of contact between cancellous bone and the screw. Additionally, cancellous screws are designed with a greater thread-to-core ratio to enhance bone to screw contact further.

Which of the options below is a contraindication for the use of epinephrine in local anesthetic? A. Thyrotoxicosis B. Use in toes in diabetics C. Selective Serotonin Reuptake Inhibitors D. Use in toes in non diabetics

Correct answer: (A) Thyrotoxicoxis. Explanation: A long standing urban legend in podiatry is that epinephrine is not to be used in fingers, nose and toes. The thought behind this is as these are terminal structures, the vasoconstrictive properties would cause too great of an anoxic event and necrosis would set in. The fact is that epinephrine results in only temporary vasoconstriction that lasts for 20-60 minutes and even with that said, circulation to the toe is even then not completely occluded. With that said, there are some definite contraindications for the use of epinephrine in local anesthetic. They include thyrotoxicosis, coronary artery disease, severe peripheral vascular disease, vasospastic conditions and tricyclic antidepressants, such as monoamine oxidase inhibitors, which may augment the effect of epinephrine.

What are the two distinct axes of motion at the first metatarsophalangeal joint? A. Transverse and sagittal B. Sagittal and frontal C. Transverse and frontal D. Transverse and coronal plane

Correct answer: (A) Transverse and sagittal. Explanation: The first metatarsophalangeal joint has two distinct axes of motion or degrees of freedom. The transverse or horizontal axis provides pure sagittal plane motion, and the vertical axis provides pure transverse plane motion. The transverse axis of the first metatarsal phalangeal joint is an axis whose position moves with sagittal plane motion of the joint.

Cerclage osseous fixation is adequate to provide interfragmentary compression to which of the following fractures: A. Transverse fracture, situated in juxta-articular fashion B. Transverse fractures and spiral fractures C. Spiral fractures and oblique fractures, situated in juxtarticular fashion D. Oblique fractures only

Correct answer: (A) Transverse fracture, situated in juxtarticular fashion. Explanation: Cerclage may be the only option for stabilization of fractures or osteotomies in patients with osteoporotic bone, in which screw purchase is inadequate and intramedullary fixation may not be indicated. It is defined as the encircling of a part with a ring or loop. Cerclage involves the circular application of various diameters of malleable wire to stabilize fractures not amenable to other forms of internal fixation. Cerclage can be useful when exposure to a fracture is limited, yet interfragmentary compression is required for healing. Cerclage may be the only option for stabilization of fractures or osteotomies in patients with osteoporotic bone, in which screw purchase is inadequate and intramedullary fixation may not be indicated. A modification of the cerclage technique involves the transosseous or intraosseous passage of malleable wire. This technique allows for interfragmentary compression to be generated across transverse osteotomies or fractures that are situated in a juxtaarticular fashion.

Consider the following local anesthetics. Which local anesthetic below has the most rapid onset? A. A B. B C. C D. D

Correct answer: (B) Explanation: The speed of onset of a local anesthetic is determined by the pKa. The pKa is indicative of the pH at which the ionized and unionized forms of a local anesthetic are present in equal amount in a solution. The smaller the pKa, the higher the concentration of the active base molecule and subsequently the quicker the onset of the local anesthetic. Agents with a lower pKa, such as 7.7 to 7.8 such as lidocaine, prilocaine and etidocaine, have a faster onset action than bupivicaine, tertacaine and procaine which have a pKa ranging from 8.1 to 8.6.

Please place the following in order of first step to last step in the Ponseti Technique for clubfoot casting. 1. Achilles tendon lengthening/ tenotomy 2. Supinating the forefoot and dorsiflexing the 1st metatarsal 3. Foot is abducted with counter pressure over the talus 4. Use of braces A. 3,2,1,4 B. 2,3,1,4 C. 1,2,3,4 D. 4,2,1,3

Correct answer: (B) 2,3,1,4. Explanation: The Ponseti technique for casting out clubfoot is the mainstay of conservative treatment in this day and age. The first step is to supinate the forefoot and dorsiflex the 1st metatarsal. It is important to never pronate the foot. The second step is foot abduction with counter pressure over the talus. This is important for correcting the varus and adduction. It is important to not touch the heel while doing this. The next step is to do the TAL or tenotomy. This is always done after evaluation of equinus. Addressing the equinus before the above can lead to a rocker bottom foot. The last step is use of braces with foot in abduction- usually want to use all the time for 2-3 months and then at night for a few years after correction.

The patient pictured below complains of forefoot pain. MR images are provided below. What is the diagnosis? A. Capsulitis 2nd mtp joint B. 2nd metatarsal stress fracture C. Bone bruise second metatarsal D. Avascular necrosis second metatarsal

Correct answer: (B) 2nd metatarsal stress fracture. Explanation: The appearance of stress fractures, whether fatigue or insufficiency, on MRI is uniform regardless of location. Stress fractures, as well as the edema and inflammation they cause, have low signal intensity on T1-weighted and have high signal intensity on T2-weighted sequences. The most common location for stress fractures of the foot is in the forefoot, more specifically the shaft or neck of the second or third metatarsal.

You are preparing to perform a scarf osteotomy on a 45 year old female. You are sending her for preoperative testing prior to surgery. She states that she recently had laboratory testing recently, which was all normal. What is the maximum age of test results that should be used to prepare for surgery? A. 2-3 months B. 4 to 6 months C. 6 to 7 months D. 6 months to 1 year

Correct answer: (B) 4 to 6 months. Explanation: Preoperative laboratory testing should be selective and not routine. Current recommendations are for current laboratory tests based on specific signs, symptoms and diagnosis. Normal laboratory results obtained 4 to 6 months before surgery may be used as preoperative tests, provided there are no changes in the clinical status of the patient. Preoperative laboratory studies include a complete blood count, extensive blood chemistry, coagulation profile, urinalysis, electrocardiogram (ECG) chest radiographs.

In regards to Lisfranc dislocations, often times these dislocation require closed reduction prior to surgical intervention. What is the longest delay acceptable from time of dislocation until closed reduction is achieved? A. 2-4 hours B. 4-6 hours C. 6-8 hours D. 8-10 hours

Correct answer: (B) 4-6 hours. Explanation: Closed reduction of Lisfranc dislocations should occur 4-6 hours after the injury. (A) A 2-4 delay from time of injury to time of closed reduction for a Lisfranc dislocation would be ideal. The question, however, did ask for the longest acceptable delay which is 4-6 hours. (C) A 6-8 hour delay from time of injury until closed reduction of a Lisfranc dislocation is unacceptable. (D) A 8-10 hour delay from time of injury until closed reduction of a Lisfranc dislocation is unacceptable.

What is the average axis of deviation of subtalar joint? A. 4 degrees from the transverse plane and 6 degrees from the sagittal plane B. 42 degrees from the transverse plane and 16 degrees from the sagittal plane C. 42 degrees from the sagittal plane and 16 degrees from the transverse plane D. 20 degrees from the transverse plane and 10 degrees from the sagittal plane

Correct answer: (B) 42 degrees from the transverse plane and 16 degrees from the sagittal plane. Explanation: The average axis deviation is 42 degrees from the transverse plane and 16 degrees from the sagittal plane. There is three times as much transverse and frontal plane motion as sagittal plane motion with average axis deviations.

A 47 year old patient is having bunion and hammertoe surgery completed at the surgery center. The anesthesiologist will be giving the patient sedation anesthesia and asks you to give local anesthesia. You decide that you want to give Lidocaine 1% plain. You start giving the local anesthesia and you give a total of 25 cc. The patient still has some feeling of pain in her 5th toe. What is the maximum ADDITIONAL amount of Lidocaine 1% plain you can give to patient before you reach maximum dose- non weight based? A. 10cc B. 5cc C. 1cc D. No more- switch to ester local anesthetic

Correct answer: (B) 5cc. Explanation: The first thing that you need to pull out of this question is that the local anesthetic being used is Lidocaine 1%. Lidocaine 1% has 10mg/cc. Lidocaine 2% has 20mg/cc. Marcaine 0.5% has 5mg/cc and Marcaine 0.25% has 2.5mg/cc. You then need to know the maximum dose (non weight based). The maximum dose of Lidocaine 1% plain is 300mg. You then divide 300mg/10mg per CC and get your maximum dose of 30cc of Lidocaine 1% plain. Since the podiatrist already gave the patient 25cc, the max additional you could give is 5cc..

Regarding nail bed lacerations, primary repair of the nail bed can be attempted up to _____ days following laceration: A. 3 B. 7 C. 10 D. 14

Correct answer: (B) 7. Explanation: After seven days, the presence of contracture and granulation tissue preclude accurate reapproximation.

Which of the following statement regarding digital syndactlization is correct? It is not an effective procedure for correction of a heloma molle If deforming forces of a flail toe are not corrected then the syndactlization itself is inadequate to provide long term correction Alignment of the adjacent digits are not important in a syndactlization procedure Dissection is through subcutaneous tissue

Correct answer: (B) If deforming forces of a flail toe are not corrected then the syndactlization itself is inadequate to provide long term correction. Explanation: Syndactyly is the side-to-side anastomosis of two neighboring toes by resection dermoplasty. It is indicated for flail digits and heloma molle. This procedure is performed only at the depth of the dermis. It is imperative that for successful correction the deforming forces be eliminated from the flail and that the adjacent toe be a stable functioning rectus digit.

Prior to performing a distal metatarsal osteotomy for a bunion correction, a surgeon injects 16 cc of a 50:50 mixture of 1% lidocaine without epinephrine and 0.5% bupivacaine without epinephrine in a Mayo block. How many milligrams of each anesthetic did he inject? A. 160 mg lidocaine and 80 mg bupivacaine B. 80 mg lidocaine and 40 mg bupivacaine C. 160 mg lidocaine and 160 mg bupivacaine D. 80 mg lidocaine and 80 mg bupivacaine

Correct answer: (B) 80 mg lidocaine and 40 mg bupivacaine. Explanation: Local anesthetics are designated as a percentage based upon their concentration. For example, an anesthetic that is 1% is equal to 10 mg/mL Lidocaine, whether with or without epinephrine, often comes in either 1% or 2% concentrations, that is 10 mg/mL or 20 mg/mL. Bupivicaine frequently comes in 0.25% or 0.5% concentrations, or 2.5 mg/mL and 5 mg/mL, respectively. Assuming that the mixture that was injected was in a perfect 1:1 ratio, 8 mL of each anesthetic was injected. Since the lidocaine is 10 mg per mL, multiply 8 (the total number of mLs injected) by 10 (the number of mg per mL), there were 80 mg of lidocaine injected. Using the same principles, multiplying 8 (the total number of mLs of bupivacaine) by 5 (the number of mg per mL), there were 40 mg of bupivacaine injected.

Nuclear scintigraphy plays a role in the diagnosis of osteomyelitis. Unlike plain radiographs, changes can be seen on scintigraphic studies within hours of onset. Which nuclear imaging study is best for diagnosing osteomyelitis? A. Technetium 99 MDP B. 99m-Tc-hexamethylpropylene amine oxide scan C. Gallium D. Indium

Correct answer: (B) 99m-Tc-hexamethylpropylene amine oxide scan. Explanation: 99m-Tc-hexamethylpropylene amine oxide scan (Ceretec) is a white cell labeled scan that shows excellent sensitivity, specificity and accuracy for osteomyelitis. It is the most accepted white cell scan for this diagnosis. It is easier to run than Indium and superior from the standpoint of outcomes. It also may show promise in differentiating osteomyelitis from Charcot. Technetium 99 MDP is the "bone scan"and is very sensitive, but less specific which is not as helpful in diagnosis of osteomyelitis. The isotope is specific for hydroxyapatite matrix of bone. Positive scans can be seen in anything that show increase bone metabolic activity such as: infection, fractures, osseous surgery, tumors, Charcot, and arthritidies. It is broken into three phases: Flow (taken immediately); Blood pool (taken 5-10 minutes later); Bone (after about 4 hours). Gallium scans are more specific for soft tissue than bone. However it can be combined with a Technetium 99 MDP scan and provide some useful information. If uptake is greater in the Gallium study, but fades in the third phase of the technetium study, bone infection can be ruled out. If uptake in intensely is focal in both than osteomyelitis can be assumed. Indium scans are a white blood cell labeled scan that is useful in detecting osteomyelitis, but not commonly used due to price and more improved scans which have surpassed the indium scan.

A 55yo male patient (BMI 31) presents for outpatient surgery with a history of well controlled IDDM and hypertension, without functional systemic limitations. What ASA class would this patient be under? A. Class I B. Class II C. Class III D. Class IV E. Class V

Correct answer: (B) ASA Class II. Explanation: According to the American Society of Anesthesiologists Class II is a patient with mild systemic disease without substantial functional limitations i.e. controlled Diabetes, controlled hypertension without systemic effects, cigarette smoker without COPD, anemia, mild obesity (BMI 30-40), age over 70, age under 1, and/or pregnancy. Class I is for a healthy patient with no systemic diseases. Class III is for a patient with systemic disease with systemic manifestations ie uncontrolled diabetes/HTN, smoker with COPD or BMI >40. Class IV is patient with severe systemic disease that is a threat to life. Class V is a moribund patient who is not expected to survive without surgery.

What is the appropriate position for a Kirschner wire axis guide in the Austin bunion procedure? A. Aiming at the head of the third metatarsal; angled 20° plantarly B. Aiming at the head of the fourth metatarsal; angled 20° plantarly C. Aiming at the head of the third metatarsal; angled 30° plantarly D. Aiming at the head of the fourth metatarsal; angled 30° plantarly

Correct answer: (B) Aiming at the head of the fourth metatarsal; angled 20° plantarly. Explanation: The correct position for a Kirschner wire axis guide in the Austin bunion procedure is for the wire to be aiming at the fourth metatarsal head and angled 20° plantarly. This orientation causes slight plantarflexion with lateral translation of the head. The plantarflexion is helpful in compensating for the bone loss from the kerf of the saw blade. (A) The 20° plantar obliquity is correct, however the third metatarsal target is incorrect. This would cause elongation and plantarflexion. (C) An axis guide aiming toward the head of the third metatarsal and angling 30° plantarly is incorrect. This orientation would cause excessive plantarflexion and elongation. (D) The aiming target of the fourth metatarsal is correct, but the angulation of 30° planetary would cause excessive plantarflexion.

Peripheral nerves consists of three different nerve types depending on their respective fiber properties. The three peripheral nerve types are A, B, and C. Which nerve type is the easiest to block with local anesthetic infiltration? A. A fibers B. B fibers C. C fibers D. They are all equally easy and quick to block.

Correct answer: (B) B fibers. Explanation: The A fibers are myelinated somatic nerve fibers and are further subdivided into alpha, beta, gamma and delta fibers. The B fibers are also myelinated with preganglionic nerve fibers are smaller than any fiber type in the entire A group. The B fiber is the easiest and quickest fiber type to block in the body. The C fibers are unmyelinated, postganglionic nerve fibers that are the smallest in the entire body. They conduct pain, pressure and temperature sensation. Due to their unmyelinated nature, they are more difficult to block than many of the larger fiber types. The reason for this is that myelination decreases the length of the exposed nerve to only the nodes of Ranvier which is where the local anesthetic imparts its action. In contrast, in unmyelinated nerves the full circumference and length of the nerve must be bathed in local anesthetic for a block to be effective.

Which statement relating to the picture below is incorrect? A. The distraction nut is on the correct side of the clamp. B. It is advisable to place the pins in the incision line. C. The absence of a bolt on the distal clamp can potentially destabilize the fixator. D. An axial kirschner wire is advisable in the procedure above.

Correct answer: (B) It is advisable to place the pins in the incision line. Explanation: The picture above shows a very irritated foot undergoing a brachymetatarsia correction. A less than healthy looking incision is seen with serous drainage, inflammation and swelling. This in large part is due to the fixator being placed in the incision line and the skin healing around the pins. Therefore it is not advisable to place the pins in the incision line if it can be avoided. The absence of a bolt on the distal clamp can allow distal movement of the clamp along the bar which in turn potentially has a destabilizing effect. On the other hand, the distraction nut is correctly positioned relative to the clamp.

56 year old male who is postoperative day 6 from a Left BKA develops shortness of breath. Upon examination you determine there is high clinical suspicion for a Pulmonary Embolism. Heart rate is 103 bpm, BP 120/87, T-99.1F, RR-24. Which of the following is the gold standard test to diagnose this acute PE? A. Lower Extremity Venous Doppler Ultrasound B. CT Angiogram C. D-Dimer D. Chest X-Ray

Correct answer: (B) CT Angiogram. Explanation: The gold standard diagnosis for acute PE is a CT angiogram (B). Pulmonary embolism usually occurs in a post-operative patient from a DVT that forms in the legs and then embolizes to the pulmonary artery of the lung. Tachycardia and shortness of breath are common signs. Chest X-ray is not commonly used to diagnose a PE, but sometimes one can see the Hampton's Hump sign which has a correlation with PE. D-Dimer is commonly used to rule out a PE but with strong clinical suspicion a CT angiogram should not be delayed waiting on a D- Dimer result. With low clinical suspicion a D-Dimer is used to help rule out. Lower extremity venous Doppler ultrasound is used to diagnose a DVT.

A 53 year old female presents pre-operatively for elective ankle arthroscopy. She is diabetic and her most recent hemoglobin A1C is 6.2. She has no other significant past medical history. What class is she according to the ASA physical status classification system? A. Class I B. Class II C. Class III D. Class V

Correct answer: (B) Class II. Explanation: The ASA physical status classification system is a widely used system to stratify patients according to their pre-operative risk for potential complications. Class I patients have no systemic diseases and are considered healthy. They are at the lowest risk for post-operative complications. Class II patients are patients with mild systemic disease with only minimal functional limitations. This class would include patients with well-controlled diabetes or hypertension, obesity, current smokers, or social alcohol drinkers. Class III patients are those with severe systemic disease with substantive physical limitations. This would include poorly- controlled diabetes or hypertension, morbid obesity, dialysis dependent patients who are regularly receiving dialysis, and patients with a remote (> 3 months) history of MI, CVA, TIA, or CAD/stents. Class V patients are moribund patients who are not expected to survive more than 24 hours without the operation. The patient in this question is a well-controlled diabetic as evidenced by her hemoglobin A1C level of 6.2. She would be a class II patient.

A patient is undergoing first metatarsophalangeal joint arthroplasty with a silastic implant. The patient has previous history of penicillin allergy with an anaphylactic reaction. Which of the following is the most appropriate antibiotic for surgical prophylaxis? A. Cefazolin B. Clindamycin C. Vancomycin D. Cefepime

Correct answer: (B) Clindamycin. Explanation: Clindamycin has good penetration of the glycocalyx, which can surround a prosthetic implant. It is an effective alternative for patients with penicillin allergies. (A) Cefazolin is commonly used for procedure prophylaxis. If a patient has had a previous mild reaction to penicillin, cefazolin may be used with caution. However, if the patient has history of anaphylaxis or angioedema, potential cross-reactivity poses a significant risk and should not be used. (C) Vancomycin may be used for patients with history of severe reaction to penicillin. It is particularly beneficial in a hospital setting with increased risk of MRSA. Although it is an appropriate option, it is not the best choice for this patient undergoing a joint arthroplasty with implant. (D) Cefepime is a fourth general cephalosporin. This class of drugs does not have adequate staph coverage and is not routinely used for procedure prophylaxis.

Nearly all local anesthetics produce a vasodilatory effect. The sole exception to this is cocaine, which produces a vasoconstrictive action. What is the mechanism of action by which cocaine produces vasoconstriction? Cocaine causes constriction of vascular striated muscle which subsequently narrows the vessel's lumen. Cocaine constricts blood vessels by potentiating the action of norepinephrine, thereby preventing its own absorption through vasoconstriction. Cocaine constricts blood vessels by potentiating the action of epinephrine, thereby preventing its own absorption through vasoconstriction. The mechanism by which cocaine causes vasoconstriction is unknown.

Correct answer: (B) Cocaine constricts blood vessels by potentiating the action of norepinephrine, thereby preventing its own absorption through vasoconstriction. Explanation: The rate of local anesthetic absorption is largely influenced by the agent's degree of vasodilation, site of injection, total dose administered, and presence of a vasoconstrictive agent. All local anesthetics, with the exception of cocaine, cause vasodilation by relaxation of vascular smooth muscle. In contrast, cocaine causes vasoconstriction of blood vessels by potentiating the action of norepinephrine. Norepinephrine causes vasoconstriction by increasing the vascular tone of the vessel by alpha adrenergic receptor activation.

Iatrogenic interruption of the arterial supply and vasospasm are common causes of vascular embarrassment following neuroma resection. From the options below what is another common cause of vascular insult following neuroma resection? A. deep space hematoma B. congestion secondary to post operative edema C. medial calcific sclerosis D. Beurger's disease

Correct answer: (B) Congestion secondary to post operative edema. Explanation: Arterial embarrassment following an intermetatarsal neuroma resections an uncommon, but very serious complication. Iatrogenic injury to the interdigital artery can lead to an ischemic event of the toe(s). The interdigital artery is in relatively close proximity to the interdigital artery. Vasospasm is another cause of an ischemic situation for the digit. Vasospasm can occur from iatrogenic manipulation, over stretching of the vessel, or an underlying vasospastic disorder. Another cause for vascular embarrassment is from congestion secondary to post operative edema. In this scenario the edema combined with the surgical dressing create a environment where the foot is under significant compression which in turn compromises the arterial digital supply.

Which of the following is true of propofol? A. It provides pain relief. B. It is an amnesic. C. It is less commonly used than sodium thiopental. D. It acts through GABA and calcium channel blockade.

Correct answer: (B) It is an amnesic. Explanation: Propofol is a short-acting IV amnesic medication used in the induction and maintenance of general anesthesia. It has largely replaced sodium thiopental because it is shorter- acting and has faster recovery time. It does NOT provide any pain relief and so it is administered with a pain medication, often fentanyl. Propofol acts via potentiation of GABA and sodium channel blockade, among other proposed mechanisms. The half-life of elimination of propofol has been estimated to be between 2 and 24 hours. However, its duration of clinical effect is much shorter, because propofol is rapidly distributed into peripheral tissues. When used for IV sedation, a single dose of propofol typically wears off within minutes.

A 67 year old man with a history of atrial fibrillation, and hypertension is scheduled to have a right hallucal nail avulsion with phenol matrixectomy. He is taking warfarin with an international normalized ratio (INR) of 2.0 to 3.0. How should he be managed for his upcoming procedure? A. Stop Warfarin 3 days prior and start SCD's. B. Continue his warfarin perioperatively. C. Avoid warfarin the morning of the procedure. D. Stop warfarin 3 days prior and bridge with heparin 5,000 units subQ.

Correct answer: (B) Continue his warfarin perioperatively. Explanation: The question to be asked for any patient on warfarin scheduled for surgery is: is the warfarin cessation needed? While most surgical procedures require holding this medication. It is important to recognize procedures that can safely be performed while on warfarin. Certain procedures do not pose a significant increased risk of bleeding while on warfarin, some of those procedures are: minor podiatric procedures such as nail avulsions and phenol matrixecctomies, joint and soft tissue aspirations and injections, skin procedures such as Mohs surgery, simple excisions, cataract extractions and trabeculotomies, many gastrointestinal procedures such as upper endoscopy and colonoscopy with or without biopsy, endoscopic retrograde choloangiopancreatography (ERCP) without sphinterectomy, biliary stent insertion without sphinterectomy, endosonography without fine needle aspiration, and push entroscopy, dental procedures such as restorations, endotonics, micrographic surgery and simple excisions.

What is the term given when one party performs or promises to perform something that is of value to another party? A. Negligence B. Contract C. Res Ipsa Loquitor D. Libel

Correct answer: (B) Contract. Explanation: A contract is when one party performs or promises to perform something that is of value to another party. If one party fails to complete his promise, that party is in breach of their contractual responsibility and the aggrieved party may seek money damages in court as a remedy. A contract may be either implied or expressed. An implied contract is most similar to the doctor-patient relationship in which a contract is established without verbally expressed terms. It's implied by the fact the patient is presenting to the doctor and allowing the doctor to examine the patient, that the patient is going to be treated, and the doctor will be compensated for the treatment provided. Express contract is an actual agreement between two parties using explicit language, either orally or written, to openly discuss and agree upon the terms.

A patient comes into your office with a chief complaint of callus on the 5th toe at the level of the PIPJ. Patient states that she has tried pads and trimming the callus and she would like surgery to have the toe straightened. When you evaluate the patient, you notice an adductovarus deformity of the toe. In the operating room, which of the following is the correct direction for skin-plasty? A. Distal lateral to proximal medial B. Distal medial to proximal lateral C. Plantar distal to dorsal proximal D. Dorsal distal to plantar proximal

Correct answer: (B) Distal medial to proximal lateral. Explanation: The 5th toe adductovarus toe is a very common complaint in podiatry practice. The skin plasty wedge that is removed in a distal medial to proximal lateral direction in order to allow for derotation of the toe. In addition to this skin wedge excision, commonly an arthroplasty of the 5th toe proximal phalanx is often completed to prevent the callus from returning.

These intrinsic muscles are bipennate: A. Flexor digitorum brevis muscle B. Dorsal interosseous muscles C. Plantar interosseous muscles D. Lumbricale muscles

Correct answer: (B) Dorsal interosseous muscles. Explanation: Dorsal interossei. The four dorsal interossei muscles are bipenniform/bipennate that originate from the lateral surface of the metatatarsals delineating the corresponding intermetatarsal space. The first dorsal interosseous attaches to the entire tibial surface of the second metatarsal bone and the inferior surface of the fifth metatarsalphalangeal joint and the base of the proximal phalanx of the fifth toe.

The patient with the following muscular dystrophy is at risk of developing malignant hyperthermia. A. Becker's dystrophy B. Duchene's muscular dystrophy C. Facioscapulohumeral muscular dystrophy D. Myotonic dystrophy

Correct answer: (B) Duchene's muscular dystrophy. Explanation: Muscular dystrophies are susceptible to multiple perioperative complications, including respiratory, cardiac, and musculoskeletal systems. Acute rhabdomyolysis can result from the use of certain inhalational anesthetic agents, as Duchenne's muscular dystrophy. Muscular dystrophy 's patients are at risk of malignant hyperkalemia. (A) Becker's muscular dystrophy as Duchenne's patients suffer from a progressive loss of muscle mass. But Becker's is not associated with malignant hyperthermia when using inhalational anesthetic agents. (C) Facioscapulohumeral muscular dystrophy is the 3rd most common muscular dystrophy, after myotonic and Duchenne. It presents usually in the second decade of life, but may be later; shoulder weakness is the most common symptom, with scapular winging present in all patients. (D) Myotonic dystrophy is the most common muscular dystrophy in adults. It is autosomal dominant with weakness worse in the hands and legs, systemic involvements include cataracts, hypogonadism, diabetes mellitus, cardiac conduction defects, frontal balding.

The patient is on aspirin, scheduled for surgery, as assessment and preoperative preparation, you advise the patient to stop taking any aspirin or aspirin products 7 days prior to the scheduled surgical operation. You are explaining the reason to your patient. Please choose the most appropriate. A. Due to its irreversible antiplatelet effect B. Due to irreversible inhibitor of platelet cyclooxygenase C. Because of risk of rhabdomyolysis D. Because of risk of acidosis

Correct answer: (B) Due to irreversible inhibitor of platelet cyclooxygenase. Explanation: Aspirin should be held 7-10 days prior to surgery, due to irreversible inhibitor activity of platelet cyclooxygenase. Continuing antiplatelets agents' perioperatively carries the risk of bleeding. While discontinuation may increase cardiovascular events, irreversible agents like Aspirin must be withheld 7 to days before effects fully abate. (A) This is due to Clopidrogrel. (C) There is a risk of rhabdomyolosis with lipid lowering medications. (D) Metformin.

A complication not associated with bone grafting in the treatment of brachymetatarsia is: A. Limited amount of lengthening B. Neurovascular compromise C. Avascular necrosis and chodrolysis D. Metatarsalgia

Correct answer: (C) Avascular necrosis and chodrolysis. Explanation: AVN and chondrolysis have never been reported in the literature. (A) The amount of lengthening is limited using bone grafting, usually up to a maximum of 10mm. (B) Neurovascular compromise is a concern with acute lengthening. In JBJSAm 1978, Urano reported no problems with impaired circulation in 82 patients, following acute lengthening of a metatarsal. There was one case of temporary hypesthesia. (D) Overlengthening with acute grafting has been reported, despite the use of intraoperative x- ray. It is easier to "dial in" the appropriate metatarsal parabola with gradual lengthening.

What term is used to define the tensile strength of a suture when it is has been looped and knotted? A. Tensile Strength B. Effective Tensile Strength C. Stiffness D. Plasticity

Correct answer: (B) Effective Tensile Strength. Explanation: Effective tensile strength is the tensile strength of a suture when it is has been looped and knotted. This is important, as a knotted suture may only possess one-third of the tensile strength of the unknotted suture. Tensile strength is defined that the amount of force required to break a suture in half, or the amount of weight needed to break a suture divided by its cross-sectional area. Stainless steel has the highest tensile strength in terms of materials that are used for suturing where as natural materials like silk have the least tensile strength. (A) Tensile strength is defined that the amount of force required to break a suture in half, or the amount of weight needed to break a suture divided by its cross-sectional area. However, a suture may only have one-third of its normal tensile strength after it have been tied into a knot. (C) Stiffness is term used to describe the ability of a material to resist stretching. This is an important feature to suture material as it determines the ability of the material to be a good option for closure of wounds that may be in areas that will undergo expansion, such as the skin when post-operative swelling occurs. If the stiffness of the material is too low, the soft tissues being re-approximated by the material may gap when swelling occurs leading to a variety of possible complications. On the other hand, if the material has a high level of stiffness, the material may actually damage the soft tissue during its expansion and lead to the material "cutting out," or cut through the tissue as it expands. (D) Plasticity refers to the ability of a material to retain is stretched out and deformed shape after being stretched, without breaking. A material with a higher level of plasticity will retain more of its tensile strength when being stretched, as well as keeping its stretched-out length.

What term best describes the image below? A. Torsion wedge nonunion B. Elephant foot nonunion C. Oligotrophic nonunion D. Defect nonunion

Correct answer: (B) Elephant foot nonunion. Explanation: This question focuses on nonunions. There are two main categories of nonunions. Hypertrophic nonunions are those where union is possible in the right environment. Atrophic nonunions are those where union is not possible due to prone bone quality. These two main categories can be further broken down into subcategories. Hypertrophic nonunions are divided into three variants: elephant foot; horses hoof; oligotrophic. Atrophic nonunions are divided into 4 variants: torsion wedge; comminuted; defect; atrophic. Pictured above is a classic example of an elephant foot variant. This is a variant with significant callus formation and where there is very poor fixation or early weightbearing. Oligotrophic nonunion is another hypertrophic variant where there is absent callus. This variant commonly occurs with highly displaced fracture fragments. Defect and torsion wedge are atrophic variants. The torsion wedge variant has an intermediate fragment that heals to one main fragment, but not the other. The defect variant occurs when there is gapping at the fracture site due to a loss of a fragment.

Which of the following is a radiographic sign of bone graft failure? A. Gradual blurring of cortical margins B. Fixation movement C. Crossing of trabecular patterns across the graft-bone junction D. Increased radio density

Correct answer: (B) Fixation movement. Explanation: Radiographic evidence of bone graft to host integration failure include the following: sclerosis; dissolution of the graft; graft or fixation movement. In contrast, radiographic evidence of progressing graft to host integration include: gradual blurring of cortical margins and crossing of trabecular patterns across the graft-bone junction. In regards to radio density, cancellous bone grafts initially will appear more radio dense. Conversely, cortical bone grafts will initially appear more radiolucent. Clinical evidence of healing to determine good progression of graft incorporation include decreased edema, decreased pain and no evidence of instability.

Which of the following is the most common pathomechanical etiology for hammertoe deformity? A. Flexor Substitution B. Flexor Stabilization C. Extensor Substitution D. Extensor Stabilization

Correct answer: (B) Flexor Stabilization. Explanation: The most common pathomechanical etiology for hammertoes is flexor stabilization. This is most commonly seen in a pronated foot type. The long flexor tendons gain a mechanical advantage and overpower the interossei muscles leading to the formation of hammertoe. Flexor substitution is another cause which occurs primarily after injury to the superifical posterior compartment musculature. Extensor substitution is seen in cavus feet. With this cause, the long extensor tendons gain a mechanical advantage over the lumbrical muscles during the swing phase. The hallmark of this condition is hammertoes that primarily occur during the swing phase of gait. There is no such thing as extensor stabilization as an etiology of hammertoes.

Floating toe syndrome is a common complication after digital surgery. It results in a toe that does not purchase the ground when the foot is weightbearing. From the list below, what structures is at least in part for this complication? A. extensor digitorum longus tendon B. flexor digitorum longus tendon C. extensor digitorum brevis tendon D. plantar interossei muscle

Correct answer: (B) Flexor digitorum longus tendon. Explanation: The floating toe complication can indeed be a very frustrating problem. Even though seldom is pain involved, patients are very unsettled with the feeling that their toe toes not touch the ground like their other toes. The origin of the floating toe is one which the extensor digitorum longus, extensor digitorum brevis and slip of the plantar fascia to the respective digit fail to load the ray during weight bearing. Normally, during weight bearing, tension is placed on the flexors and the plantar fascia thereby stabilizing the proximal phalanx on the metatarsal head. Failure of the ray rot load results in failure to load the strand of plantar fascia and plantar plate. The ultimate result is an unstable and floating digit. The extensor tendons do not play a role in the formation of a floating toe and neither does the interossei muscles.

This is the least common cause of lesser digits deformity: A. Flexor stabilization B. Flexor substitution C. Extensor substitution D. Extensor stabilization

Correct answer: (B) Flexor substitution. Explanation: The least common cause of digital deformity, in this particular instance, the flexors are able to gain a mechanical advantage over the interossei when the deep posterior and lateral muscles attempt to substitute for a weak triceps surae. This may also be seen after excessive lengthening of the Achilles tendon.

The patient pictured below underwent a first metatarsophalangeal arthrodesis. During the procedure the surgeon noted the bone was more osteopenic than expected. Intraoperatively the decision was reached to use the plate and screw construct pictured below. What is the role of the plate in this fixation construct? A. Buttress plate B. Neutralization plate C. Compression plate D. None of the above

Correct answer: (B) Neutralization plate. Explanation: The oblique lag screw from distal medial to proximal lateral is the primary fixation and is providing interfragmentary compression. The dorsally placed plate is acting as a neutralization plate in the above example. Its purpose is to protect the primary fixation from the extrinsic forces that could jeopardize the fixation. The plate is orientated perpendicular or oblique, as in this example, to the axis of the lag screw. In contrast, a buttress plate is used to maintain length and alignment as in a severely comminuted fracture or the use of an interposition bone graft. Similarly, compression plates provide compression perpendicular to the plate. This is accomplished by eccentrically drilling the two closest screw holes to the desired fracture, fusion, or osteotomy site.

Pictured below is an 82 year old non-insulin diabetic female with the distal toe ulceration of the third toe. The deformity is semi-reducible. To date she has failed toe offloading and accommo- dative dressings. The patient has an apropulsive, shuffling gait with walker assistance. Which option below is most appropriate? A. Proximal interphalangeal joint arthrodesis B. Flexor tenotomy C. Proximal interphalangeal joint arthroplasty D. Distal interphalangeal arthroplasty

Correct answer: (B) Flexor tenotomy. Explanation: This is an ideal indication for a flexor tenotomy. This maneuver will allow manual reduction of the deformity, effectively offloading the tip of the toe. Additionally, it won't require significant healing like the boney procedures listed above. It will be acceptable from a functional standpoint as well as the patient ambulates minimally and in an propulsive manner. (A) The proximal interphalangeal joint arthrodesis would work, but given the specifics of the pa- tient and the deformity, the flexor tenotomy is ideally indicated. (C) The proximal interphalangeal joint arthroplasty would work, but given the specifics of the pa-tient and the deformity, the flexor tenotomy is ideally indicated. (D) The distal interphalangeal joint arthroplasty is simply a correction at the incorrect joint level.

When significant trauma occurs to digits immediate amputation is often indicated. Similar recommendations are warranted in several other scenarios. Which clinical scenario below warrants a 2 to 6 month delay before performing digital amputation? A. Electrical injury B. Frostbite C. Burns D. Malignancy

Correct answer: (B) Frostbite. Explanation: In the case of frostbite it is prudent to wait 2 to 6 months after the injury to allow devitalized tissue to demarcate before performing digital amputation. (A) Electrical injury creates extensive tissue damage that isn't always appreciated at first. Immediate amputation is indicated after tissue damage from electrical injury. (C) Burns can create extensive tissue damage which can necessitate immediate digital amputation. (D) Malignancy occurring in a toe, once confirmed, warrants immediate digital amputation or perhaps even more proximal amputation.

An 82 year old obese female is evaluated at an assisted living facility for the complaint of right heel pain. She states her heel has hurt for the last 3 months and denies any changes in shoes or activity level. She states she has tried topical analgesics as well as oral naproxen. Neither provided relief. Physical exam reveals tenderness on palpation to the plantar central portion of the heel. What would be an appropriate treatment for this patient? A. Endoscopic plantar fasciotomy B. Gel heel cup C. Functional orthotic D. Physical therapy referral

Correct answer: (B) Gel heel cup. Explanation: The key to this question is the location of the heel pain. Plantar central point tenderness (Calcodynia) is seen in the elderly and attributed to heel fat pad atrophy. This compounded by the patient's weight solidify the diagnosis of calcodynia. With this being the case the treatment of choice from the options above would be the incorporation of a gel heel cup to replace the shock absorption property lost with the fat pad atrophy. A functional orthotic would likely not be well tolerated. Similarly physical therapy wouldn't be of much value. An endoscopic plantar fasciotomy would not be indicate as this is not a case of plantar fasciitis.

An x-ray of a pediatric patient is obtained after an injury in which a bending force was applied to the diaphysis of a long bone. The convex side has a fracture, however there is no fracture on the concave side of the bone. This is referred to as: A. Plastic deformation B. Greenstick fracture C. Torus fracture D. Pathologic fracture

Correct answer: (B) Greenstick fracture. Explanation: A greenstick fracture occurs from a bending force applied to a bone. The concave side of the bone undergoes compression of the cortex, but it remains intact. The convex side undergoes a fracture due to the deforming force. (A) Plastic deformation occurs when a force is applied to a long bone, and the deformity exceeds the level of elastic deformation. Once the deforming force is removed, the cortex has been compressed to a level that it cannot recover to its original shape. (C) Torus fractures are compression injuries in which the cortex on one side of the bone buckles on itself and causes microfractures while the opposite cortex remains intact. (D) Pathologic fractures are those that occur through pre-existing areas of weakening from various conditions such as benign or malignant lesions.

What will you see during the physical examination of the congenital talipes equinovarus? A. Metatarsus adductus B. Heel inversion irreducible due to calcaneus size C. Tibial torsion D. Flexible pes planus

Correct answer: (B) Heel inversion irreducible due to calcaneus size. Explanation: During foot and ankle pediatric examination, the congenital talipes equinovarus will present with the following: hip ROM normal, knee ROM normal, ankle plantarflexion can be difficult to correct due to shortened Achilles tendon and deep posterior musculature, the heel inversion is often hard to correct due to the size of the calcaneus, inversion of the forefoot difficult to correct due to talocalcaneal joint subluxation.

Which of the following is defined as an increased number of phalanges in a given digit? A. Hemimelia B. Hyperphalangism C. Phocomelia D. Sympodia E. Polydactyly

Correct answer: (B) Hyperphalangism. Explanation: Hyperphalangism is an increased number of phalanges. Brachydactyly is an abnormal shortness of the toes. Syndactyly is the fusion or fleshy webbing between the digits of the hands or feet. Hemimelia is the absence of all or part of the distal limb, the proximal limb is normal. Amelia is a congenital absence of a limb or limbs. Phocomelia is a congenital absence of the proximal limbs such that the feet or hands are attached to the trunk by an anomalous segment. Sympodia is the fusion of the lower extremities. Polydactyly is the presence of supernumerary digits on the hands or feet.

An 82 year old female presents in the morning for an elective bunion surgery. Her overall health is well documented. She has no systemic illness or disease, is not on any medications, and has an allergy to penicillin. She takes an over the counter multivitamin daily and walks daily an average of two miles, however she is a smoker. She is to undergo the bunion operation under monitored anesthesia care. She is evaluated by an anesthesiologist. What physical status class is the above patient in per the American Society of Anesthesiologists? A. I Normal healthy individual B. II Mild systemic disease C. III Severe non-incapacitating systemic disease D. IV Severe incapacitating disease

Correct answer: (B) II Mild systemic disease. Explanation: The key to this question is the fact that the patient is a smoker. According to the American Society of Anesthesiologists, an individual who is a smoker is automatically at least an ASA class II - as are various mild diseases with mild functional limitations.

When discussing intravenous anesthetics, what agent of intravenous anesthesia is best described as a structural analogue of phencyclidine. This agent is known for its dissociative properties. It has rapid redistribution. It has multiple effects on the central nervous system as well as being a potent bronchodilator. It causes an increase in salivation. A very appealing aspect of this agent is that while being used as an inductions agent it preserves heart function, breathing and airway reflexes. This agent has also been known to cause nightmares and delirium. What intravenous anesthetic agent is being described? A. Benzodiazepines B. Ketamine C. Etomidate D. Propofol

Correct answer: (B) Ketamine. Explanation: Ketamine is an intravenous anesthetic used for inducing and maintaining anesthesia. It is a structural analog of phencyclidine, another dissociative agent. It is classified as a NMDA receptor antagonist, however it acts on several other fronts, including opioid receptors and monomania transporters. Ketamine has rapid distribution. It also is a potent bronchodilator useful in asthmatics. Indications for ketamine include anesthesia in children, in asthmatics, individuals for COPD, as a sedative of painful emergency procedures and to supplement spinal anesthesia. It causes an increase in salivation. A very appealing aspect of Ketamine is that while being used as an inductions agent it preserves heart function, breathing and airway reflexes. Ketamine has also been known to cause nightmares and delirium.

Which joint of the lower extremity is the main shock absorbing mechanism during the contact phase of gait? A. Hip Joint B. Knee Joint C. Ankle Joint D. Subtalar Joint

Correct answer: (B) Knee Joint. Explanation: During contact phase of gait, as the heel strikes, plantarflexion at the ankle joint rapidly moves the tibia forward while the femur lags behind, resulting in flexion of the knee. The quadriceps restrains rapid flexion as the hamstrings resist hyperextension. Together they dampen the impact load while maintaining stability of the joint. In addition, pronation at the subtalar joint permits faster internal rotation of the tibia to continue, resulting in medial rotation of the femur and further unlocking the knee. Motion at the hip is minimal to stabilize the trunk over the limb. The ankle joint rapidly plantarflexes until the pretibial muscles gain sufficient strength to effective resist this motion. The slow deceleration action of the muscles serves to absorb SOME of the ground reaction force. The ground reactive forces exerting force to the lateral tubercle of the calcaneus cause the subtalar joint to pronate. Furthermore, body weight has not yet fully moved over the new weightbearing limb so the center of gravity falls medial to the subtalar joint axis, further encouraging pronation at the joint. The pronation assists in the internal rotation of the limb, turning vertical forces into horizontal movement, absorbing some shock and energy.

In reference to the clinical photograph below what treatment option listed would be contraindicated? A. nail avulsion B. matrixectomy C. incision and drainage D. systemic antibiotics

Correct answer: (B) Matrixectomy. Explanation: The key to this question is the acknowledgement of the infectious process in play in this clinical scenario. A consequence of this infectious process is that any type of matrixectomy is contraindicated. Not only will the infectious process drastically decrease the effectiveness of the matrixectomy, but the insult to the tissue in the presence of an infectious process may facilitate a deeper infection, even an osteomyelitic one. A nail avulsion would be the most commonly accepted treatment of choice and then once the infection has cleared then a matrixectomy could be performed 4-5 weeks later. Incision and drainage would be indicated here as would systemic antibiotics.

Which facet of the subtalar joint is most commonly involved with talocalcaneal coalitions? A. Anterior B. Middle C. Lateral D. Posterior

Correct answer: (B) Middle. Explanation: The most common bridge between the talus and calcaneus involves the middle facet of the subtalar joint. The middle facet involves the sustentaculum tali and the medial aspect of calcaneus. This type of coalition less commonly involves the anterior facet and rarely involves the posterior facet. A talocalcaneal coalition cannot be readily viewed on your typical three view radiographs of the foot. However, talocalcaneal coalitions can be visualized through a Harris view, a variation of an axial calcaneal view taken at 45 degrees to the rearfoot. Evidence of talocalcaneal coalitions can be suggested on the typical 3 views. These secondary findings include talar beaking, posterior facet narrowing, and flattening or broadening of the lateral process of the talus.

At what stage of gait is maximum passive dorsiflexion of the ankle required? A. Heel Strike B. Midstance C. Propulsion D. Swing

Correct answer: (B) Midstance. Explanation: The momentum of the trunk brings the tibia over the ankle causing the ankle to passively dorsiflex as the knee extends and the forefoot continues to load. The dorsiflexion of the ankle is eventually stopped by the tension from the gastrosoleal complex, in come cases not allowing enough ankle joint passive dorsiflexion, such as in equinus.

Which of the following statements is true regarding general anesthesia? Inhaled anesthetics are all analgesics. Minimal alveolar concentration is the concentration in the lungs that is required to prevent movement in response to surgical stimuli in 50% of patients. The anesthetic with the lowest minimal alveolar concentration is Nitrous oxide. Nitrous oxide is commonly used as a sole agent in general anesthesia.

Correct answer: (B) Minimal alveolar concentration is the concentration in the lungs that is required to prevent movement in response to surgical stimuli in 50% of patients. Explanation: It is important to recognize that inhaled anesthetics are not analgesics. The anesthesiologist gives the patient an opioid such as fentanyl commonly along with the anesthetic agent. Answer choice B is correct as this is the correct definition of minimal alveolar concentration (MAC). The anesthetic agent with the lowest MAC is Isoflurane and is thus the most potent of the general anesthetic inhalational agents. Nitrous oxide has a very high minimal alveolar concentration and is not usually used alone for this reason. It is commonly used in combination with other agents.

A tibial sesamoid position of _____ or greater indicates crista erosion of the first metatarsal. A. 2 B. 3 C. 4 D. 5

Correct answer: (C) 4. Explanation: Tibial sesamoid demonstrates the medial movement of the head of the metatarsal. As the hallux valgus defomity increases the head will move more medially over the stationary sesamoids. The sesamoids are enveloped by the flexor hallucis longus tendon and does not move. The tibial sesamoid position ranges from 1 to 7. With one being in the correct alignment and 7 having falling completely into the innerspace. Tibial sesamoid of 3 occurs when the lateral border of the tibial sesamoid is just barely underlying the crista of the met head. Tibial position of 4 is immediately adjacent to the crista and will degenerate the crista with movement and weightbearing.

All of the following statements regarding surgical drains are true except? A. They provide a conduit by which external bacteria can enter the wound. B. Penrose-type drains are preferable in most situations over closed section drains. C. Drains cause a localized inflammatory response. D. Drains should be removed as soon as possible.

Correct answer: (B) Penrose-type drains are preferable in most situations over closed section drains. Explanation: Surgical drains are commonly used as part of dead space management. The have some considerable drawbacks that must be taken into account. They indeed do act as a conduit for external bacteria to enter the surgical site. Penrose-type drains are particularly bad on this aspect thereby making them the least preferable choice of drains. Closed suction drains are always preferable. Likewise, as they are a foreign body, they do cause a localized inflammatory response which may promote localized tissue damage and prevent neutrophils from being available to attack any bacteria. For all the reasons mentioned drains should be removed as soon as possible.

A 45 year old male presents complaining of a stiff, swollen painful left ankle. It has become progressively worse over the last four month. The aptient states it hurts all the time, but becomes significantly worse when he is on it and active. On physical exam the patient's neuromuscular status is intact. The ankle is mildly swollen and has a "boggy" feel to it. Range of motion of the left ankle is 5 degrees of dorsiflexion and 35 degrees of plantarflexion. Left ankle joint aspiration yields a blood tinged fluid. Below are radiographic and MRI images. What is the diagnosis? A. Synovial osteochondromatosis B. Pigmented villonodular synovitis C. Nodular synovitis D. Giant cell tumor

Correct answer: (B) Pigmented villonodular synovitis. Explanation: This is a classic example of a PVNS lesion. PVNS is a benign tumor of the linings of joints, tendon sheaths and bursa. The patient usually presents with pain and swelling in the ankle. The ankle, on exam, is swollen and feels "boggy." There is a loss of range of motion. Aspiration of the joint typically yields a bloody fluid. The MRI appearance of PVNS is pathognomonic because of the iron deposition in the tumor, which has a magnetic moment. So the key to the entire question is the MRI image. Synovial osteochondromatosis presents with intra-articular mineralized nodular masses in radiograph which are not present above. Nodular synovitis, although a form of PVNS, presents is a single, multilobulated mass. The MRI above demonstrates this is clearly not the case.

In AO/ASIF screw, this refers to the distance between adjacent threads: A. Threads B. Pitch C. Thread-to-core ratio D. The rake

Correct answer: (B) Pitch. Explanation: Pitch refers to the distance between adjacent threads. Cortical screws have a smaller pitch (20 to 32 threads per inch) that allows for a greater number of threads to engage compact or dense bone. Cancellous screws have a larger pitch because cancellous bone is less dense and more porous. Increasing the pitch should provide a larger area of contact between cancellous bone and the screw. Additionally, cancellous screws are designed with a greater thread-to-core ratio to enhance bone to screw contact further.

Which of the following best describes the action of the flexor digitorum brevis? A. Plantarflexion of the MTPJ B. Plantarflexion of the MTPJ and PIPJ C. Plantarflexion at the PIPJ D. Plantarflexion at the MTPJ, PIPJ, DIPJ

Correct answer: (B) Plantarflexion of the MTPJ and PIPJ. Explanation: Plantarflex the lesser digits, begins at the proximal interphalangeal joints, then the metatarsophalangeal joints. The extensor digitorum brevis has only 4 tendon slips, that course to each of the corresponding lesser digits. The insertion of the FDB tendons are at the middle phalanx of the lesser digits, which cause plantarflexion at the MTPJ and the PIPJ while non weightbearing.

A 57 year old male patient with a history of Atrial fibrillation presents to the surgery center for a bunionectomy. Patient has been taking Coumadin chronically since he was diagnosed with A-Fib 6 years ago. Upon questioning by the nurse, patient states that he took his last dose of Coumadin yesterday AM and his INR is 3.1. What is the best course of action? A. Complete Surgery as you normally would. B. Postpone surgery until the INR is lower. C. Complete surgery but keep tourniquet up for 30 minutes after case complete. D. INR should have no effect on surgery.

Correct answer: (B) Postpone surgery until the INR is lower. Explanation: Coumadin is a blood thinner that people take for many different conditions. Coumadin works on the extrinsic clotting pathway. In normal patients who are not on Coumadin, INR values should be around 1. In patients on Coumadin, the therapeutic range is from 2-3.5 depending on source. Before surgical procedure it is important to stop the Coumadin 3-5 days prior to surgery to be sure INR is within normal prior to surgery. Patients who are too high risk to stop Coumadin can be bridged to heparin which can be stopped a few hours before surgery.

When discussing circular external ring fixation one of the most basic techniques to provide compression is to fixate a transfixion wire so that it bows and then when tensioning that wire it will start to straighten and compress the bone segment into an adjacent bone segment. What is the name for this technique? A. Ilizarov compression B. russianing C. axial loading D. translational compression

Correct answer: (B) Russianing. Explanation: The simplest method of generating compression is by applying a thin wire through an osseous segment, but instead of attaching the ends of the wire to a ring so that the wire is straight, it is applied distal to the ring so that when the wire is attached at the other end, it bows. When the wire is tensioned, the wire straightens and in doing so, compresses the attached segment to the proximal part. This technique is referred to as "Russianing." An alternative to "Russianing" is the application of compression by translating a distal ring toward a more proximal frame, along parallel connecting rods.

A patient underwent neuroma resection of the second interspace. Post-operatively, the patient loses abduction strength of the 2nd digit. Which structure was most likely damaged? A. First dorsal interossei B. Second dorsal interossei C. First plantar interossei D. Second plantar interossei

Correct answer: (B) Second dorsal interossei. Explanation: There are four dorsal interossei muscles which provide abduction, and three plantar interossei which provide adduction. The second interspace contains the second dorsal interosseus muscle which abducts the ray, and the first plantar interosseus muscle which adducts the ray toward the midline (middle of the second ray). (A) The first dorsal interosseus muscle is in the first webspace, providing abduction of the 2nd ray. (C) The first plantar interosseus muscle is in the third webspace, not the second webspace. (D) The second plantar interosseus muscle is within the fourth webspace, not the second webspace.

The patient pictured below is four months post transmetatarsal amputation. What is the cause of the deformity seen below? A. Spasm of the anterior tibial tendon B. Severe imbalance frontal plane pull of the invertors as opposed to the evertors C. Weakening of the peroneus brevis tendon D. Contraction of the posterior tibial tendon

Correct answer: (B) Severe imbalance frontal plane pull of the invertors as opposed to the evertors. Explanation: The deformity pictured above is quite common after midfoot amputations. It is an equinovarus deformity that results from the majority of the dorsiflexors and evertors having been severed in the amputation. The only remaining dorsiflexor is the anterior tibial tendon which, as its pull is medial to the subtalar joint, it supinates as it dorsiflexes. The long extensors of the toes have been severed which would counter act the supination caused by the anterior tibial tendon during dorsiflexion. The only remaining evertor of the foot is the peroneus brevis. It is a rather small muscle and cannot counterbalance against the much more power anterior tibial tendon, posterior tibial tendon, and achilles tendon.

A 56-year-old NIDDM male presents to your office complaining of discoloration and swelling of the right foot. Patient relates history of stepping in a hole last week. Denies any pain at that time or currently. His medical history consists of NIDDM, peripheral neuropathy, obesity, hypertension, hypercholesterolemia, gout and psoriasis. Patient denies fever, nausea and chills. He is able to ambulate on the foot. Physical exam reveals a dependent rubor, edema, warmth to the midfoot, strong palpable pulses. What differential diagnosis should you be most concerned about? A. Acute gout attack B. Stage 0 Charcot neuroarthropathy C. Cellulitis D. Midtarsal sprain

Correct answer: (B) Stage 0 Charcot neuroarthropathy. Explanation: All four differential listed above are plausible. However, the differential diagnosis that one must be most concerned about is the Stage 0 Charcot neuroarthropathy. The sequela of a missed or delayed Stage 0 Charcot neuroarthropathy is disastrous, debilitating and can lead to amputation. Every diabetic who experiences trauma, whether minor or major, must be evaluated for peripheral neuropathy. Dependent rubor, warmth, bounding pulses and swelling in the presence of neuropathy and trauma are clinical findings consistent with Stage 0 Charcot. The radiograph is unremarkable in the midst which is typical and the MRI shows bone marrow edema. When all this data is taken into account the most logical and most critical differential diagnosis is Stage 0 Charcot neuroarthropathy.

Berndt and Harty recommended nonsurgical treatment for which type of talar OCD? A. Stage III lateral talar dome fracture B. Stage III medial talar dome fracture C. Stage IV medial talar dome fracture D. Stage IV lateral talar dome fracture

Correct answer: (B) Stage III medial talar dome fracture. Explanation: Berndt and Harty recommended nonsurgical treatment for Stage I and II OCD lesions which generally consists of immobilization for 6 weeks followed by physical therapy. Stage III medial talar dome fractures are generally posterior within a cup-shaped portion of the talar dome and are nonsurgical due to the difficulty in accessing the posterior medial lesions. (A) Stage III lateral talar dome fractures are considered surgical due to the instability of the fragment and difficulty consolidating. The lateral talar dome is described as wafer-shaped and the lesions are generally on the anterior portion, making them accessible for surgical intervention. (C) (D) Berndt and Harty Stage IV is a completely detached and displaced talar fragment. Due to the instability of these lesions, they are surgically treated.

In regards to skin flap physiology which term below is best defined as the change in length divided by the original length of the given tissue to which a force is applied? A. Stress B. Strain C. Creep D. Stress relaxation

Correct answer: (B) Strain. Explanation: There are four important concepts pertaining to flap physiology: stress, strain, creep and stress relaxation. Strain refers to the change in length divided by the original length of the given tissue to which a force is applied. Stress refers to the force plaid per cross-sectional area. Creep refers to the increase in strain seen when skin is under constant stress. This occurs in a matter of minutes and is due to an extrusion of fluid from the dermis and a breakdown of the dermal framework. Stress relaxation is the decrease in stress when skin is held in tension at a constant strain for a given time. This occurs over a matter of days to weeks and is due to an increase in skin cellularity and the permanent stretching of skin components.

Spinal anesthesia is achieved by injecting local anesthetic into: A. Epidural space B. Subarachnoid space C. Dura mater D. Subdural space

Correct answer: (B) Subarachnoid space. Explanation: Spinal anesthesia is achieved by injecting local anesthetic into the subarachnoid space. The block is typically done around L3-L4 or L4-L5 below the distal aspect of the spinal cord. The local anesthetic mixes with the CSF to act directly on the spinal cord. Epidural anesthesia is achieved by injecting local anesthetic into the epidural space, which is a potential space deep to the ligamentum flavum but short of the dura mater. Dura mater is the outermost covering of the brain and spinal cord. Local anesthetic is not injected directly into the covering of the spinal cord. Subdural space is a potential space between the layers of dura mater and arachnoid mater where local anesthetic is not injected for a spinal block.

A patient underwent ORIF of a left Weber B ankle fracture using a slightly anterior- lateral approach to the fibula. The patient complains of post-op burning, tingling and numbness. Which structure was most likely injured? A. Sural nerve B. Superficial peroneal nerve C. Deep peroneal nerve D. Saphenous nerve

Correct answer: (B) Superficial peroneal nerve. Explanation: The superficial peroneal nerve branches from the common peroneal nerve just distal to the fibular head. It courses adjacent and anterior to the fibula before it branches into the medial and intermediate dorsal cutaneous nerves. (A) The sural nerve courses along the posterior lateral aspect of the ankle adjacent to the Achilles tendon. The sural nerve is at higher risk when a posterolateral approach is used to gain access to the fibula, rather than a straight lateral approach. (C) The deep peroneal nerve course further anterior and deep than the superficial peroneal nerve, thus is at less risk of injury during ankle ORIF. (D) The saphenous nerve courses on the medial aspect of the leg and is not at risk of direct injury during ankle ORIF.

A patient presents with pain to the lesser digits, involving the dorsiflexion of the MTPJ, and plantarflexion of both PIPJ and DIPJ. What is your diagnosis? A. Hammertoe B. Mallet toe C. Claw toe D. Flail toe

Correct answer: (C) Claw toe. Explanation: Clawing of the toes is produced by the simultaneous pull of the extensor digitorum longus, the flexor digitorum longus, and the flexor digitorum brevis, leading to hyperextension at the metatarsophalangeal joint and flexion at the interphalangeal joint. As the flexors begin to contract, the toe is unable to buckle proximally. Instead, the flexors are able to provide a stabilizing force on the metatarsals and a stable insertion for the flexor muscles.

Digital ischemia is not a common complication following a neurectomy, but it can have disastrous consequences. Underlying vascular disease, vasospastic conditions, and previous neurectomy in the same webspace are all risk factors for this complications. Out of the options below, which one is also a risk factor for a postoperative ischemic toe? A. epinephrine in the local anesthetic B. surgery performed on adjacent webspace from a dorsal approach C. surgery performed on adjacent webspace from a plantar approach D. use of oral contraceptives

Correct answer: (B) Surgery performed on adjacent webspace from a dorsal approach. Explanation: Arterial compromise to a toe is a rather uncommon, but seriously complication after a neurectomy. Direct arterial injury, underlying vascular disease, vasospastic conditions, congestion secondary to postoperative edema can predispose vascular embarrassment to the digits. Another risk factor for arterial compromise following a neurectomy is simultaneous surgery performed on adjacent web spaces through a dorsal incisional approach. Simultaneous dorsal exposure and dissection to adjacent webspaces such as the second and third can ultimately lead to injury and/or spasm of multiple arteries and lead to adverse effects of the vascular supply. On the contrast performing simultaneous adjacent neurectomies form a plantar approach has much less dissection and potential vascular trauma which is why vascular embarrassment is almost nonexistent when a plantar approach is employed.

Flexor Digitorum Longus tendon transfer for correction of a hammertoe deformity is contraindicated for which type of patient? A. Rheumatoid arthritis B. T10 Paraplegic C. Cerebral Palsy D. Charcot Marie Tooth Disease (Type II)

Correct answer: (B) T10 Paraplegic. Explanation: The FDL transfer for a hammertoe repair hinges on functional, active muscle/tendon complexes. In the procedure the FDL is transferred to the dorsal aspect of the proximal phalanx. This turns a once deforming force into a corrective force with each muscular contraction. They key for this question is the need for the muscular contraction. The patient with T10 paralysis would fail to have muscle contraction at this level and would have a flaccid toe deformity. The other three conditions would not cause a problem with the technical effect of this correction.

The rerouting of the course of a normal muscle tendon without detachment to assist other functions is the description of which term listed below? A. Tendon Transfer B. Tendon Transposition C. Muscle-Tendon Transplant D. Tendon Suspension

Correct answer: (B) Tendon Transposition. Explanation: This question reviews the fundamental definitions of tendon surgery. The rerouting of the course of a normal muscle tendon without detachment to assist other functions is known as tendon transposition. Tendon transfer is the detachment of a tendon of a functioning muscle at its insertion and then its relocation to a new insertion or attachment. The detachment of a muscle tendon at both its origin and its insertion and moving it to a new location along with its neuromuscular support structures is known as Muscle-Tendon Transplantation. Lastly, tendon suspension refers to tendon procedures designed to support a structure.

Which of the following is not a source of secondary radiation? A. Leakage B. The X-ray beam as it leaves the tube C. Scattered radiation D. All of the above are sources of secondary radiation

Correct answer: (B) The X-ray beam as it leaves the tube. Explanation: The x-ray beam is also known as the primary beam and it is a source of primary radiation. It should never be directed at the radiographer. Secondary sources include leakage and scatter.

Which of the following statements regarding the radiographic image below is correct? Latency period for this clinical scenario is shorter than that of callus distraction in an evans osteotomy. The bone in this area is relatively less active than the metaphyseal bone. It is more difficult to form a viable periosteal sleeve in the location around this osteotomy. An osteotomy in the proximal metaphysis would be less stable.

Correct answer: (B) The bone in this area is relatively less active than the metaphyseal bone. Explanation: The osteotomy pictured above is in the metatarsal diaphysis which is biological less active than the metaphyseal bone. Due to the high proportion of cancellous bone in the calcaneus relatively to that of the metatarsal the latency period in the evans osteotomy is shorter than that of a metatarsal osteotomy. Dissection in the diaphyseal region as in the above example is easier and the periosteum is thicker. This allows for easy dissection and creation of a consistent periosteal sleeve. Additionally the mechanical strength of a structure varies with the square of the diameter when subjected to bending loads. Therefore a wider segment of bone (metaphysis) will tend to be more stable throughout lengthening and would lead to a more stable end result.

This articulation provides the midtarsal joint with an osseous locking mechanism in gait: A. The calcaneonacular joint B. The calcaneocuboid joint C. The talonavicular joint D. The subtalar joint

Correct answer: (B) The calcaneocuboid joint. Explanation: The calcaneocuboid joint provides the midtarsal joint with an osseous locking mechanism in gait. Excessive pronation is blocked when the rotating cuboid abuts the dorsolateral calcaneus. This locking mechanism is assisted by compression of the lesser tarsus by the synergistic action of the posterior tibialis, peroneus brevis, and peroneus longus muscles during the late midstance and propulsive phases of gait.

In the Ponseti technique for clubfoot correction what critical step must occur before the calcaneus can be dorsiflexed via the Achilles tenotomy? A. The forefoot must be in neutral. B. The calcaneus must be abducted out from under the talus. C. The fibula must be externally rotated. D. The calcaneus must be externally rotated out from under the talus.

Correct answer: (B) The calcaneus must be abducted out from under the talus. Explanation: It is only after the calcaneus abducts from under the talus that it can be dorsiflexed by the Achilles tenotomy. The calcaneus is ready to be dorsiflexed when the foot is abducted approximately 70o externally. The reduction is gradual, with stretching of the tarsal ligaments, and it is maintained by casting. The foot is "massaged" into the corrected position without undue force or bone and cartilage damage.

The most compelling reason for not repairing most pedal syndactyly is: A. Skin grafting is required. B. The child is not symptomatic. C. Vascular accidents are common. D. Growth arrest results in a short toe in adulthood.

Correct answer: (B) The child is not symptomatic. Explanation: Most children have no symptoms, and the risk of long-term problem of syndactyly if left alone is extremely small. Of course, there are clearly justifiable symptoms that warrant separation. These include asymmetrical growth, pain, psychological issues, etc. Although skin grafting is required it is an easy graft to obtain and heal both the recipient and the donor site. Vascular accidents with this procedure are also exceedingly rare as little to no dissection is performed. The procedure is purely at the skin and subcutaneous level. As there is no involvement of bone there is no concern of bone growth arrest.

A surgical consultation is being performed on the patient pictured below. The following test was performed. Given the result from this test pictured below which of the following statements is true? The clinical photograph above represents the silfversiold test and shows the need to incorporate a lateralizing calcaneal osteotomy such as a Dwyer osteotomy or lateral displacement osteotomy in the surgical planning. The clinical photograph above represents the Coleman-Block test and shows the need to incorporate a dorsiflexory wedge osteotomy of the first metatarsal in the surgical planning. The clinical photograph above represents the silfversiold test and shows the need to incorporate a dorsiflexory wedge osteotomy of the first metatarsal in the surgical planning. The clinical photograph above represents the Coleman-Block test and shows the need to incorporate a lateralizing calcaneal osteotomy such as a Dwyer osteotomy or lateral displacement osteotomy in the surgical planning.

Correct answer: (B) The clinical photograph above represents the Coleman-Block test and shows the need to incorporate a dorsiflexory wedge osteotomy of the first metatarsal in the surgical planning. Explanation: The test pictured above is the Coleman-Block exam which demonstrates the forefoot contribution in the varus hindfoot deformity. The Silfversiold exam is an examination to determine the contribution of the soleus in the equines deformity. Additionally the fact that when the first ray is offloaded in the test above the refract assumes a rectus orientation. This translates to the need to incorporate a dorsiflexory wedge osteotomy of the first metatarsal in the surgical planning. If when the first ray was offloaded and the heel remained in a varus position this would indicate that the varus position is fixed in the rearfoot itself. In this case, a lateralizing calcaneal osteotomy such as a Dwyer osteotomy or lateral displacement osteotomy would be indicated.

There are two main types of implants when discussing osseous fixation methods: load-sharing implants and load-sparing implants. What is the main goal of a load sparing implant in fracture fixation? A. To promote callous fixation B. To promote less callous formation C. To share stress with bone, allowing bone to work while the fracture is repaired D. To accelerate bone healing

Correct answer: (B) To promote less callous formation. Explanation: Loads sparing implants totally remove stress from the bone. Because there is less biologic activity, there is less callous formation. An example of load sparing implants are compression plates. Examples of load sparing situations are bed rest and nonweightbearing.

Which of the following below is an extrinsic failure of a silicone first metatarsophalangeal joint implant? A. deformation B. trimming the ends of a silicone implant C. fatigue fracture D. microfragmentation

Correct answer: (B) Trimming the ends of a silicone implant. Explanation: Extrinsic implant failure is the result of tampering with the implant or artificially creating defects in its structure which, in turn, negatively influence the physical properties of the implant. Trimming the ends of a silicone implant would be an example of this. The other three options are examples of intrinsic failure. Intrinsic failures pahthogical forces imparted on the implant while it is in vivo exceed the physical limits of the implant. This leads to implant failure through deformation, fatigue fracture and/or microfracture.

A medial-oblique projection of the foot reveals a large, "D-shaped" osseous structure located immediately adjacent to the navicular tuberosity. There is a cartilaginous attachment between this accessory navicular and the tuberosity. Based on this radiographic description, how would Lawson classify this ossicle? A. Type I B. Type II C. Type III D. Os tibiale externum

Correct answer: (B) Type II. Explanation: Lawson's classification helps to differentiate the various forms of the accessory navicular. Three types exist and as you work your way towards the navicular tuberosity from proximal to distal, the ossicle types are named accordingly (Type I -> Type II -> Type III). The Type I accessory navicular is typically the smallest in size, round in shape, and located within the posterior tibial tendon. This variation is positioned most proximal of the three types and is often referred to as an Os tibiale externum. It is considered a sesamoid rather than an accessory ossification center. Based on the description above, the bone in question cannot be a Type 1 accessory navicular which makes answers (A) and (D) incorrect. A Type II accessory navicular is larger, semi-circular in shape, and lies immediately next to the tuberosity. It is considered an accessory ossification center that remains attached to the tuberosity by cartilage or fibrous tissue. (B) is therefore the correct answer. A Type III accessory navicular is the final form and radiographically it will appear like a very large navicular that wraps around the talar head. It may also be called a cornuate navicular. Like the Type II accessory navicular, this type is also an accessory ossification center. The difference being that in a Type III there is an osseous union between the tuberosity and this bone.

The rate of callus distraction in brachymetatarsia is: A. 1mm every 24 hours B. 0.5mm every 24 hours C. 0.25mm every 6 hours D. 0.25mm every 3 hours

Correct answer: (C) 0.25mm every 6 hours. Explanation: External fixation is a widely used modality for callus distraction in cases of brachymetatarsia. It is important to know that there is a latency period from time of surgery to initiation of distraction, most commonly seven days. Studies have shown that a distraction rate of 0.25mm every 6 hours (total 1mm per day) have resulted in predictable rates of bone remodeling. (A) Although the goal of distraction is to increase 1mm per day, it is more stable to distract in small increments several times a day rather than all at once. (B) 0.5mm per 24 hours would result in half the ideal distraction per day. This would double the time of treatment. (C) 0.25mm every 3 hours would lead to 2mm of distraction per day. This may be damaging to surrounding soft tissue and not allow adequate time for bone remodeling.

An 88 year old female complains of a painful crossover second toe deformity on her left foot. To date she has tried and has failed shoegear modifications, padding, and taping. Radiographs shoe complete medial dislocation of the second digit with a long 2nd metatarsal. What would be the best surgical option? A. Plantar plate repair B. Flexor tendon transfer with a distal metatarsal osteotomy C. 2nd digit amputation D. Extensor digitorum longus transfer and PIPJ arthrodesis

Correct answer: (C) 2nd digit amputation. Explanation: Digital amputation can play a very important role in the geriatric population with severe digital deformities. The crossover deformity is complicated and surgical correction would be extensive. The second toe amputation would heal quickly and provide immediate relief. (A) A plantar plate repair would likely be too extensive for this patient. (B) Similar to above the flexor tendon transfer with a distal metatarsal osteotomy would likely be too extensive for this patient. (D) Extensor digitorum longus transfer and PIPJ arthrodesis would likely be too extensive for this patient.

Ultrasonography has been shown to be a very effective modality in diagnosis of intermetatarsal neuromas. With ultrasound, the interdigital nerve diameter can be measured with accurate and reproducible results. The normal diameter of an interdigital nerve in the forefoot is 2 mm. What is the diameter threshold where it is commonly believed interdigital nerves become symptomatic? A. 3 mm B. 4 mm C. 5 mm D. 6 mm

Correct answer: (C) 5 mm. Explanation: Ultrasound has developed into a very effective imaging modality in the work up of inter metatarsal neuromas. Findings during ultrasound exams have shown to be between 95% to 98% accurate with what has been seen intraoperatively. Tremendous detail can be garnered from these exams. One such piece of information that can be ascertained from ultrasound is the diameter of the interdigital nerve. Normal interdigital nerve anatomy in the forefoot exhibits a nerve diameter of about 2 mm. We know that form the pathogenesis of a neuroma, or more accurately perineural fibrosis, that as the nerve becomes more irritated/injured it enlarges as fibrosis occurs. Levine, Myerson, and Shapiro demonstrated that when an interdigital nerve reaches a diameter of 5 mm or more it becomes symptomatic. An important distinction in the question is that specifically the nerve diameter threshold was asked. With that being said the answer as to what the nerve diameter threshold for when the interdigital nerve becomes symptomatic is 5 mm.

To take a sesamoid axial projection, the foot is positioned with the front of the poser device placed against the cassette so the toes are closest to the film. What is the appropriate tubehead angulation and where will the central ray be directed? A. 45 degrees from vertical and center of the calcaneus posteriorly B. 0 degrees from vertical and center of back of axial poser C. 90 degrees from vertical and center of axial poser D. 0 degrees from vertical and center of axial poser

Correct answer: (C) 90 degrees from vertical and center of axial poser. Explanation: 90 degrees from vertical and center of the axial poser, the film placement is upright in slot or well of orthoposer. The patient stands on the axialposer, heel and toes are elevated so that the metatarsophalangeal joints are the most inferiorly positioned anatomic structures. The lead apron is on the patient's back.

All of the following are true about electrocoagulation except? It is a method of thermal hemostasis. There is more damage to the surrounding soft tissue to due the use of heat to achieve hemostasis as compared to mechanical methods. A monopolar electrode produces a smaller area of tissue damage while a bipolar electrode produces a larger area of tissue damage. Electrocoagulation is not synonymous with electrocautery.

Correct answer: (C) A monopolar electrode produces a smaller area of tissue damage while a bipolar electrode produces a larger area of tissue damage. Explanation: Electrocoagulation is a commonly employed thermal hemostatic method in the surgical arena. Due to the dispersement of an electrical current through the tissue which produces heat, there is an area of tissue damage that is unavoidable. The bipolar forcep style electrode only passes electricity between the two prongs which produces a more narrow field of tissue damage when compared to the monopolar electrode which delivers an electrical current to the tissue and a grounding pad grounds the patient. Mechanical methods such as direct pressure or ligation do not cause as much damage as thermal methods. Electrocautery uses electricity to heat only a filament tip and does not apply electricity directly to the patient.

You are scheduling a bunionectomy surgery a patient on 25 mg prednisone for two months, finished her treatment 2 months ago. Please choose the appropriate answer. Continue with scheduled surgery, no precautions needed, her hypothalamic pituitary axis should be able to handle the stress for surgery. Postpone surgery for another 1 month. Administer 50 to 75 mg hydrocortisone on the day of surgery. Administer 50 to 75 mg hydrocortisone the day before surgery.

Correct answer: (C) Administer 50 to 75 mg hydrocortisone on the day of surgery. Explanation: It is always necessary to clarify the duration of prior corticosteroid therapy. In general any patient who received the equivalent of 20 mg/d of prednisone for greater than 5 days is at risk of suppression of the hypothalamic pituitary axis, and if they have been on therapy for approximately a month, they may have HPA suppression for up to 6-to 12 months. This patient was on prednisone for more than a month, with a dose of 25 mg, therefore she is at risk of developing HPA suppression, and she is undergoing a bunionectomy which can be considered a moderate stress, then it is recommended 50 to 75 mg of hydrocortisone the day of surgery. (A) To avoid adrenal crisis with this patient, it is recommended a dose of glucocorticosteroids the day of surgery. (B) A patient with a recent history of glucocorticoid therapy of 20 mg or more for greater than a month, is at risk of HPA suppression for 6 to 12 months. Postponing the surgery another month for this patient still does not rule out the risk of HPA crisis. (D) For this patient 50 to 75 mg of hydrocortisone should be administer the day of surgery, not 24 hours before.

After performing a tumbling pass, 19 year old female gymnast felt a sudden, sharp pain on the outer aspect of her left foot. Immediately she was not able to bear weight on that aspect of her foot. She is a healthy female on no medication, negative past medical history, past surgical history of tonsillectomy and is a non-smoker. Evaluation at the emergency department revealed point tenderness with localized edema left foot. Early ecchymotic changes noted as well. Her skin is intact. The following image was obtained. Which would be the most appropriate initial treatment? Percutaneous intermedullary screw fixation with nonweightbearing below the knee cast for 4-6 weeks. Placement of small unilateral external fixator and nonweightbearing for four weeks. Application of a weightbearing below the knee cast for 4-6 weeks. Application of a nonweightbearing below knee cast for 4-6 weeks.

Correct answer: (C) Application of a weightbearing below the knee cast for 4-6 weeks. Explanation: The standard of care for a non displaced or minimally displaced distal fifth metatarsal fracture is a below the knee weight bearing cast worn for 4-6 weeks. If there is concern about stability the first 2-3 weeks can be nonweightbearing, but as can be seen on the radiograph above with the lateral cortex intact this provides an inherently stable scenario. Percutaneous intermediary screw fixation is only for Jones-type fractures not distal ones as shown above. The placement of a unilateral external fixture is not indicated and not the standard of care. Again as mentioned previously, there is not a need to nonweightbear this cast as in the last answer as the intact lateral cortex provides some inherent stability.

Middle facet coalitions of the subtalar joint severely restrict hindfoot frontal plane motion. What is an adaptive compensatory change sometimes see due this restriction of frontal plane hindfoot motion? A. Rearfoot valgus B. Forefoot supinatus C. Ball and socket ankle D. Talar beaking

Correct answer: (C) Ball and socket ankle. Explanation: Due to the severe restrict of frontal plane hindfoot motion, a ball and socket ankle joint can form. This is a direct adaptation stemming from the restricted subtalar motion. (A) A rearfoot valgus deformity is not an adaptive change seen in the middle facet subtalar joint coalition. (B) A forefoot supinatus deformity is not an adaptive change seen in the middle facet subtalar joint coalition. (D) Talar beaking is not an adaptive change seen in the middle facet subtalar joint coalition.

Kirschner wires and Steinmann pins can cause increase temperature in the bone during insertion because of the following: A. Hard bone B. Wire driver C. Because of their lack of cutting facets and flutes D. Surgeon inexperience

Correct answer: (C) Because of their lack of cutting facets and flutes. Explanation: Heat production during insertion of Kirschner wires can be significant because these wires lack true cutting facets and flutes. Overheating of bone during wire insertion can be associated with greater degrees of bone necrosis. The extent of heat production during the passage of fine Kirschner wires (0.8 to 2 mm) and stainless steel Kirschner wires with trocar-tip and diamond-tip configurations were tested in metatarsal bones. Heat production was significantly greater for thinner (less than 1.1 mm) versus thicker wires. Tip geometry influences the amount of heat produced during insertion. Tip designs that allow for clearance of chip debris during drilling are associated with lower temperatures. The diamond tip has a flattened region behind the cutting edge that facilitates clearance of bone debris. Diamond-tip wires are associated with lower temperatures during insertion compared with wires with trocar or smooth tips. Tip design can also influence pullout strength of wires. The cutting edges of a trocar-tip wire generate a circular defect for its shaft, whereas the diamond tip produces an elliptic hole even though its shaft is circular. Lack of congruence between the drill hole and the wire shaft reduces pullout resistance associated with lower temperatures during insertion compared with wires with trocar or smooth tips. Tip design can also influence pullout strength of wires. The cutting edges of a trocar-tip wire generate a circular defect for its shaft, whereas the diamond tip produces an elliptic hole even though its shaft is circular. Lack of congruence between the drill hole and the wire shaft reduces pullout resistance.

What advantage does ultrasound have over MRI when it pertains to the evaluation of the Achilles tendon? A. Provides more detailed information B. Can show abnormalities in size C. Can be used intra-operatively D. Can show abnormalities in substance

Correct answer: (C) Can be used intra-operatively. Explanation: One of the advantages of ultrasound over MRI is the ability to use ultrasound intra- operatively to image Achilles tendons. Like an MRI, the ultrasound can give information both on size and substance as well. (A) The MRI provides far more detail as compared to ultrasound in regards to imaging Achilles tendon. (B) Both MRI and ultrasound are able to shoe abnormalities in size. (D) Both MRI and ultrasound are able to shoe abnormalities in substance.

Upon reviewing a lateral projection foot radiograph, it is noted that there is a 33o calcaneal inclination angle, elevation of the first and fifth metatarsal, and a posterior break in the cyma line. What type of foot is this? A. Planus B. Rectus C. Cavus D. Equinovarus

Correct answer: (C) Cavus. Explanation: Radiographically, the pes cavus foot type is characterized by its distinctive high arch morphology. The cavus foot type is best evaluated with weight bearing dorsoplantar and lateral projections in the angle and base of gait. On the lateral projection an increased calcaneal inclination angle exceeding 30o is revealed. There is also an increased inclination of the first and fifth metatarsals. An accentuation of the subtalar joint region as well as the sinus tarsi is also usually noted, and it is not uncommon to find a calcaneocuboid joint fault with digital contractures. The cyma line in the cavus foot type will present with a posterior break. An illustrative example is provided below.

Following injection of local anesthetic, the patient becomes restless, nervous, dizzy and develops confusion, tremors and eventually convulsions. This may be attributed to: A. Dermal reaction from topical ethyl chloride B. Anaphylaxis reaction from lidocaine C. Central nervous system effects of lidocaine toxicity D. Cardiovascular manifestations of lidocaine toxicity

Correct answer: (C) Central nervous system effects of lidocaine toxicity. Explanation: Generally, lidocaine toxicity presents initially with CNS symptoms including restlessness, nervousness, dizziness, confusion, tremors and convulsion seizures. (A) Ethyl chloride is used as a topical anesthetic prior to injection. However, the symptoms described involve more than a topical skin reaction. (B) An anaphylaxis reaction may present with hoarseness, wheezing and diaphoresis. In the case of an allergic reaction, epinephrine should be administered. (D) Cardiovascular manifestations of lidocaine toxicity may include bradycardia, hypotension and ventricular arrhythmias.

A 44-year-old male patient presents for evaluation of a warm, swollen foot. He states that he twisted his foot a few days ago and initially he did not feel much pain. Over the next few days, he noticed his foot became progressively warm and swollen, but he still denies pain. His medical history is significant for insulin dependent diabetes mellitus (IDDM). His most recent glycosylated hemoglobin level (HbA1c) was 9.4%. He is afebrile with normal vital signs and examination reveals warmth, erythema, and edema over the dorsal aspect of the right foot, but there are no obvious bony deformities and he has no open lesions. X-rays show complete fragmentation of the Lisfranc articulation with dislocation and subluxation of bones. What is the most likely diagnosis? A. Osteomyelitis B. Gout C. Charcot neuroarthropathy D. Cellulitis

Correct answer: (C) Charcot neuroarthropathy. Explanation: The differential diagnoses of a unilateral swollen joint includes all of the choices given, but based upon the radiographic appearance of the patient's joint, the diagnosis is Charcot neuroarthropathy. The best choice would be Charcot neuroarthropathy, based on the radiographic appearance of the affected. Charcot joint is a disorder sometimes seen in patients with neuropathy (DM, HIV, Leprosy, Tabes dorsalis). Diabetic patients primarily develop neuroarthropathy of the foot and ankle joints. The clinical picture demonstrates swelling and warmth with minimal pain, with absence of constitutional signs associated with infection, often in the setting of a history of minor trauma. There are 3 stages that can be demonstrated radiographically: the acute fragmentation stage; a coalescent stage, characterized by fracture healing, bony debris resorption, and formation of new bone: in the remodeling stage, signs of inflammation diminish and radiographic evidence of bony deformity is seen, including "rocker- bottom" feet. Immobilization is the modality of choice during the acute and coalescence stage. Early recognition is important so that prompt treatment can be initiated, as late stage changes are frequently irreversible. Osteomyelitis is less likely as there is no open lesion or ulceration. A bone biopsy can distinguish the two. Gout does not usually show complete destruction and subluxation of the bones. Cellulitis alone does not cause any bony changes as this is an infection of the skin.

A 52 year old female presents complaint of pain and irritation of her bunion area. She states that about a year and a half ago she had a bunionectomy performed. Ever since then she notice a bump at the top of the joint that gets rubbed by shoes and becomes painful. She denies pain on the ball of the foot. What would be the procedure of choice? A. Sagittal plane distal crescentic osteotomy B. Waterman procedure C. Cheilectomy D. Dorsally based proximal opening wedge osteotomy

Correct answer: (C) Cheilectomy. Explanation: The cheilectomy is the simplest and most effective procedure to deal with the problem described above. As the patient expresses no plantar pain or metatarsalgia-type pain then all the patient's symptomatology arises from irritation of the dorsal prominence and possibly joint jamming. The cause of the dorsal prominence if a dorsally shifted bunion procedure malunion. (A) If adjacent lesser metatarsalgia was present then perhaps a revision osteotomy like the sagittal plane distal crescentic osteotomy would be indicated. However, in the example above not evidence of metatarsalgia was not present. (B) A Waterman procedure wound not be indicated here for a couple of reasons. First there is no metatarsalgia to suggest the need to plantar flex the capital fragment. Secondly, with the removal of the wedge there would be additional shortening of the metatarsal. (D) A dorsally based proximal opening wedge osteotomy would plantarflex the metatarsal head which as explained previously is not necessary as there are no symptoms of metatarsalgia.

When performing debridement, what is a good clinical indicator of the viability of a muscle? A. Bleeding B. Coloration C. Contractility D. Consistency

Correct answer: (C) Contractility. Explanation: The most consistent indicator of muscle viability if its contractile or twitching response to manual and/or electrical stimulation. Twitching and muscle contracture can only occur if the muscle fibers are still viable. (A) Bleeding is not a a reliable indicator of muscle viability as nonviable muscle has been shown to bleed for several hours. (B) The color of muscle is also not a consistent reliable indicator of muscle viability. (D) The consistency and/or texture of muscle is also not a good clinical indicator of muscle viability.

A 51 year old female presents to clinic complaining of pain in the ball of her right foot. Physical exam a mild hallux valgus deformity with a hypermobile first ray. Additionally sagittal plane con-tracture is noted of the second and third digits and an adductovarus deformity is noted on the fourth and fifth digits. There is a medial deviation of the second digit, however the second digit does not contact or underlap to overlap the hallux. There is a somewhat diffuse painful area under the third and fourth met head and in the 3rd interspace. Additionally there is pain with a side to side compression of the right forefoot. Below is a ultrasound image of the patient's right foot. With the information provided and the clinical image below what would be the most appropriate treatment option listed? A. Implementation of a bone stimulator B. Budin splint/toe strapping C. Corticosteroid injection third interspace D. MRI

Correct answer: (C) Corticosteroid injection third interspace. Explanation: The foot described above is a foot prone to many problems. It is an overpronated foot with a hypermobile first ray. This propagated a hallux valgus deformity and concurring pa- thology related to lateral overload of pedal pressure due to the insufficiency of the first ray not able to bear its portion of weight. All the conditions listed above are possible with this foot type. However, the key component in arriving to the correct answer rests in the diagnostic ultrasound picture. Highlighted by the yellow arrow is a sizable neuroma in the third interspace (Morton's neuroma). It can be seen protruding plantarly beyond the metatarsal heads and occupying the entirety of the interspace. With that being said the most appropriate treatment for a neuroma would be a corticosteroid injection.

If sutures are necessary in an infected wound, all are acceptable choices except: A. Monofilament nylon B. Polypropylene C. Cotton D. Polybutester

Correct answer: (C) Cotton. Explanation: The use of sutures in infected or contaminated wounds should be avoided altogether if possible. When sutures are necessary in these instances, synthetic, monofilament materials such as polybutester, polypropylene, or monofilament nylon are best suited. Natural fiber nonabsorbables such as untreated silk and cotton have the highest potential for converting a contaminated wound into an infected wound, whereas monofilament synthetics such as nylon and polypropylene have the least.

In regards to skin flap physiology which term below is best defined as the increase in strain seen when skin is under constant stress? A. Stress B. Strain C. Creep D. Stress relaxation

Correct answer: (C) Creep. Explanation: There are four important concepts pertaining to flap physiology: stress, strain, creep and stress relaxation. Creep refers to the increase in strain seen when skin is under constant stress. This occurs in a matter of minutes and is due to an extrusion of fluid from the dermis and a breakdown of the dermal framework. Strain refers to the change in length divided by the original length of the given tissue to which a force is applied. Stress refers to the force applied per cross-sectional area. Stress relaxation is the decrease in stress when skin is held in tension at a constant strain for a given time. This occurs over a matter of days to weeks and is due to an increase in skin cellularity and the permanent stretching of skin components.

An RA patient is scheduled for a Hoffmann procedure, first MPJ fusion, and lesser digital arthrodesis, left foot. During your pre-operative H+P, the patient relates a feeling of "something in my throat all the time", periods of hoarseness, and odonophagia. On physical examination, pain with pressure is elicited against the anterior aspect of the thyroid cartilage. Based upon your history and physical findings, what do you suspect? A. Strep pharynigitis B. Laryngeal carcinoma C. Cricoarytenoid arthritis D. Esophagitis

Correct answer: (C) Cricoarytenoid arthritis. Explanation: Although this could be many things - strep pharynigitis, laryngeal carcinoma, or esophagitis are possibilities-the history of RA combined with the signs and symptoms should lead you to this diagnosis. It is important in taking the Boards, to first use your history to lead you to the most logical diagnosis. Here, the incidence of cricoarytenoid arthritis in RA is greater than first thought. Lofgren reported an incidence of 26%. Upon autopsy, the incidence can be as high as 85%. Failure to diagnose this problem in the surgical RA patient can result in tremendous intubation problems at surgery. Laryngoscopy can be utilized pre-operatively. (A) Symptoms may include erythema, fever, and rash with cervical adenopathy. A culture will help in ruling this diagnosis in or out. (B) Signs and symptoms can include hemoptysis, weight loss, unilateral cervical adenopathy, and a neck mass. (D) Signs and symptoms can include acid reflux, N+V, a cough, and chest pain.

What would be the appropriate treatment for the fracture blister pictured below? A. Compressive dressing B. Aspiration C. Deroofing D. No treatment

Correct answer: (C) Deroofing. Explanation: Hemorrhagic fracture blisters are always deroofed and then treated as a second- degree burn. The base is usually covered with a silver sulfadiazine and a non adherent gauge dressing. Reepithelialization occurs in 3 to 7 days at which time incisions can be made directly through the area. The presence of any type of fracture blister will delay the timing of surgical intervention. In the foot, calcaneal and ankle fractures are more likely to involve fracture blisters. The presence of fracture blisters lessens the incidence of compartment syndrome.

Systemic toxicity is more likely in which of the following anesthetics and why? A. Lidocaine because breaks down in the blood B. Marcaine because it breaks down in blood C. Marcaine because it breaks down in liver D. Procaine because it breaks down in liver

Correct answer: (C) Marcaine because it breaks down in liver. Explanation: This question is two fold. It is important to remember that amides (lidocaine and marcaine) are broken down in the liver and esters (procaine, chlorprocaine) are broken down in the blood. Metabolism of amides is slower than that of esters because it occurs in the liver and since it is slower, the anesthetic stays in the body longer and thus has a higher likelihood of systemic toxicity.

The patient pictured below, who had Keller bunionectomies on bilateral feet, presents complaining of a recent onset of painful calluses sub second metatarsal head. Her radiograph is pictured below. Based on her radiograph what is the most likely cause of her calluses? A. Elongated second metatarsals B. Long second digit C. Dysfunction of the first metatarsophalangeal joint D. Plantarflexed nature of the second metatarsal

Correct answer: (C) Dysfunction of the first metatarsophalangeal joint. Explanation: The above pictures demonstrates an individual who has had a Keller procedure or resection of the first proximal phalangeal base. This has led to severe dysfunction of the first metatarsophalangeal joint. This is not only indicative by the calluses, but also by the cortical wall thickening of the second metatarsals. The second metatarsal length is not the reason for the calluses. The first metatarsal have not been operated on so the metatarsal parabola is unaltered. The long second digit is exaggerated by the severe shortening of the hallux with the Keller. Additionally from the x-ray above it is not possible to determine the sagittal plane position of the second metatarsal.

Which of the following, when added to local anesthetics, increases the toxic dose of that anesthetic? A. Saline B. Dextrose C. Epinephrine D. Steroids

Correct answer: (C) Epinephrine. Explanation: Epinephrine leads to vasoconstriction and thus slows absorption of the local anesthetic, and therefore higher doses can be used safely.

Which is the most common factor associated with hallux abducto valgus deformity? A. Family history B. Long first metatarsal C. Female gender D. Round metatarsal head E. Decreased mobility of first metatarsocuneiform joint

Correct answer: (C) Female gender. Explanation: Hallux abducto valgus is associated with hypermobility of the first metatarsocuneiform joint. Family history is found in 83% of cases of hallux abducto valgus, female gender in 90-92%. Long first metatarsal is found in 71% of cases, oval or round metatarsal head also in 71% of cases.

A 75 year old IDDM patient whose last HbA1c was 11.2 presented to the ED with a chief complaint of ulceration and pain in his left foot with malodor and redness. Blood work was drawn on him with a WBC: 28.4 and Blood Sugar 336 mg/dl. The podiatry resident was called on the phone and asked the ED physician to order x-rays. Once complete the ED physician called the podiatry resident to tell him that the x-ray was read as no bony changes but + radio-lucency in the soft tissues surrounding the ulceration. What diagnosis is most likely with these findings and what is the most common organism causing it? A. Osteomyelitis- Staph aureus B. Gas Gangrene- Staph aureus C. Gas Gangrene- Clostridium perfringes D. Osteomyelitis- clostridium perfringes

Correct answer: (C) Gas Gangrene- Clostridium perfringes. Explanation: By only reading the clinical and lab part of this question, you should be able to determine that it is a type of infectious process as the WBC is elevated as well as the presence of pain, redness and malodor. As you continued reading, you notice that bone changes were negative which excludes osteomyelitis in this question. In reality radiographic change does lag clinical bone change by 10-14 days therefore this patient may have osteomyelitis but this is not the best answer for this question. As you continue reading, it discussed radiolucency in the soft tissue (aka soft tissue emphysema.) This fits the clinical picture as well. The most common organism causing soft tissue emphysema or gas gangrene is the clostridium species. The most common organism causing osteomyelitis is staph aureus.

When performing the crescentic procedure for a bunionectomy, what is the necessary orientation of the saw blade when performing the osteotomy? A. Perpendicular to the bottom of the foot B. Perpendicular to the first metatarsal C. Halfway between perpendicular to the first metatarsal and perpendicular to the bottom of the foot D. Perpendicular to the floor

Correct answer: (C) Halfway between perpendicular to the first metatarsal and perpendicular to the bottom of the foot. Explanation: The correct plane of the osteotomy in the crescentic procedure is halfway between perpendicular to the first metatarsal and perpendicular to the bottom of the foot. It provides optimal bone contact and predictable correction. (A) An osteotomy perpendicular to the bottom of the foot would cause dorsiflexion with rotation. (B) An osteotomy perpendicular to the first metatarsal would create an osteotomy too oblique and decrease bone contact of the osteotomy. (D) An osteotomy perpendicular to the bottom of the floor would cause dorsiflexion with rotation.

A 9 year old boy complains of right heel pain at the end of the soccer season. X-rays are obtained and the calcaneal apophysis is noted to be sclerotic and slightly fragmented. There are no fractures, no bone tumors or associated pathology. Which is the best treatment course at this time? A. Surgical intervention to fixate the fragmented apophysis B. Non-weightbearing in a below knee cast 8-12 weeks C. Heel cushion in athletic shoes and monitor D. Arthroereisis of the subtalar joint

Correct answer: (C) Heel cushion in athletic shoes and monitor. Explanation: Sclerosis of the apophysis, seen on x-ray as increased density compared to the surrounding calcaneal bone, is a normal finding of weight bearing. Fragmentation may also be a normal finding as there are multiple ossification centers. Initial conservative treatment should include heel cushions and decreased activity if warranted. (A) Surgical intervention is not warranted in calcaneal apophysitis as the symptoms will resolve with completion of growth, even in cases of chronic pain. If symptoms persist after closure of the growth plates, the diagnosis should be re-evaluated. (B) If the heel pain is severe, the child may require 4-6 weeks in a below knee cast for heel relief. However, 8-12 weeks would be excessive and would not be the initial conservative treatment. Alternate pathologies should be ruled out if pain persists. (D) Arthroereisis of the subtalar joint has no indication in calcaneal apophysitis.

Which of the following dietary supplements should not be discontinued one week before surgery? A. Ginger B. Ginkgo biloba C. Hyssop D. Ginseng E. Vitamin E

Correct answer: (C) Hyssop. Explanation: Ginger, Ginkgo biloba, ginseng, Vitamin E, as well as garlic and fish oil are among the dietary supplements that alter coagulation and should be discontinued one week preoperatively.

Which of the following is an absolute contraindication when using epinephrine with local anesthesia? A. Pregnancy B. Psychological instability C. Severe peripheral vascular disease D. Patient on Lithium

Correct answer: (C) Severe peripheral vascular disease. Explanation: Epinephrine is contraindicated in patients with pheochromocytoma, hyperthyroidism, severe hypertension, and severe peripheral vascular disease. (A) This is a relative contraindication in pregnancy. (B) Psychological instability is also a relative contraindication. (D) Lithium is a mood stabilizer that can be used in psychological instability, which is a relative contraindication, not an absolute contraindication.

A patient is in the operating room for excision of plantar fibroma left foot. The area has been bothering him for a long time and he would like excision of the lesion. The surgeon decides to remove the entire medial band of the plantar fascia so the patient has the lowest chance of recurrence. Which of the following is the correct order of structures seen from most plantar to most dorsal? I: Abductor Hallucis II: Medial band plantar fascia III: Flexor Digitorum Longus IV: 1st dorsal interossei A. I,II,III,IV B. II,I,IV,III C. II,I,III,IV D. IV,III,I,II

Correct answer: (C) II,I,III,IV. Explanation: This question is testing to see if you know the anatomy of the plantar aspect of the foot. The most plantar structure of the above is the plantar fascia which has three different bands. (Medial, Central, Lateral) The first layer of muscles in the plantar aspect of the foot include the abductor hallucis, flexor digitorum brevis and abductor digiti minimi. The second layer of muscles include the quadratus plantae and the lumbricals. The flexor digitorum longus also runs within this layer. The third layer of plantar musculature includes the adductor hallucis, flexor digiti minimi brevis and the flexor hallucis brevis. The fourth layer (the most dorsal layer although still on the bottom of the foot) contains the dorsal and plantar interossei.

A 45 y/o female presents with a previous history of tarsal tunnel surgery one year ago. She relates the pain is now actually worse; she did have short-term improvement after the surgery. Clinical inspection reveals a 3cm incision at approximately the superior aspect of the flexor retinaculum behind the medial malleolus. With percussion and palpation, there is no pain at the incision site, but there is pain at the inferior aspect of the retinaculum. The foot is warm. Muscle strength is 4/5 to all tested muscles, with normal muscle mass. The stretch test is negative. Normal foot structure. X-rays unremarkable. The most likely cause of this failed surgery is: A. Mis-diagnosis B. Adhesive neuritis C. Incomplete release D. CRPS

Correct answer: (C) Incomplete release. Explanation: The small incision leads one to think of an endoscopic release . With the incision's location and no pain at that location, the nerve was adequately release there, but not released at the distal branches; the MPN and LPN. The fascia of the abductor hallucis was not released. If this is the case, there is pain distally, at the inferior aspect of the retinaculum, where the two branches and the muscle are located. (A) If a patient has a mis-diagnosis, the patient will have no relief after surgery, not even short- term. TPTD with heel valgus stretching the tibial nerve, space-occupying lesions within the canal, or lumbosacral plexopathy are some diagnoses to be considered. (B) Adhesive neuritis is secondary to external scarring around the nerve, sometimes from aggressive dissection and post-operative bleeding, which is highly unlikely with an endoscopic release. There may be temporary relief of symptoms. With adhesive neuralgia, there is pain over the scar, with ankle joint ROM pain. A stretch test holds the foot in maximum DF and eversion for thirty seconds; this may reproduce the symptoms. (D) The surgery occurred one year ago. By this time, if CRPS had developed, it would be at least Stage II, maybe III, with muscle atrophy, unrelenting and unbearable pain, osteopenia on x-ray, foot deformities, vasomotor dysfunction, and significant motor and trophic changes.

One of the following fracture fixations is referred to as a splint: A. Rollabout knee walker B. Fracture sling C. Intramedullary rod D. Biologic fixation

Correct answer: (C) Intramedullary rod. Explanation: There two basically different mechanisms of fracture fixation: splinting and compression. The two differ in the mechanism of stabilization and in the degree of stability achieved. Splinting is a fracture fixation that consists in connecting a stiff device to the fractured bone. This device reduces mobility of the fracture in proportion to its stiffness. Splinting is achieved using various methods, ranging from external splints such as fiberglass casts, internal fixation by plates (buttress), intramedullary nails, and external fixators.

Which inhaled anesthetic has the following characteristics? - MAC = 1.2 - minimal cardiac depression - pungent odor - better maintenance than induction A. sevoflurane B. halothane C. isoflurane D. desflurane

Correct answer: (C) Isoflurane. Explanation: Isoflurane is a halogenated ether originally used for both induction and maintenance anesthesia. Isoflurane has a minimum alveolar concentration of 1.2. It's pungent odor prohibits its use for induction in modern day anesthesia. It has largely been replaced with sevoflurane, desflurane and the intravenous induction agent, propofol. Typically, isoflurane is administered in combination with air, oxygen, and/or nitrous oxide.

Which of the following is true of ketamine? A. It causes muscle paralysis. B. It is an NMDA receptor agonist. C. It causes amnesia. D. It is a schedule II drug.

Correct answer: (C) It causes amnesia. Explanation: Ketamine is an NMDA receptor antagonist that is most commonly used for starting and maintaining anesthesia. It causes amnesia but does not cause paralysis (reflexes and breathing generally remain normal). It is a schedule III drug, meaning it has potential for abuse but less so than schedule I and II drugs and has medically acceptable uses. Ketamine is commonly used in pediatric patients undergoing painful or possibly traumatic procedures.

A patient with a large laceration undergoes local anesthetic infiltration with the long-acting anesthetic agent, achieving 9 hours of anesthesia. Which of the following is true of the medication that was most probably used in this case? Increases membrane permeability to sodium ions. It has been associated with seizures and cardiopulmonary arrest in patients under the age of 3. It is an amide. It works by cooling the skin surface to desensitize pain receptors. Anesthetic is concentrated into micron-sized droplets that increase efficacy.

Correct answer: (C) It is an amide. Explanation: The longest lasting local anesthetic is bupivacaine, an amide agent. It lasts 2 to 8 hours. (A) Bupivacaine decreases membrane permeability to sodium ions. (B) Lidocaine has been associated with seizures and cardiopulmonary arrest in patients under the age of 3. (D) Vapocoolant sprays act by surface cooling of the skin immediately before venipuncture, IM injection, or placement of an intravenous catheter. Ethyl chloride is a commonly used vapocoolant spray. (E) EMLA is a topical anesthetic that combines lidocaine and prilocaine in a cream base. The anesthetics are concentrated in micron-sized droplets, which, along with the high concentration gradient, promote penetration of anesthetic through intact skin to result in a sensory and motor blockade primarily by reversible sodium channel inhibition at the axonal membrane to result in blocked action potential conduction.

A 23 year old male presents to the emergency room with a laceration on the plantar surface of the left foot. He reports stepping on a piece of glass. He complains of weakness in his third, fourth and fifth toes. On exam the laceration measure 2.3 cm in length and is located mid arch plantar to the cuneiform complex. When the clinician stabilizes the metatarsophalangeal joints and asks the patient to flex his digits he is unable to flex the third and fourth digits. What is the diagnosis? A. laceration of the third intermetatarsal nerve B. laceration of the flexor digitorum brevis C. laceration of the flexor digitorum slips to the third and fourth toes D. interdigital traumatic neuroma

Correct answer: (C) Laceration of the flexor digitorum slips to the third and fourth toes. Explanation: Ruptures of the flexor digitorum longus tendon nearly always occur secondary to a penetrating wound. The other circumstance where flexor digitorum longus tendon ruptures have occurred is after repeated corticosteroid injections for an interdigital neuroma. The biggest key point in this scenario is that when the digits were immobilized at the mtp joint level the patient was not able to flex the third and fourth digits. As the long flexors attach on the distal phalanx the fact that the digits wouldn't flex demonstrates the disruption of the flexor tendon slips. In other words, because specific function of the FDL is at the DIP joint, lack of flexion power indicates a rupture.

Two months after performing an excision of a plantar fibroma, hyperextension at the metatarsophalangeal joints of the hallux, second and third digits is noted. A possible cause could be an iatrogenic injury to which of the following muscles or nerves? A. Saphenous nerve B. Flexor digitorum minimi C. Medial plantar nerve D. Quadratus plantae

Correct answer: (C) Medial plantar nerve. Explanation: The medial plantar nerve provides innervation to the plantar musculature to the hallux, second and third digits. With their attachments on the proximal phalanges of the respective digits and a nonfunctioning nerve the extensors would now be unopposed at the metatarsophalangeal joint level. This would thereby result in hyperextension of the metatarsophalangeal joint of the hallux, second and third digits. Transection of the quadratus plantae would result in adductovarus deviation of the fourth and fifth digits. The saphenous nerve is completely sensory and if injured would not result in a mechanical deformity.

When calculating the annual dose limit for a radiographer, which of the following radiation sources are included? A. Cosmic radiation B. Medical procedures C. Occupational exposures D. Radon E. All of the above

Correct answer: (C) Occupational exposures. Explanation: When calculating radiation exposure annual dose limit for radiographers and other medical personnel, only occupational exposures are included.

A 47 year old male has returned to the office for follow-up on a MRI of his left foot. The MRI confirmed the presence of an aneurysmal bone cyst. Below is an image from the MRI. What would be the most appropriate next step in the treatment of this individual? A. Offload in a CAM walker B. Surgical debridement and packing C. Oncology referral D. Vascular referral

Correct answer: (C) Oncology referral. Explanation: Aneurysmal bone cysts almost always occur in children and young adults. In patients over 20 years old, an aneurysmal bone cyst is usually associated with an underlying bone tumor such as a giant cell tumor, fibrous dysplasia, chondroblastoma, or chondromyxoid fibro-ma. This, in turn, means that whenever an aneurysmal bone cyst is found in an individual older than twenty, that the practice of good medicine to refer the individual to oncology for workup and exploration for the likely present bone tumor.

What is the standard of care for lesser metatarsal neck fractures with joint dislocation? A. Closed reduction B. Closed reduction with percutaneous fixation C. Open reduction and internal fixation D. Cast immobilization and nonweightbearing for 4 to 6 weeks

Correct answer: (C) Open reduction and internal fixation. Explanation: Open reduction and internal fixation is the standard of care for lesser metatarsal neck fractures with joint location. Closed reduction is almost always unsuccessful as the metatarsal head often gets wedges under the plantar plate. The mechanism of injury for this fracture is an excessive dorsiflexion of the proximal phalanx over the metatarsal head. A retrograde plantarflexory force against the metatarsal against the metatarsal head results. (A) (B) For the reasons stated above, the metatarsal head becomes stuck in a plantar flexed attitude which makes closed reduction impossible. (D) Cast immobilization and nonweightbearing for 4-6 weeks fails to address the dislocation and therefore is not the correct choice.

Midstance phase is a period in the stance phase of gait. What initiates and ends this subphase of the gait cycle? A. Heel contact to toe off of the opposite limb B. Heel contact to toe of the same limb C. Opposite side toe off to heel lift of the support foot D. Heel off to heel strike of the opposite limb

Correct answer: (C) Opposite side toe off to heel lift of the support foot. Explanation: Midstance phase is that period from opposite side toe off to heel lift of the support limb. It occupies 20%of the gait cycle and 30% of the stance phase.

When does midstance phase of gait begin in the gait cycle? A. Heel contact B. Heel lift C. Opposite-side toe-off D. Opposite-side heel lift

Correct answer: (C) Opposite-side toe-off. Explanation: Midstance phase is the initial single support period which begins with opposite-side toe-off and full forefoot loading of the grounded limb. It ends at heel-lift of the ground foot. It is said to occur when one limb is responsible for supporting the entire weight of the body. During this time, the limb and trunk stability are key as the swing leg passes from posterior to the grounded leg to anterior to the grounded leg. This requires the foot to become more stable in order to preserve to forward momentum of the body in anticipation of the heel lift and increased ground reactive forces.

Which type of instrument is similar to a chisel except they have two surfaces tapering to form a fine cutting edge rather than one surface like that of a chisel? A. Periosteal Elevators B. Gouges C. Osteotomes D. Trephines

Correct answer: (C) Osteotomes. Explanation: Osteotomes are similar to chisels but differ from chisels in that both sides of the osteotome taper to a fine edge compared to a chisel in which only one surface of it tapers to meet the other flat surface. Both can used for osteotomies and resection of prominent boney edges. They are typically used with a surgical mallet to drive the osteotome or chisel through the bone. (A) Periosteal elevator is incorrect. The instruments have smooth and wide blades that vary in sharpness. They are typical used for reflecting the periosteum from the bone to which it is connected, as well as being occasionally used retracting deeper tissues. Examples of periosteal elevators include the freer elevator, Sayre elevator, key elevator, and Crego elevators. (B) Gouges are incorrect. Gouges are very similar to chisels in that they both only have one surface that tapers to a fine edge. However, gouges have a curved cutting edge making them useful for smoothing curved surfaces and rounding flat surfaces of bone. They come in a variety of sizes. One common gouge used in podiatric surgery is the McGlamery metatarsal elevator. (D) Trephines are incorrect. Trephines have a circular edge at the end of the instrument, which is typically T-shaped with a hollow tube. It is used to remove plugs of bone, such as for bone biopsies.

67 year old female with a pacemaker presents who requires a complex bunionectomy. What is the optimal approach on managing her cardiac issues? A. Order a cardiac stress test B. Risk is too high, she is not a good surgical candidate C. Obtain cardiac clearance prior to the procedure D. Order an echocardiography

Correct answer: (C) The patient should obtain cardiac clearance prior to the procedure. Explanation: The use of electrocautery intraoperatively can have adverse effects on the function of implanted cardiac devices. A variety of errors can occur from resetting of the device to inadvertent discharge of implantable cardioverters defibrillators. Complications are rare. But the primary care provider, and cardiologist should be informed of the upcoming procedure and the cardiologist should give cardiac clearance.

In the arena metatarsal fracture management, intramedullary nailing has grown in popularity. Although reaming of the metatarsal shaft provides internal bone grafting to the fracture site, it also obliterates the medullary afferent and efferent blood supply. It has been shown to take 2-3 weeks for the medullary canal to revascularize. During this time where does the fracture receive its vascular support? A. Nutrient artery B. Medullary blood flow C. Periosteum D. Adjacent musculature

Correct answer: (C) Periosteum. Explanation: The percutaneous nature of intramedullary nailing of metatarsal fractures preserves the periosteal tissue which can provide the necessary vascularity during the initial 2-3 weeks after medullary reaming. (A) The nutrient artery although very possibly intact can only provide blood flow to the distal segment. As it cannot span the fracture, it cannot provide blood flow to the proximal segment. (B) Reaming is to be performed 1 mm less that the minimal canal diameter. This leaves essentially no medullary contents intact and undisturbed. (D) Adjacent muscular does not provide in vascular support to the neighboring metatarsal.

These intrinsic muscles are unipennate: A. Flexor digitorum brevis muscle B. Dorsal interosseous muscles C. Plantar interosseous muscles D. Lumbricale muscles

Correct answer: (C) Plantar interosseous muscles. Explanation: The 3 plantar interossei are smaller than the corresponding dorsal interossei. They are single headed, fusiform, unipennate and arise from the inferior segment and border of the tibial surface of the third, fourth and fifth metatarsals.

Mechanism of action of the flexor digitorum longus: A. Plantarflexion at the MTPJ and IPJ B. Plantarflexion at the IPJ and DIPJ C. Plantarflexion at the DIPJ, the PIPJ and the MTPJ D. Plantarflexion at the PIPJ only

Correct answer: (C) Plantarflexion at the DIPJ, the PIPJ and the MTPJ. Explanation: Plantar flexes the lesser digits, begins at the distal interphalangeal joints, then the proximal interphalangeal joints and then the metatarsophalangeal joints.

When using calcium sulfate as a local antibiotic delivery vehicle which of the following statements is correct? A. typically requires removal B. use liquid antibiotic when mixing C. pre-made antibiotic loaded beads have 120 day shelf life D. elutes over 7 days

Correct answer: (C) Pre-made antibiotic loaded beads have 120 day shelf life. Explanation: Calcium sulfate is a popular vehicle that is used for local antibiotic delivery. It does not have even a fraction of the exothermic reaction of polymethylmethacrylate so nearly any antibiotic can be used. One caveat is that the antibiotics must be in powder form. Once implanted they break down and resorb therefore removal is not necessary. The elution profile shows a peak at 3 hours and it elutes over 72 hours. The beads can be pre-made with antibiotics and stored on the shelf for up to 120 days.

Which of the following statement regarding an open joint injury is correct?. Cannot occur without an intra-articular fracture To lessen the chance of a septic joint, closure of the capsule is immediate after copious irrigation Prevention of a septic joint is a major consideration Replenishment of synovial fluid can be performed by injection of hyaluronan approximately 1 week after capsular closure to accelerate recovery

Correct answer: (C) Prevention of a septic joint is a major consideration. Explanation: Prevention of a septic joint is an important tenet of treatment of open joint injuries. Open joint injuries comprise two different scenarios: An open fracture that enters into the joint (intra-articular) and an open joint injury without a fracture. Both types of injuries require an arthrotomy and copious amounts of irrigation. All recesses need to be exposed and inspected. Daily irrigation of the joint is performed. These wounds are packed open and closure occurs in 3 to 5 days. No indication for hyaluronan therapy has been associated with open joint injuries. Often external fixation is used to stabilize fractures in these injuries. Meticulous pin care is essential to lessen chance of infection. The fixator is removed as early as possible to facilitate mobilization.

Local anesthesia is commonly used in foot and ankle surgery. Which of the following is an ester type of anesthesia? A. Lidocaine B. Marcaine C. Procaine D. Mepivicaine

Correct answer: (C) Procaine. Explanation: The two types of local anesthetics are esters and amides. Esters are metabolized in the blood while amides are metabolized in the liver. Amides consist of Lidocaine, Marcaine, Mepivicaine and Prilocaine.

What is the primary purpose of the Green modification to the Reverdin osteotomy? A. Increase osteotomy stability B. Facilitate screw fixation C. Protect the sesamoids D. Increased proximal articular set angle correction

Correct answer: (C) Protect the sesamoids. Explanation: The Green modification of the Reverdin osteotomy is a plantar cut added to the inferior portion of the Reverdin which exits proximal to the sesamoids. The primary purpose of this modification is to protect the sesamoid apparatus thereby limiting the chance of post- operative sesamoid arthritis. (A) The Green modification of the Reverdin osteotomy does increase the stability of the osteotomy, but this is not the primary function of the modification. (B) The Green modification to the Reverdin osteotomy does little in facilitating screw fixation. (D) The Reverdin osteotomy is designed to primarily correct the proximal articular set angle. The Green modification does not alter the Reverdin's capacity in proximal articular set angle correction.

A 15 year old male and his mother present to your clinic. The teenager reports that his right foot is painful whenever he plays basketball. Running and excessive walking on uneven surfaces also seems to aggravate his symptoms. You suspect that the patient is suffering from a middle facet subtalar joint coalition and obtain x-rays to confirm this diagnosis. If your diagnosis is in fact correct, what would you likely see radiographically to support your diagnosis of a subtalar joint coalition? A. Anteater nose B. Putter sign C. Rounding of the lateral process of the talus D. Parallell middle & posterior facets on a Harris-Beath view

Correct answer: (C) Rounding of the lateral process of the talus. Explanation: The anteater nose is a term often used to denote a calcaneo-navicular coalition on a lateral foot projection. On this view, the anterior process of the calcaneus appears extended/lengthened at its superior aspect. This type of coalition does not involve the subtalar joint which consists of the anterior, middle and posterior facets between the talus and calcaneus. The putter sign is a radiographic finding on a dorsal-plantar foot projection, and its presence means there is a union between the talus and navicular. It looks like a putter used when golfing, hence the name.Again this type of coalition does not involve the talus and calcaneus. A Harris- Beath projection allows you to visualize both the middle and posterior subtalar joint facets in the axial view. In a normal foot without pathology, the facets are parallel to one another. When a coalition exists at the middle facet, the middle subtalar joint is obliquely oriented. The middle facet may be absent if an osseous union is present or may be narrowed if cartilage or fibrous tissue is uniting the two bones. Since the question lists parallel facets as an option, we know that answer (D) is incorrect. On a lateral foot projection, radiographic findings that are indicative of a middle facet STJ coalition are talar beaking, (C) sign, and flattening or rounding of the lateral process of the talus. Therefore, answer (C) is correct.

The Vassal principle of fracture fixation is a critical component in trauma surgery. What statement best describes the Vassal principle of fracture fixation? Fixation on tension side of the bone will allow physiologic compression and is an optimal fixation construct. When possible apply compression screw fixation in two planes. The first being perpendicular to the fracture line and the second being perpendicular to the long axis of the bone. Stabilization of the dominant fracture by rigid means will enhance the alignment of the lesser fractures because of all the soft tissue elements. A distance of at least 25% of the bone's cross-sectional width must be maintained from the nutrient artery of the fractured bone to prevent an avascular necrosis.

Correct answer: (C) Stabilization of the dominant fracture by rigid means will enhance the alignment of the lesser fractures because of all the soft tissue elements. Explanation: The Vassal principle of fracture fixation is best described by the third statement. It is a critical component to fracture fixation. The two most common examples in the foot and ankle of this principle occurs in multiple metatarsal fractures and fibular fractures. When the most dominant fracture is reduced and fixated the other fractures more or less fall in to place. Statements A and B are correct, but are not the Vassal principle. These are simply sound principles of internal fixation. On the other hand, statement D is erroneous and has no validity.

You are a podiatric resident on an anesthesia rotation. The patient was just given anesthesia and the patient has lost his swallow reflex and conjunctival reflex but his laryngeal reflex and corneal reflex are still present. What stage of anesthesia is this patient in? A. Stage 1 B. Stage 2 C. Stage 3 Plane 1 D. Stage 3 Plane 2

Correct answer: (C) Stage 3 Plane 1. Explanation: To answer this question, you need to know Guedel's Classification of the stages of anesthesia. Stage 1 is stage of analgesia/ disorientation. Stage 2 is the stage of excitement and delirium. Stage 3 is broken into 4 different planes. Plane 1 is when eyelid reflex is lost, swallowing reflex is lost and conjunctival reflex is lost. Plane 2 is when Laryngeal reflex is lost, corneal reflex is lost and secretion of tears increase. Plane 3 is when the light reflex is abolished and the pupils are dilated. Plane 4 is complete intercostal paralysis to diaphragmatic paralysis and Stage 4 is from respiratory stoppage to death.

Following total ankle arthroplasty, there is concern that there is inadequate peripheral cortical coverage of the posterior tibial component. The patient will be closely monitored post- operatively for: A. Wound dehiscence B. Joint stiffness C. Subsidence D. Incision skin necrosis

Correct answer: (C) Subsidence. Explanation: Subsidence is the migration of the tibial or talar components within the osseous substrate. The cause can be multifactorial, often related to osteolysis, inadequate sizing, or improper placement. If there is inadequate peripheral cortical bone coverage, the component is at higher risk for subsidence. (A) Although wound dehiscence is a possible complication of total ankle arthroplasty, it is not directly correlated with the cortical coverage of the implant components. (B) Joint stiffness is a common complication following total ankle arthroplasty, however there is no direct correlation with the coverage of cortical bone. (D) Delicate tissue handling is crucial during total ankle arthroplasty to prevent skin necrosis. Although this is a potential complication, it is not directly related to the cortical coverage of the components.

How is a gastrocnemius equinus compensated for in the foot? A. Locking of the midtarsal joint B. Dorsiflexion of the first metatarsophalangeal joint C. Subtalar joint pronation D. Subtalar joint supination

Correct answer: (C) Subtalar joint pronation. Explanation: Gastrocnemius equinus deformity occurs when there is a lack of dorsiflexion at ankle joint secondary to a tight or short gastrocnemius muscle. 5-10 degrees of ankle dorsiflexion is required during midstance phase of gait in order for the heel to reach the ground to allow for the tibia to pass over the foot. When this cannot be done, the subtalar joint responds by pronating. Pronation about the subtalar joint unlocks the midtarsal joint. Unlocking of the midtarsal joint, specifically the oblique axis, will allow the forefoot to dorsiflex relative to the rearfoot such that 5-10 degrees of foot-to-leg motion may occur. There are numerous other ways in which the extremity can compensate for the varying types of equinus deformities such as knee flexion, abductory twist, genu recurvatum, early heel lift.

What is the name of the clinical sign that is represented below? A. Gauthier's sign B. Mulder's sign C. Sullivan's sign D. Homan's sign

Correct answer: (C) Sullivan's sign. Explanation: Pictured above is Sullivan's sign. Sullivan's sign is a divergence of the toes created by the neuroma physically spreading the toes apart. It is indicative of an intermetatarsal neuroma. A Homan's sign is a clinical sign/test for deep vein thrombosis. A mulder's sign, also called a Mulder's click, is a palpable and sometimes audible click that occurs when the intermetatarsal space is compressed in a dorsal/plantar manner with simultaneous side-to-side compression of the metatarsal heads. A positive Gauthier's test/sign is when complete dorsiflexion of the metatarsophalangeal and interphalangeal joints causes pain to the affected interspace.

Which nerve about the foot and ankle has the most variable distribution pattern? A. Tibial B. Sural C. Superficial peroneal D. Deep peroneal

Correct answer: (C) Superficial peroneal. Explanation: The superficial peroneal nerve has the most variable distribution as it courses distal through the leg, ankle and foot. The superficial peroneal nerve provides innervation to the muscles of the lateral compartment of the leg and then pierces the fascia between the peroneal brevis and extensor digitorum longus. One of the biggest variables is where the superficial peroneal nerve pierces the fascia. Significant variation has been found regarding this point. From this point it descends down the leg and splits into the medial and intermediate dorsal cutaneous nerve. The path that these branches take is quite varied. On occasion the superficial nerve and its branches can be palpated when the foot is placed in plantarflexion and inversion.

Your patient is about to have a dorsoplantar calcaneal axial projection study for subtalar joint and ankle pain. Where should the patient wear the lead apron? No lead apron needed the most sensitive organs are facing anteriorly. With the tubehead facing at 25 degrees angulation from the vertical directed anteriorly the patient will not need protection, the radiation will be insignificant. The lead apron should be reversed so that it is placed over the patient's back. The lead apron should be on the front of the patient.

Correct answer: (C) The lead apron should be reversed so that it is placed over the patient's back. Explanation: The lead apron should be reversed so that it is placed over the patient's back. The patient stands on the cassette, the heel is placed so that the its posterior aspect is about 1 and 1⁄2 inch away from the edge of the cassette closest to the x- ray tube. The patient can be instructed to flex the knees slightly to a "ski jump" configuration so that the foot is dorsiflexed at the ankle joint.

Which of the following statement about the clinical picture below is correct? A. A long arm scarf osteotomy is illustrated above. B. Good correction of the hallux abductus angle is shown. C. The phalangeal screw is too long. D. Inadequate correction of the intermetatarsal angle is shown.

Correct answer: (C) The phalangeal screw is too long. Explanation: Pictured above is a short arm scarf osteotomy with an akin procedure. The screw in the Phalanx is clearly too long. A variable pitch headless screw was used here and is not adequately sunk in the proximal cortex and yet still has 3 threads exiting distally. If the screw head was inserted properly it would protrude even more distally. Also of note is that the hallux is positioned in varus so the HA angle is not well corrected. Based on the great correction of the intermetatarsal angle the akin procedure was likely not necessary and in this case caused an overcorrection.

Refer to the fixation involved in the surgical correction pictured below. Which is a true statement regarding the Evans procedure pictured below? A. The capital fragment is displaced dorsally. B. There is a delayed union of the graft site. C. The staple with 2 screws is providing compression fixation and stability. D. The staple is providing stability only.

Correct answer: (C) The staple with 2 screws is providing compression fixation and stability. Explanation: Pictured above is an excellent example of a flatfoot correction. Two fully threaded non-cannulated screws were used in the fixation of the first metatarsal cuneiform arthrodesis. They were inserted using a lag technique therefore the proximal cortex must be over-drilled. A compression staple was utilized on the Evans procedure and is stabilizing the bone graft and providing excellent compression. The method in which this staple provides compression is that once it is inserted an instrument is utilized to open up the diamond which draws the legs of the staple closer together providing compression. The anterior calcaneus is well aligned.

What articulations are combined to form the midtarsal joint? A. The talonavicular joint and the calcaneonavicular joint B. The calcaneonavicular joint and the calcaneocuboid joint C. The talonavicular joint and the calcaneocuboid joint D. The talonavicular joint only

Correct answer: (C) The talonavicular joint and the calcaneocuboid joint. Explanation: The midtarsal joint (transverse tarsal joint, Chopart's joint) consists of the combined articulations of the talonavicular and calcaneocuboid joints. It represents the functional articulation between the hindfoot (talus and calcaneus) and midfoot (navicular and cuboid). These articulations have been described anatomically as "plane" or "gliding" joints. The osseous movements of the midtarsal joint components when the subtalar joint is pronating and supinating have been described by Kapanji.

All the following statements are true regarding split thickness skin grafts except: Split thickness skin grafts have a better chance of survival under conditions of vascular compromise as compared to full thickness skin grafts. The most common thickness of a split thickness skin graft is between 0.012 and 0.018 inches. There is less graft contraction compared to full thickness skin grafts. An immediate post-operative bolster dressing is left undisturbed for 5 to 7 days as this is the time required for inosculation by vascular buds.

Correct answer: (C) There is less graft contraction compared to full thickness skin grafts. Explanation: Split thickness skin grafts contract much more than full thickness skin grafts. However, split thickness skin grafts have a far better survival rate even in the vascular compromised patients due to the thinness of the tissue as opposed to full thickness skin grafts. It is factual that the most common thickness of a split thickness skin graft is between 0.012 and 0.018 inches. The vascularization of the split thickness skin graft is by inosculation via vascular budding into the graft. This occurs 5 days after graft application.

A fifth ray resection can result in an adductovarus foot deformity due to loss of the peroneus brevis. An appropriate tendon transfer to prevent this complication is the: A. EDL to the midfoot B. Tibialis anterior and FDL to the medial column C. Tibialis posterior tendon to the third cuneiform D. Peroneus brevis to peroneus longus transfer

Correct answer: (C) Tibialis posterior tendon to the third cuneiform. Explanation: The antagonist to the peroneus brevis is the tibialis posterior. Loss of the brevis results in an overpowering by the tibialis posterior. To prevent a post-operative flatfoot, the FDL can be transferred to the navicular, to replace the absent tibialis posterior. (A) Transfer of the EDL to the midfoot is the typical Hibbs procedure, commonly used in CMT disease, or with flexible forefoot equinus with flexible hammertoes. (B) Transfer of the tibialis anterior and FDL to the medial column is a flatfoot procedure, to address medial column sag. (D) The brevis to longus transfer is used for peritalar subluxation with peroneal spastic flatfoot or first ray elevatus caused by naviculocuneiform or cuneiform-first metatarsal sag.

Refer to the radiograph below. What is the purpose of the two olive wires in the metatarsals? A. to translate the forefoot B. to compress the metatarsals together C. to prevent the forefoot from sliding on the wires D. to fixate osteopenic bone

Correct answer: (C) To prevent the forefoot from sliding on the wires. Explanation: The two wire in the forefoot are called olive wires. The medial olive is abutting against the first metatarsal and the lateral olive is abutting against the fifth metatarsal. The purpose of the "opposing" olive wires are to stabilize the forefoot and to prevent it from sliding back and forth on the wires when the patient ambulates. Olive wires can be used to translate bone segments in gradual deformity correction and in comminuted fractures such as pilon fractures. Wires both smooth and with olives work well in osteopenic bone. There are even washers that can be used concomitantly with the olive wires to further increase the surface area of contact. Lastly, there is no mechanism pictured above to provided compression of the metatarsals.

Which amputation procedure for diabetic foot ulcers produces the highest postoperative ambulatory levels and longest durability? A. Chopart amputation B. Symes amputation C. Transmetatarsal amputation D. Partial calcanectomy E. Total calcanectomy

Correct answer: (C) Transmetatarsal amputation. Explanation: Of the pedal amputations for diabetic foot ulcers, the transmetatarsal amputation provides the highest postoperative ambulatory level and the longest durability. (A) The Chopart amputation provides favorable postoperative ambulatory level, but with a shorter period of disability, with a risk of subsequent proximal ipsilateral amputation within 3 years. (B) The Symes amputation has much shorter durability than tha transmetatarsal amputation. (D) The partial calcanectomy offers a much poorer postoperative ambulatory level. (E) The total calcanectomy has much higher morbidity and mortality rates than the transmetatarsal amputation.

What is the primary indication for the Evans calcaneal osteotomy with insertion of a bone graft? A. congenital vertical talus B. residual talipes equinovarus C. transverse plane flatfoot deformity D. tarsal coalition

Correct answer: (C) Transverse plane flatfoot deformity. Explanation: An Evans calcaneal osteotomy is indicated for a transverse plane flatfoot deformity with the intention to lengthen a short lateral column. It is performed 1-1.5cm proximal from the calcaneal cuboid joint with an insertion of a bone graft 0.5-1cm long. In both congenital vertical talus and tarsal coalition you have hindfoot valgus, but not due to a short lateral column. In addition both of these deformities are rigid which would not be indicated for an Evans calcaneal osteotomy. In a residual talipes equinovarus you have a hindfoot varus for which a lateral column lengthening osteotomy is not indicated.

Ulcer excision with primary closure is a fundamental adjunctive procedure done concomitantly with many elective and nonelective surgeries on the diabetic foot. Which of the following statements regarding ulcer excision is false? A. It facilitates faster healing therefore decreases risk of infection. B. It decreases the risk of re-ulceration as compared to healing by secondary intention. C. Ulcer excision and primary closure should be avoided on the plantar surface of the foot. D. Wounds that are excised and closed with primary closure require less blood supply than those healed by secondary intention.

Correct answer: (C) Ulcer excision and primary closure should be avoided on the plantar surface of the foot. Explanation: Ulcer excision with primary closure is the preferred technique regardless of location on the foot. It is equally as effective on the plantar skin as anywhere else of the foot. This validates that answer (C) is the false statement. Ulcer excision with primary closure do indeed heal faster than those that heal by secondary intent. Additionally, wounds that heal by secondary intent do have a higher rate of re-ulceration because they are inelastic and less durable. Lastly, wounds that heal by secondary intention do require a greater amount of blood supply justifying the last statement.

Which of the following is a major error and is not recommended while using the Ponseti Method for clubfoot casting? Supinating or inverting the foot. Reducing the equinus deformity last. Using below the knee casts during correction. Using shoes or molded orthotics attached to bar in external rotation after complete correction.

Correct answer: (C) Using below the knee casts during correction. Explanation: Using below knee casts should always be avoided. You should use toe to groin casts for this. You should never allow the family to remove the cast the night before coming back to see you. (A) Supinating and inverting the foot is important while doing the Ponseti method for clubfoot correction. Pronating the foot should be avoided as it will only make the deformity worse. (B) The equinus deformity should always be the last component corrected. If this is corrected prior to adduction and varus components, you are likely to get a rocker bottom foot. (D) You must always use shoes or molded orthotics attached to bar in external rotation after correction. This should be done around the clock for the first 3-4 months and then at night for the next 2-4 years to avoid re-contracture.

As per the guidelines established by Gustilo, what is the antibiotic recommendation for a grade I fracture in an individual who is allergic to penicllin? A. Cefazolin B. Clindamycin C. Vancomycin D. Gentamicin

Correct answer: (C) Vancomycin. Explanation: In the penicillin allergic patient the recommended antibiotic therapy for a Gustilo grade I fracture would be intravenous vancomycin. In the patient who is not allergic to penicillin, the recommended antibiotic therapy is Cefazolin. (A) Cefazolin would be contraindicated in a patient with a penicillin allergy. In the patient no allergic to penicillin the recommended antibiotic therapy is Cefazolin. (B) Clindamycin is not the recommended antibiotic for a Gustilo grade I fracture. (D) Gentamicin is not the recommended antibiotic for a Gustilo grade I fracture.

The best surgical procedure for the CP patient with a HAV deformity is the: A. McBride B. Austin C. Lapidus D. 1st MTPJ fusion

Correct answer: (D) 1st MTPJ fusion. Explanation: The majority of CP patients have equinus, with poor toe clearance during the swing phase of gait. There is a stiff-knee gait secondary to rectus femoris dysfunction; this leads to toe drag at the initial point of stance, where the medial hallux and first ray contact the supporting surface first. The only surgery that can withstand the deforming forces at the first MPJ is a 1st MTPJ fusion, also known as a McKeever fusion; all others are doomed to failure.

The images below are the radiographs of a 48 year old woman who has pain at the first metatarsophalangeal (MTP) joint with various types of shoe wear and prescribed inserts, which now limits her activities. Her first metatarsophalangeal joint is has painful range of motion with dorsiflexion limited to 5 degrees and plantarflexion limited to 15 degrees. The surgical correction that will best treat her pain should consist of: A. Cheilectomy B. Distal metatarsal osteotomy C. Lapidus procedure ( first tarsometatarsal arthrodesis) D. 1st metatarsophalangeal fusion

Correct answer: (D) 1st metatarsophalangeal fusion. Explanation: Given the patients failed conservative treatment, limited and painful range of motion, and arthritis of the first MTP joint as seen on radiographs, a fusion of the first MTP will most reliably treat this patient's symptoms. (A) A simple cheilectomy will not address the arthritis seen at the MTP joint and therefore will less reliably alleviate the above patient's symptoms. (B) A simple distal metatarsal osteotomy will not address the arthritis seen at the MTP joint and therefore will less reliably alleviate the above patient's symptoms. (C) Although a Lapdius procedure ( first tarsometatarsal arthrodesis) is an excellent option for correcting moderate to severe cases hallux valgus, this patient has an arthritic MTP joint that a Lapidus procedure will not address.

Depending on the extent of a talar neck fracture, a patient is at variable risk for development of talar avascular necrosis. The branches providing blood supply to the talus originate from which major branches? A. Anterior tibial artery, perforating peroneal artery B. Posterior tibial artery, perforating peroneal artery C. Anterior tibial artery, posterior tibial artery D. Anterior tibial artery, posterior tibial artery, perforating peroneal artery

Correct answer: (D) Anterior tibial artery, posterior tibial artery, perforating peroneal artery. Explanation: The talus is at risk of AVN depending on the extent of injury. The main arterial branches supplying the talus include the arterior tibial artery, the posterior tibial artery, and the perforating peroneal artery. From the perforating peroneal artery and anterior tibial artery branches the artery of sinus tarsi. From the posterior tibial artery are the deltoid branches and artery of tarsal canal. The arteries create anastomoses with the communicating branches. (A) (B) (C) Although there are variable reported degrees of AVN for Hawkins type I, II, III and IV, the more branches providing blood supply that are damaged in the injury will result in higher risk of AVN.

Failure of a cheilectomy to resolve pain in a hallux limitus deformity is associated with a Coughlin-Shurnas grade: A. 1 B. 2 C. 3 D. 4

Correct answer: (D) 4. Explanation: Grade 4 patients have pain at mid-range of passive motion; this will most likely result in failure of the cheilectomy procedure; the pain will persist. There will be joint space narrowing and cystic change at the MPJ. The sesamoids are cystic, irregular, and/or enlarged. There is probably less than 50% cartilage left; the general rule of thumb for joint preservation procedures. Thus, an implant or first MPJ fusion would be indicated. (A) Grade 1 patients have approximately 30-40 degrees of dorsiflexion at the first MPJ. There are minimal degenerative changes at the first MPJ; in fact, the dorsal osteophyte is the main finding, and most amenable to a cheilectomy procedure. First MPJ pain is elicited only at extremes of dorsiflexion or plantarflexion. (B) Grade 2 demonstrates 10-30 degrees of dorsiflexion. The first metatarsal head is flattened, with dorsal, medial, and lateral osteophytosis. First MPJ ROM pain is noted just before maximum dorsiflexion or plantarflexion. On radiograph, it would appear that degenerative disease involves no more than the dorsal one quarter of the metahead, making the joint amenable to an isolated cheilectomy procedure. (C) Grade 3 has less than ten degrees of dorsiflexion, like Grade 4. There is ROM pain, but no mid-range pain, which differentiates it from Grade 4. There are cystic changes, and more than one quarter of the dorsal joint space is involved on a lateral radiograph. There is sesamoid involvement. The use of cheilectomy as a sole procedure is controversial with Grade 3. Some advocate it; others recommend a phalangeal osteotomy in conjunction. Intra-operative inspection would dictate which road to follow.

How many centers of motion does the first metatarsophalangeal joint have? A. 1 B. 2 C. 3 D. 4

Correct answer: (D) 4. Explanation: The 1st MPJ has four centers of motion forming an arc. Rolling motion occurs around the first center of motion corresponding to the hinge motion of the joint. Sliding motion occurs around the second and third centers of motion corresponding to arthrodial motion of joint. The fourth center of motion produces a compressive force at the end of range of motion. The 1st MPJ has two distinct axes of motion. A ginglymoid or hinge motion occurs for the first 20 degrees of dorsiflexion. Arthrodial motion begins at 20 degrees. For normal arthrodial motion to occur at the 1st MPJ the first ray needs to plantarflex with heel-lift, subtalar joint supination, normal sesamoid function, and a second metatarsal longer than the first.

What is the correct osteotomy angle in the frontal plane in the Wilson bunion procedure? A. 25° B. 30° C. 35° D. 45°

Correct answer: (D) 45°. Explanation: The osteotomy in the Wilson bunion procedure is angled 45° in the frontal plane and 90° in the sagittal plane. Due to this geometry, the lateral displacement needed to correct the valgus deformity necessitates shortening of the first metatarsal. This osteotomy also does not provide any inherent stability. (A) A 25° angled osteotomy in the frontal plane is not the angle described in the Wilson bunion procedure. (B) A 30° angled osteotomy in the frontal plane is not the angle described in the Wilson bunion procedure. (C) A 35° angled osteotomy in the frontal plane is not the angle described in the Wilson bunion procedure.

Which of the following is indicated when performing an AP projection of the foot? A. The central ray (CR) is directed at the head of the third metatarsal. B. The CR is directed 10 degrees caudad. C. The dorsal aspect of the foot should rest on the image receptor (IR). D. A higher arch requires an angle of approximately 15 degrees to be perpendicular to the metatarsals.

Correct answer: (D) A higher arch requires an angle of approximately 15 degrees to be perpendicular to the metatarsals. Explanation: A foot series includes an AP, oblique, and lateral radiograph. The AP or frontal projection is a dorsiplantar projection that is performed with the central ray (CR) parallel to the medial and intermediate cuneiform and medial Lisfranc joint space. The CR is angled posteriorly and perpendicular to the metatarsals. The CR should be directed to the base of the third metatarsal. The plantar surface should be flat and resting on the image receptor (IR). A high arch requires a greater angle of 15 degrees and a low arch requires an angle near 5 degrees to be perpendicular to the metatarsals.

Which of the following conditions will not exhibit acroosteolysis? A. Scleroderma B. Hyperparathyroidism C. Psoriatic Arthritis D. Acromegaly

Correct answer: (D) Acromegaly. Explanation: Resorption of the distal phalangeal tufts is known as acroosteolysis. Acro- meaning tip, osteo- meaning bone, and lysis- meaning destruction/loss. The phalanges may take on the appearance of a cone as the tip is whittled down and resorbed. In advanced cases the phalanges may be completely destroyed. Of the conditions listed above, the only one that does not manifest tuftal resorption is acromegaly making (D) the correct answer. Acromegaly occurs as a result of excess growth hormone in adults. Radiographically this condition results in soft tissue and osseous structures growing larger in size. Increased width of the heel pad, enlarged metatarsal heads and shafts, and spur formation is commonly seen on plain film. The cartilage often thickens which results in widening in of joints spaces. Additionally, one would see growth and widening of the distal phalangeal tufts, which is the opposite of acroosteolysis.

Where is the os sustentaculum tali located? A. In close apposition of the calcaneal anterior process along its superomedial surface B. Along the superior aspect of the talonavicular joint C. In the peroneus longus tendon D. Along the posterior aspect of the sustentaculum tali

Correct answer: (D) Along the posterior aspect of the sustentaculum tali. Explanation: This is a rare ossicle, also known as the os sustentaculi, found along the posterior aspect of the sustentaculum tali.

What term is defined as the angle formed by the longitudingal axis of the foot and the line of progression? A. Stride length B. Step C. Base of Gait D. Angle of Gait

Correct answer: (D) Angle of Gait. Explanation: The angle of gait is measured by a line bisecting the center of the heel and the first interspace with the sagittal plane. The average angle of gait is 7 degrees toe out per side, or an average of 12-15 degrees total. Torsional or positional changes in the hip, femur, and tibia have major influence on this position. The angle of gait also varies during child development as the bones and soft tissues become more developed.

A 47-year-old male postal worker presents for a surgical consultation for hallux limitus. His X- ray is presented below. On physical exam pain is noted through the ROM of the first metatarsophalangeal joint with crepitation. What would be the preferred procedure listed below? A. decompression osteotomy B. cheilectomy C. implant arthroplasty D. arthrodesis

Correct answer: (D) Arthrodesis. Explanation: The foundation of this question rests on the knowledge of procedure selection for hallux limits as it relates to pathology and patient factors. The correct answer for our patient above is a first metatarsophalangeal joint arthrodesis. There are multiple clues above that make this the only logical selection. First is the deformity present. With significant joint space narrowing, presence of an osteophyte and pain through the entire range of motion is indicative of significant cartilage loss (> 50%). Additionally at 47 and an active individual the arthrodesis is the most definitive, functional procedure. Implant arthroplasty would provide a poor functional result and failure of the implant would occur too soon to make this a viable choice for this individual. A decompression osteotomy would be indicated for less severe disease as would cheilectomy.

A patient develops a fever 18 hours following elective foot surgery in which the patient received general anesthesia. What is the mostly likely cause of the fever? A. Wound infection B. Drug reaction C. DVT D. Atelectasis

Correct answer: (D) Atelectasis. Explanation: A fever developing within the first 24 hours of surgery is often caused by atelectasis. It may produce a self-limiting low-grade fever. Also during this time period, the body may mount a fever as a metabolic response to injury. A common mnemonic for remembering the common causes of post-operative fever is "Wind, Water, Wound, Walk, Wonder drugs." "Wind" represents atelectasis and other respiratory causes of fever that typically occur within the first 24 hours. "Water" represents urinary tract infections in the case of patient being catherized during the procedure and typically occurs after post-op day (POD) three. "Wound" represents an infection from the surgical wound and typically occurs after POD5, but can be anytime. "Walk" represents DVT, PE, or thrombophlebitis and typically occurs between POD7 and POD10. And "Wonder drugs" represents drug-induced fever that can occur at anytime.

Recurrence of congenital vertical talus after surgery can result from all of the following except: A. Residual foot abduction B. Incomplete talonavicular reduction C. Insufficient posterior ankle release D. Avascular necrosis of talus E. Associated neurologic conditions

Correct answer: (D) Avascular necrosis of talus. Explanation: Although most cases of perioperative recurrence of congenital vertical talus are due to undercorrection including incomplete talonavicular reduction and insufficient posterior ankle release, associated neurologic conditions and residual foot abduction can also lead to recurrence. Avascular necrosis of the talus is a rare early complication of congenital vertical talus surgery, and is usually seen in two-stage releases. Although it does not lead to recurrence, it can result in chronic ankle pain, stiffness, and weakness.

According to the law of Bergonie and Tribondeau, what is the correct order to which the following tissues are sensitive to radiation? A. Central nervous system, bone marrow, GI tract B. GI tract, central nervous system, bone marrow C. Bone marrow, central nervous system, GI tract D. Bone marrow, GI tract, central nervous system

Correct answer: (D) Bone marrow, GI tract, central nervous system. Explanation: Based on the law of Bergonie and Tribondeau: Bone marrow, with lymphocytes is most sensitive, followed by the GI tract, and then the central nervous system is the most resistant to radiation. Hematopoietic death occurs between 35-1,000 rems, death in 10-21 days, GI death 1,000-10,000 rems and death will occur in 3-10 days. Finally, the central nervous radiation greater than 10, 000 rems will lead to death in 72 hours due to cardiovascular collapse.

During the performance of an Austin osteotomy, the capital fragment falls on the floor. The best treatment protocol is: A. Mechanical scrub, autoclaving the fragment, and reimplantation B. Mechanical scrub, immersion in saline solution, and reimplantation C. Bulb lavage irrigation, immersion in chlorhexidine solution, and reimplantation D. Bulb lavage irrigation, immersion in povidone-iodine solution, and reimplantation

Correct answer: (D) Bulb lavage irrigation, immersion in povidone-iodine solution, and reimplantation. Explanation: Two different journal articles were reviewed, with some differing opinions. What must be considered is decrease in contamination vs. cell viability. Is it best to attempt to completely eradicate the bacteria from the bone with aggressive scrubbing, which will decrease the number of viable regenerative cells, or to be more gentle in an attempt to preserve as many regenerative cells as possible? Irrigation via bulb syringe was best to maintain cell viability. The fragment was then immersed in 10% povidone-iodine for five minutes followed by a saline rinse. (A) Easily the worst choice of the four. The aggressive scrubbing kills many cells, and what cells you have left will be destroyed by the autoclaving. (B) Although some advocate for aggressive mechanical scrubbing, using only saline to provide decontamination is ineffective; the saline by itself adds nothing to the process. (C) 2% or 4% chlorhexidine gluconate is an effective decontaminant, but left few to no viable cells; close to the kill rate of autoclaving.

An 11 year old boy presents complaining of bilateral heel pain. The pain has been going for about 5 days. He reports that the pain is much worse in his left heel than in his right heel. He as well as his parents deny any known trauma. His medical history is unremarkable. He reports that about a week ago he started soccer practice. Physical exam reveals marked tenderness to the left heel upon side to side compression. The same stimulus yields pain on the right heel as well, just not as severe. A radiograph is provided below. What is the diagnosis? A. Plantar fasciitis B. Insertional achilles tendonitis C. Calcaneal stress fracture D. Calcaneal apophysitis

Correct answer: (D) Calcaneal apophysitis. Explanation: Calcaneal apophysitis is a very common complaint of heel pain particularly in young males between 10-13 years of age. Young girls can get it as well, but not as often. The condition frequently presents bilaterally but often asymmetric relative to symptom level. There is a frequent association with tight Achilles tendons and a recent growth spurt. Calcaneal tenderness on compression is virtually pathognomonic. A clear history of a recent increase in training or competition is often elicited. Plain radiographs may be normal, but multipartite apophyses with or without sclerosis are common.

Which of the following deformities should be corrected first when using the Ponseti method of casting for clubfoot deformity? A. Equinus B. Varus C. Adductus D. Cavus

Correct answer: (D) Cavus. Explanation: Using the acronym "CAVE" the cavus deformity is corrected first by dorsiflexing the first ray is first to stretch the plantar fascia and unlock the talonavicular joint. Forefoot adductus, hindfoot varus, and equinus are then corrected, in that order.

Nuclear medicine scans are obtained on a patient with an infected ulceration with questionable osteomyelitis. Both a technetium and a gallium scan were obtained. The technetium scan was negative, but the gallium scan was positive. What is the diagnosis? A. acute osteomyelitis B. chronic osteomyelitis C. infectious periostitis D. cellulitis

Correct answer: (D) Cellulitis. Explanation: When technetium is positive and gallium is negative, chronic osteomyelitis is present. When both technetium and gallium are positive, the diagnosis is either acute osteomyelitis or a septic arthritis. Lastly, when a technetium scan is negative an a gallium scan is positive, the diagnosis is cellulitis. Technetium is a bone-imaging radionuclide while gallium is an inflammatory-imaging radionuclide. Gallium binds to white blood cells, plasma proteins, transferrin, ferritin, lactoferrin and siderophores and travels to areas of inflammation. Technetium binds to hydroxyapatite crystals and travels to area of high bone turnover.

The strength of a screw is determined by all the following factors except: A. Pitch B. Thread number C. Thread shape D. Cost

Correct answer: (D) Cost. Explanation: The pitch can influence the number of threads that engage the bone thus providing compression and strength. Cost is not a significant factor in determining strength because some of the cheaper screws could provide more compression and thus strength, compared to more expensive, newer screws.

What is the best protection against occupational radiation exposure? A. Lead apron B. Collimator C. Filter D. Distance

Correct answer: (D) Distance. Explanation: Distance is the most effective protection against ionizing radiation. The dose of radiation is proportionate to the inverse square of the distance. The dose of radiation decreases exponentially with distance. The intensity of the scattered beam is approximately 1/1000th the intensity of the primary beam at a distance of 1 meter and at 90 degrees. This law should be used during fluoroscopy when close patient contact is not required and should be used during mobile radiography. The cardinal principles of radiation protection are time, distance, and shielding.

What x-linked condition is associated with a pseduohypertrophy of the calves along with difficulty walking or standing by age 10? A. Becker Muscular Dystrophy B. Blount Disease C. Emery-Dreifuss Muscular Dystrophy D. Duschenne's Muscular Dystrophy

Correct answer: (D) Duschenne's Muscular Dystrophy. Explanation: Duschenne's muscular dystrophy is an x-linked, hereditary disorder associated with pseudohypertrophy of the calves, along with the inability to walk by the ages of 10-11. Being that it x-linked, it only affects young boys, with the age of symptom onset being near walking age. In most cases, the ability to walk to either delayed or never occurs. Some of the first signs of the disease is muscle weakness, usually up to age 5. Boys with Duschenne's muscular dystrophy often have an abnormal gait, such as waddling or toe walking, with hyperloardosis noted. The hyperlordosis is a compensatory mechanism for the progressive muscle weakness of the girdle muscles, more noticeable in the legs than the arms. The patient ages and the disease progresses, the child will likely be unable to stand from a seated position on the floor without the use of their arms to climb up their legs, also known as a Gowers Sign. By the age of 10-11, the ability to walk or stand is lost, confining the child to a wheelchair. By mid-teens, respiratory and cardiac muscle weakness often leads to the death of the child. (A) Becker muscular dystrophy (BMD) is an x-linked condition associated with pseuodhypertrophy of the calves but has a much slower progression allowing the child to walk even after puberty. Life expectancy for someone with BMD is between 40 and 60 years old and largely depends on the degree of respiratory insufficiency. Like Duschenne's, the child will often have a waddling gait and lordosis, but will complain of cramping after any physical exertion. Muscle enzymes will be elevated on work-up and an abnormal dystrophin can be differentiated from the Duschenne type. (B) Bount disease is not an x-linked condition is associated with a pseduohypertrophy of the calves along with difficulty walking or standing by age 10. Rather, it is the necrosis of the proximal medial tibial epiphysis, often leading to a tibial varum deformity of the child. (C) Emery-Dreifuss Muscular Dystrophy (EDMD) is an x-linked recessive muscular dystrophy that affects older children, teens, and even adults. It too presents with walking difficulties but may also present with cardiac arrhythmias. EDMD progresses slowly with slow stiffening of the spine resulting in fixed deformities of the extremities. Muscle biopsy reveals dystrophic muscle fibers mixed with atrophied muscle.

Which of the following conditions does not cause compensatory subtalar joint pronation? A. Rearfoot Varus B. Rearfoot Valgus C. Equinus D. Forefoot Valgus

Correct answer: (D) Forefoot Valgus. Explanation: Forefoot valgus is compensated through midtarsal joint supination and if available midtarsal joint supination is utilized, then subtalar joint supination. With forefoot valgus, the head of the first metatarsal strikes the ground before the head of the fifth metatarsal. This causes a ground reactive force to push the first metatarsal up thereby supinating the midtarsal joint. In the rearfoot varus foot the subtalar joint will pronate in order to equalize the ground reactive forces across the calcaneus. In the rearfoot valgus type, the body's center of mass is placed more medial to the subtalar joint axis thereby increasing the pronator moment across the subtalar joint during the gait cycle. Lastly, equinus causes potentially pathologic pronation. Equinus is the ankle's inability to achieve 10 degrees of dorsiflexion while the subtalar joint is in neutral. This causes pronation of the foot around the oblique midtarsal joint. This will cause a dorsiflexion and abduction of the forefoot relative to the rearfoot. This action will lead to plantarflexion of the reactive to the ankle joint secondary to ground reactive forces. This plantarflexion of rearfoot will then decrease the tension in the achilles tendon and allow the foot the change to obtain 10 degrees of dorsiflexion during the midstance period.

Which of the following is not a benefit of preemptive analgesia? A. Decreased post operative opioid use B. Decreased incidence of chronic pain syndrome C. Early mobilization D. Increased incidence of ileus

Correct answer: (D) Increased incidence of ileus. Explanation: Numerous studies have shown that the use of preemptive analgesia, that is analgesia prior to the skin incision, has indeed been shown to significantly reduce the use and need for postoperative opioids. Since the use of preemptive analgesia precludes the establishment of central sensitization less opioid use is required which additionally deceases the incidence of chronic pain syndrome. Due to far superior pain management, more aggressive rehabilitation and early mobilization is possible. Conversely, as sizable amounts of opioids can precipitate ileum the use of preemptive analgesia decreases the incidence of ileus.

The Isherwood projections consists of three positions that when assessed in their entirety give a complete view of the subtalar joint. The medial oblique axial view allows visualization of the middle and posterior facets. The lateral oblique axial view allows visualization of the posterior facet in profile. The third view allows excellent visualization of the anterior facet of the subtalar joint. What is this third view? A. Long leg calcaneal axial B. Hindfoot alignment view C. Lateral view D. Oblique plantardorsal view

Correct answer: (D) Oblique plantardorsal view. Explanation: The Isherwood projection consists of three different positions with the purpose of complete, circumferential visualization of the subtalar joint. The oblique plantar dorsal view is used to visualized the anterior facet. The medial oblique axial view allows visualization of the middle and posterior facets. The lateral oblique axial view allows visualization of the posterior facet in profile.

According to the kVp 15% Rule, the gray scale of contrast can be lengthened (more shades of gray or less radiographic density) by: A. Increasing mAs by 2 B. Decreasing the kVp by 15% C. Increasing mAs by 1⁄2 D. Increasing kVp by 15%

Correct answer: (D) Increasing kVp by 15%. Explanation: The 15% Rule deals with image contrast or the variation of grays within a radiographic image which in turn affects the detail of the radiograph. By adjusting the kilovoltage, which is one of the main factors in determining radiographic density, the quantity of the x-ray photons produced and the quality of the x-ray beam are affected, and ultimately the appearance of the x-ray. If the kVp is increased by 15%, the density of the image will double (increased kVp=increase density), the mAs states that when the kVp is increased by 15%, the mAs will need to be decreased by 1⁄2 in order to lengthen the scale of contrast. If higher contrast is desired (shortening the scale of contrast), the kVp will decrease by 15% and the mAs will need to increase by 2. Increase kVp=decreased contrast (more grays/lengthened gray scale/decreased density) Increase mAs=increased density (law of reciprocity)

Which structure is most commonly injured when gaining access to the anterolateral portal for ankle arthroscopy? A. Extensor hallucis longus tendon B. Anterior tibial artery C. Sural nerve D. Intermediate dorsal cutaneous nerve

Correct answer: (D) Intermediate dorsal cutaneous nerve. Explanation: The intermediate dorsal cutaneous nerve is in close proximity to the anterolateral portal for ankle arthroscopy. (A) The anterolateral portal is immediately lateral to the extensor digitorum longus. The extensor hallucis longus is further medial and less likely to be injured for this portal. (B) The neurovascular bundle is just lateral to tibialis anterior and extensor hallucis longus. However, it is still further medial than the extensor digitorum longus and at lower risk of injury. (C) The sural nerve is on the posterior aspect of the ankle, just lateral to the Achilles tendon. It is not at high risk from access to the anterolateral portal.

Nonunions of hallucal condylar fractures of the interphalangeal joint are rarely symptomatic. However, on occasion they become symptomatic. What is an acceptable initial treatment for one of these nonunions? A. Excision of fracture fragment B. Incorporation of a bone stimulator C. Orthotic with a reverse morton's extension D. Intra-articular steroid injection and immobilization

Correct answer: (D) Intra-articular steroid injection and immobilization. Explanation: Intra-articular steroid injection and immobilization of the hallux often improves the symptomatic nonunion of a hallucal condylar fracture of the hallux. (A) Excision of the fracture fragment is an acceptable treatment for he symptomatic nonunion of a hallucal condylar fracture of the hallux, however, only in recalcitrant cases were all other modalities have failed. (B) The incorporation of a bone stimulator is rarely a modality used in the treatment of hallucal condylar fracture of the hallux. (C) An orthotic with a reverse morton's extension, although might be helpful in the treatment of a hallucal condylar fracture of the hallux it is not the first line of therapy.

An 11 year female runner presents to the office complaining of pain to the lateral aspect of her right foot for the past week. She notices it mostly when she runs. She and her parents deny any trauma. On physical examination she has tenderness over the lateral aspect of the base of her left fifth metatarsal. No edema noted. A radiograph is provided below. What is the most likely diagnosis? A. Avulsion fracture of the fifth metatarsal B. Peroneus brevis tenosynovitis C. Midfoot sprain D. Iselin's disease

Correct answer: (D) Iselin's disease. Explanation: Iselin's disease is an apophysitis associated with the growth center at the base of the fifth metatarsal. Iselin's disease is a relatively common overuse injury in the pediatric athlete involved running sports. It presents with point tenderness with or without swelling. Some think that the presence of metatarsus adducts might predispose to this condition. Rest and discontinuation of the exacerbating activity, foot strapping, ice, and immobilization are all mainstays of treatment. Radiographic findings include irregular apophyseal density and shape. Avulsion fracture of the fifth metatarsal (choice A) would show a fracture most likely perpendicular to the fifth metatarsal along with radiolucency located proximally to the metaphyseal-diaphyseal junction. Peroneus brevis tenosynovitis (choice B) would be non- diagnostic with plain films, however may show a periosteal reaction in adjacent bone. A midfoot sprain (choice C) is challenging to diagnose on plain films. It is important to note that fractures in the midfoot area are frequently misdiagnosed as midfoot sprains due to the clustering of small bones in this area.

A five year old boy presents with his parents to the office. The boy complains of pain in his feet when he walks. He points to his arch when asked where the pain was located. His parents confirm this is a consistent complaint and deny any trauma. This pain has been present for about two weeks. On exam there is tenderness on palpation over the medial aspect of the proximal arch with mild edema. The range of motion of the foot is normal and full. Gait exam shows the boy walking on the lateral aspect of his foot and not fully loading the foot. What would be the most likely provisional diagnosis? A. Posterior tibial tendon dysfunction B. Plantar fasciitis C. Calcaneonavicular coalition D. Kohler's disease

Correct answer: (D) Kohler's disease. Explanation: The clinical scenario above is most likely Kohler's disease. Kohler's disease is an avascular necrosis of the navicular which occurs in boys before the age of six. They typically complain of paining the medial arch, sometime have mild edema and have an antalgic gait. Another common finding is that the patient will tend to walk onto lateral aspect their foot due to the pain. So the keys to this question are the patient's age and the location of pain. (A) Posterior tibial tendon dysfunction is not a problem of the pediatric population. (B) Plantar fasciitis is not common in children of this age and the location is not correct. (C) The diagnosis of a calcaneonavicular coalition is not likely due to location and the unrestricted range of motion.

A 49 year old poorly controlled diabetic presents for follow up from a recent hallux amputation. His foot is swollen, red and malodorous. A radiograph was obtained and is provided below. What is the most appropriate treatment listed below? A. Aggressive empiric broad-spectrum antibiotics with culture and sensitivity B. Admit to hospital for intravenous antibiotics C. Stat referral to infectious disease D. Prompt incision and drainage in the operating room

Correct answer: (D) Prompt incision and drainage in the operating room. Explanation: The key to this question is the recognition of gas in the plantar soft tissues. This is gas gangrene and is a surgical emergency to preserve the limb and life. The gas is usually a byproduct of anaerobic bacteria (classically Clostridia perfrigens) that has traveled along the fascial planes. The wound needs to be opened up and debrided aggressively and immediately. Hyperbaric oxygen should be considered postoperatively to help control the anaerobic infection.

In fixating of long bone fractures, what provides interfragmentary compression? A. Neutralization plate B. LCD- plate C. DCP plate D. Lag screw

Correct answer: (D) Lag screw. Explanation: The lag screw involves a unique method of placement that results in approximation and subsequent compression of two osseous segments. Interfragmentary compression generates greater amounts of friction and thus reduces or prevents movement between segments. Lack of motion at the osseous junction, not compression, is osteogenic. Successful application of the lag screw technique can be achieved with either a fully threaded screw or a partially threaded screw. The methods to achieve compression are slightly different with a fully threaded screw. The process involves placement of a screw so the threads engage only the far cortex of bone. Advancing the screw eventually results in abutment of the screw head against the cortex of the near fragment. Further advancement only generates compression as the far cortex is brought in approximation to the near segment. When a fully threaded screw is used as a lag screw, it is necessary to drill the near segment to a width equal to the outer screw diameter. The threads must not purchase the bone. The near segment is essentially a gliding hole for the screw, and thread purchase only occurs in the far segment. Partially threaded screws are designed as lag screws; they do not engage the far and near segments simultaneously. Should the width of the far osseous segment be less than the length of the thread pattern, threads will cross the fracture line or osteotomy, and segments will distract. Neutralization plate is a protective device because it controls bending, translational, torsional forces. Use of lag screws only without a neutralization plate for long bone diaphyseal fixation will lead to failure. Lag screws insertion provides compression between fracture elements and is referred to as interfragmentary compression.

Failure of these muscles will lead to clawing of the lesser digits, as these muscles produce plantarflexory stabilization of the MTPJ. A. Dorsal interosseous muscles B. Plantar interosseous muscles C. Plantar and dorsal interosseous D. Lumbricales, interossei muscles, EDL, EDB

Correct answer: (D) Lumbricales, interossei muscles, EDL, EDB. Explanation: Extension of the toes is the result of the combined action of the extensor digitorum longus, extensor digitorum brevis, interossei muscles, and lumbrical muscles, all of which converge to form a tendinofibroaponeurotic structure known as the extensor apparatus. The extensor hood apparatus is an extremely important and unique anatomical structure. At the level of the MTPJ, its medial and lateral fibers unite plantarly with the capsule, deep transverse metatarsal ligament, and the flexor tendon sheath, thereby functionally inserting into the MTPJ. Consequently, the pull of the EDL, EDB tendons creates significant dorsiflexory power of the proximal phalanx at the MTPJ and minimal dorsiflexory power at the PIPJ or DIPJ. As the MTPJ undergoes dorsiflexon, the passive tension that develops on the flexor tendons produces plantarflexion at the PIPJ and DIPJ due to limited extensor forces at these joints. The result is claw toe deformity.

Which of the following conditions is the exception regarding the appropriate use of a shave biopsy? A. Keratoacanthomas B. Intradermal nevus C. Basal cell carcinoma D. Melanoma

Correct answer: (D) Melanoma. Explanation: Shave biopsies are among the quickest and easiest biopsies to perform. It is a superficial method but has definite indications. It can be performed with a standard scape blade or a DermaBlade. The biggest limitation to a shave biopsy is depth. If the lesion is deeper than the superficial reticular dermis, a portion of the lesion may go unsampled. Skin conditions that are suitable for shave biopsy include keratoacanthosis, dermatofibroma, verruca vulgaris, molluscom contangiosum, cutaneous horn, acrochordon, seborrheic keratosis, actinic keratosis, fibrous papules, neurofibroma, intradermal nevus, squamous cell carcinoma, basal cell carcinoma.

A 52 year old female presents with a complaint of pain in the ball of her right foot. Physical exam reveals pain on palpation noted in both the second and third interspaces of the right foot. A diagnostic block of 1 cc 1% lidocaine without epinephrine's administered to the right third interspace. Examination after the diagnostic block it is noted that the pain to palpation persists from both the second and third interspaces, however numbness of the third interspace is recognized. What should the next step be? A. MRI of right forefoot B. third interspace neurectomy C. second and third interspace neurectomy D. nerve conduction study

Correct answer: (D) Nerve conduction study. Explanation: When looking at the question above, the interpretation of diagnostic blocks must be recognized and understood. Pain from one interspace can cause the patient to feel similar sensations in the adjacent interspaces. For this reason, diagnostic local anesthetic blocks are invaluable in a metatarsalgia work up. In the case above the isolated block of the third interspace only led to numbers of the interspace and no reduction of pain to either the second or third interspaces. This result must be interpreted as a more likely proximal cause exists. For this reason the next most clinically relevant step would be to order the nerve conduction test of the right tibial nerve. A distal tarsal tunnel syndrome affecting the medial plantar nerve is a very likely culprit for the patients diagnostic block results and the best clinical testing modality would be a nerve conduction study.

A 30 year old male sustained a Lisfranc injury while working as a furniture mover. The patient is a nonsmoker, has a BMI of 24, and denies any previous medical history. He is scheduled for an open reduction internal fixation of the Lisfranc dislocation. What is the best recommended method for venous thromboembolic event prophylaxis for this patient? A. 5000 units subcutaneous of lower Molecular weight heparin B. 5000 units of lower dose of unfractionated heparin C. Fondapirinux D. No prophylaxis needed, just early and frequent ambulation

Correct answer: (D) No prophylaxis needed, just early and frequent ambulation. Explanation: Although most patients require pharmacologic VTE prophylaxis in the perioperative setting, some patients are at low enough risk for VTE such that drug therapy is not recommended. Plus this is a young patient, (<40 years) without any VTE risk factors who is scheduled to undergo a Lisfranc injury repair. In low risk patients such as this case, no specific prophylaxis is required except early and frequent ambulation.

A 55 year old female present to the clinic for follow up of right heel pain. Radiographs last week demonstrated a well-defined, lytic lesion without central mineralization. The provisional diagnosis was a unicameral bone cyst. Due to its proximity to the subtalar joint an MRI was ordered. Below is a representative T1-weighted image. After review of the image, does the provisional diagnosis of a unicameral bone cyst remain and if not, what is the diagnosis? A. Yes B. No, aneurysmal bone cyst C. No, intraosseous ganglion D. No, intraosseous lipoma

Correct answer: (D) No, intraosseous lipoma. Explanation: The intraosseous lipoma is uncommon and considered by some to represent a degenerative phenomenon rather than a true neoplasm. Typically it is located in the tibia, fibula, femur and especially the anterior process of the calcaneus. The intraosseous lipoma may appear identical to a unicameral bone cyst on conventional radiographs. MR imaging of the intraosseous lipoma shows a signal intensity typical of fat identical to the signal characteristics of subcutaneous fat. Low signal intensity areas may be caused by central calcification (which is seen on the image above). To the contrary, aneurysmal bone cysts would be lobulated with septae. Intraosseous ganglions rarely occur in the calcaneus, have a well defined cortical rim and is multilocular with fluid characteristics.

In surgical repair of an unstable second MTPJ, the procedure which displayed the best long-term correction over time was: A. PIPJ fusion, MTPJ relocation, K-wire across the MTPJ prn B. PIPJ fusion, MTPJ relocation, EDB tendon transfer, K-wire across the MTPJ prn C. PIPJ fusion, MTPJ relocation, FDL tendon transfer, K-wire across the MTPJ prn D. PIPJ fusion, MTPJ relocation, 2nd-to-3rd syndactyly, K-wire across the MTPJ prn

Correct answer: (D) PIPJ fusion, MTPJ relocation, 2nd-to-3rd syndactyly, K-wire across the MTPJ prn. Explanation: The "fundamental intervention", termed FI, entailed PIPJ fusion, MTPJ relocation, and temporary K-wire fixation. Transverse plane derangement of the second MTPJ was quantified by the "transverse plane second MTPJ angle" using a bisection of the second metatarsal and second toe proximal phalanx. The "normal" value was 0 to +15 degrees; a negative value denoted digital adduction (a malalignment in the medial direction toward the hallux) and a positive angle denoting digital abduction. FI + syndactyly had an average pre-op angle of -4.5 degrees, an early post-op angle of +14.5 degrees, and a late post-op angle of -2 degrees, and the best long-term correction over time. (A) This represented the "fundamental intervention." Statistically, the average pre-op angle was - 1.5 degrees, the early post-op angle +9 degrees, and the late post-op angle +2 degrees. (B) FI + EDB transfer was only performed twice. The EDB was released and reattached to the lateral/plantar aspect of the joint with a non-absorbable suture. The average pre-op angle was - 4.5 degrees, the early post-op angle +2 degrees, and late post-op angle +0.5 degrees. (C) The pre-op angle was +3.0 degrees, the early post-op angle +7 degrees, and late post-op angle -2.0 degrees. The FDL transfer was performed one of two days; splitting it along its natural raphe and bringing it up both sides of the phalanx, or suturing it to the lateral side of the MTPJ.

Which of the following types of suture has the greatest amount of soft tissue reaction? A. Steel B. Nylon C. PDS (polydioxanone) D. Plain catgut

Correct answer: (D) Plain catgut. Explanation: Plain catgut is derived from the intestines of healthy cow or sheep and is made out of their collagen fibers. Catgut comes in plain as above or chromic. Chromic indicates the collagen was treated with chromium salts, which increases the number of cross-links in the collagen material making it more resilient to breakdown. All suture types are recognized by the body as a foreign body, igniting some form of a response from the soft tissue. Cells are recruited to the site to clear the foreign debris, break-up, and absorb the material. The peak of the soft tissue response will occur anywhere from two to seven days post-operatively. The extent of this reaction depends on the suture quantity implanted, its type of material, and its configuration. (A) Steel actually is one of the least reactive suture types listed, along with nylon. Both materials are synthetic monofilament nonabsorbable materials, which are typically low in tissue reactivity. (B) Nylon actually is one of the least reactive suture types listed, along with steel. Both materials are synthetic monofilament nonabsorbable materials, which are typically low in tissue reactivity. (C) PDS is a synthetic monofilament absorbable material which has more tissue reactivity than the synthetic monofilament nonabsorbable materials but less than the natural absorbable materials like that of plain catgut.

Concerning the depth of anesthesia, stage 3 is further subdivided into 4 planes. Which plane is best described by the following characteristics? - complete intercostal paralysis - paradoxical rib cage movement - irregular breathing - pupils dilated A. Plane 1 B. Plane 2 C. Plane 3 D. Plane 4

Correct answer: (D) Plane 4. Explanation: When discussing the depth of anesthesia there four separated stages as described by Guedel in 1937. Stage 1 is amnesia/analgesia, stage 2 is delirium/excitement, stage 3 is surgical anesthesia and stage 4 is overdosage. Stage 3 lasts from the onset of regular breathing to respiratory arrest. Stage 3 is further subdivided into four planes. Plane 4 is characterized by complete intercostal muscle relaxation to the extent of paralysis. This leads to paradoxical rib cage movement. The pupils are completely dilated the responsiveness to light is abolished.

The most common cause of flap failure is: A. Hematoma B. Ischemia C. Infection D. Poor flap design E. Anastomotic thrombosis

Correct answer: (D) Poor flap design. Explanation: All of the above can cause flap failure. Ischemia is considered to be an intrinsic factor leading to flap failure, while infection, hematoma, and anastomatic thrombosis are extrinsic factors. Poor flap design is also an extrinsic factor and is the leading cause of flap failure. Surgeon errors related to poor flap design include excessive skin traction, inadequate flap size, excessive compression, and kinking of the flap pedicle.

The Harris-Beath view demonstrates the following structures: Inferior aspect of the talus; Middle and posterior facets of the talocalcaneal articulation, Axial view of the calcaneus. The patient stands on the film cassette in the angle and base of gait with knees and ankles both flexed. The tube is angled from 35-45o. What anatomic structure is targeted by the X-ray beam? A. inferior aspect of the calcaneus B. posterior ankle joint C. tibial plafond D. posterior talocalcaneal joint

Correct answer: (D) Posterior talocalcaneal joint. Explanation: The Harris-Beath view is a specialized view used in podiatry. It is useful for identifying talocalcaneal coalitions. Although aspects of the calcaneus, talus, tibia and fibula can be visualize din this view, the only truly useful information is taken from the talocalcaneal joint, where the posterior and middle facets are clearly visualized. The patient stands on the film cassette in the angle and base of gait with knees and ankles both flexed. The tube is angled from 35-45o and targeted at the posterior talocalcaneal joint.

A 56 year old female who is scheduled for a triple arthrodesis recently underwent a right-sided mastectomy a year ago from breast cancer. She is currently on warfarin therapy for a pulmonary embolism that was diagnosed 2 weeks ago. The patient denies any other history of venous thromboembolic events. How should this patient's warfarin be managed perioperatively? A. Stop warfarin 5 days before if INR is 2-3. B. Stop warfarin 6 days before surgery if INR is 3-4.5. C. Administer last dose of lower molecular weight heparin 24 hours before the procedure. D. Postponed the surgery for a minimum of 1⁄2 a month to 1 month from the venous thromboembolic event.

Correct answer: (D) Postponed the surgery for a minimum of 1⁄2 a month to 1 month from the venous thromboembolic event. Explanation: Patient with prior VTE is at risk of recurrent VTE (DVT and PE). Risk stratification to determine if perioperative bridging is recommended largely depends on how recently the previous VTE event occurred. The risk is highest in the 4 weeks with an estimated recurrence of 0.3% to 1.3% per day without anticoagulation. This risk drops to 0.03% to 0.2% per day over the next 4 to 12 weeks, and to less than 0.05% per day after 12 weeks. With these numbers, elective surgery should be postponed for a minimum of 1 month from the VTE event to provide continuous time for anticoagulation during this time. Perioperative heparin bridging is recommended for patients with a history of VTE within the preceding 3 months and can be considered for VTE events within the past 3 to 6 months. A single VTE event that occurred more than 6 to 12 months ago is unlikely to justify bridging therapy. Patients with a prior VTE event and a known thrombophilia are also candidate for heparin bridging. Even without a diagnosed defect, if a patient has a clinically apparent thrombophilia based on a recurrent or unprovoked VTE events, perioperative bridging should be employed.

A patient presents eight weeks after sustaining a fracture through the neck of the talus. What finding is a prognostic indicator that the vascular supply is intact (Hawkins Sign)? A. absence of degenerative arthritis B. increased density of the talar body C. radiosclerotic bone with surrounding atrophic bone D. resorption of subchondral bone of talar dome

Correct answer: (D) Resorption of subchondral bone of talar dome. Explanation: Hawkins described a sign of subchondral radiolucency on the radiographs in the body of the talus that seems to infer an intact blood supply and is therefore a good indication that avascular necrosis will not occur. The Hawkins sign is best visualized on anteroposterior and mortise views. 8 weeks would be too soon to notice the presence or absence of degenerative arthritis. Increased density (Radiodensity) seen as prognostic indicator that the vascular supply is not intact. Radoisclerotic bone with surrounding atrophic bone is not correct.

Which of the following is not an effect of the drug midazolam? A. Muscle relaxation B. Anti-convulsant C. Amnesia D. Respiratory excitation

Correct answer: (D) Respiratory excitation. Explanation: Midazolam is marketed commonly under the trade name Versed. It is in the class of medications known as benzodiazepines. Benzodiazepines in general, and midazolam specifically, are used frequently in peri-operative patients. Midazolam is a popular medication in the peri- operative setting because of the effects it causes which include muscle relaxation and amnesia. Midazolam also is used as an anti-convulsant medication. It does not cause respiratory excitation, in fact, one sign of an overdose of midazolam is excessive respiratory depression.

What is the primary advantage of utilizing a staple as opposed to a bone graft in an epiphysiodesis procedure for juvenile hallux abducto valgus? A. No risk of nonunion B. Shorter recovery time C. No donor site morbidity D. Reversible

Correct answer: (D) Reversible. Explanation: Epiphysiodeis is the surgeon-controlled growth arrest of the lateral aspect of the physis to "grow out" the hallux vagus deformity. Utilization of a staple for this procedure has the benefit of reversibility is the staple is removed. (A) Epiphysiodesis performed with a bone graft carries the inherent risk of a nonunion of the graft whereas by use of a staple this risk is avoided. Although this is an advantage it is not the primary one. (B) As there would not be a need for a bone great to incorporate, the recovery time for a stapling technique would be shorter. Although this is an advantage it is not the primary one. (C) Since there is no graft needed there would be no bone graft donor site morbidity. Although this is an advantage it is not the primary one.

A 55 year old female presents to clinic complaining of bilateral intermittent pain at the balls of her feet. She denies history of pedal trauma and believes that her symptoms have been worsening over the past year. Plain films are obtained of bilateral feet and reveal the following: i. Symmetric narrowing of all metatarsal-phalangeal joints ii. Erosions along the medial aspect of all metatarsal heads iii. Osteopenia with coarsening of metatarsal head primary trabeculae iv. Absence of periosteal response & bone production adjacent to erosions Based on x-ray findings, what is the most likely diagnosis for this patient? A. Reactive arthritis B. Septic arthritis C. Psoriatic arthritis D. Rheumatoid arthritis

Correct answer: (D) Rheumatoid arthritis. Explanation: The radiographic findings listed above describe pathology primarily at the metatarsal-phalangeal joint level. Since there are multiple joints involved, it is very unlikely that the erosions are the result of an infection. Additionally one would expect destruction to occur on both sides of the joint, meaning erosions at the base of the proximal phalanx as well as metatarsal head erosions. Therefore, septic arthritis can be crossed out as a potential answer. Reactive arthritis (aka Reiter's Syndrome) is an inflammatory arthritide, just like the remaining choices. However, this form of arthritis is thought to arise following a genitourinary or enteric infection. Clinically the patient may present with skin lesions as well as a component or all features of the classic triad: urethritis, conjunctivitis, and polyarthritis. It has a predilection for affecting the calcaneus at the insertion sites of the Achilles tendon and plantar fascia. Erosions can occur at these anatomic locations, but with this bone loss comes periostitis and bone proliferation making the cortex appear fuzzy. This finding was not noted in the above description which means Reactive arthritis is incorrect. Psoriatic arthritis often resembles Rheumatoid arthritis, but there are a few radiographic features that help to differentiate the two. Psoriatic arthritis typically affects the distal joints of the hands and feet, those being the DIPJs, PIPJs, and IPJ of the hallux, and is asymmetric in joint involvement. This is an inflammatory arthritide so there is typically increased soft tissue contour and density surrounding the affected digit in its entirety and resulting in what is called a sausage digit. With Rheumatoid arthritis there is only fusiform swelling surrounding the affected joint and it usually does not extend beyond the joint area. Additionally, bone proliferation is noted in the vicinity of the erosions with Psoriatic arthritis. This is not seen with Rheumatoid arthritis. This bone proliferation may appear as whiskering or as endosteal proliferation creating an ivory phalanx. Therefore since there are erosions with no new bone production and since the joint involvement is symmetric in nature, the best answer is "D" Rheumatoid arthritis.

A 9 year old male patient presents to your office with his mother for a chief complaint of bowing of his legs. The mother states that this has been going on for many years but seems to be progressing so she wanted an evaluation. X-rays taken show widening of the physis as well as widening and cupping of the metaphysis. Based on the above scenario and described radiological findings, which of the following diagnoses are most likely? A. Rheumatoid Arthritis B. Acromegaly C. Osteomalacia D. Rickets

Correct answer: (D) Rickets. Explanation: The clinical scenario and x-ray findings of this patient only fits one of the above diagnoses which is Rickets. Commonly this is seen in children with Vitamin D deficiency. It is common radiologically to see bowing of the legs with widening of both the metaphysis and physis in this condition. It is also common to see rachitic rosary in the costo-chondral junctions in the ribs. Osteomalacia has similar findings but the age is what makes that answer incorrect in this question. Rheumatoid arthritis would show joint space narrowing that is uniform and symmetrical as well as lateral deviation of the digits. Acromegaly which is excessive growth hormone release would show widening of the joint spaces as well as increase in size of the bone itself.

Your 13 y/o patient stepped on a nail, at the level of the midshaft of the third metatarsal. X-rays taken in the ER were negative for fracture or retained foreign body. The patient was given amoxicillin for five days, and a tetanus shot. The patient followed up with the PCP one week later, with the foot painful, swollen, and erythematous. The primary placed the patient on dicloxicillin for seven days. The signs and symptoms resolved, only to recur once the antibiotic was finished. The patient now presents to you, again with a hot, red, swollen foot, with a draining sinus at the puncture site. The best course of action is: A. Begin dicloxicillin for an additional ten days B. Add ciprofloxacin for gram negative coverage C. Perform a tibial nerve block; explore, culture, and pack the sinus tract in the office D. Schedule a formal I&D procedure in the OR ASAP

Correct answer: (D) Schedule a formal I&D procedure in the OR ASAP. Explanation: This is a classic scenario, in which an antibiotic is started, the patient gets better, then regresses after the antibiotic is finished; a new antibiotic is started, gets better, worsens; the cycle goes on and on. Of course, an x-ray should be ordered for retained foreign body and osteomyelitis. If negative, the possibility of foreign material, especially remnants from the shoe insole, cannot be totally excluded. An I&D in the OR will allow for debridement of necrotic tissue, obtaining a good deep culture (actual tissue preferred), and a bone biopsy/culture can also be obtained, if deemed necessary. (A) The most common infecting organism for a puncture wound is Staph aureus, so dicloxicillin is not an incorrect choice, but for the reasons above, more aggressive treatment is necessary. (B) The most common infecting organism in puncture wound osteomyelitis is Pseudomonas. But, osteomyelitis has not been confirmed. Also, although opinions are changing, most still agree that fluroquinolones should not be utilized in the pediatric population due to growth plate problems. (C) This option will not give you as thorough a debridement as is required, and the culture will likely not be as reliable as a deeper culture would be, taken in the OR.

Which of the following is not a commonly used needle point design? A. Conventional cutting B. Reverse cutting C. Taper D. Scoop

Correct answer: (D) Scoop. Explanation: Suture needles are generally made from high-grade surgical steel and are designed to allow the suture to pass through soft tissue. There are multiple different suture needle point designs which are tailored to improved passage through different types of soft tissue. The most commonly used designs in foot and ankle surgery are the conventional cutting and reverse cutting. Cutting needle designs have at least 2 opposing cutting sides along the needle point. This allows the needle and suture to pass through dense, tough soft tissues with less trauma to the tissues. Taper point needles have, as the name implies, a gradual taper to a sharp, narrow point. Although spatula tip needles do exist, there is no needle point design called a scoop needle point, therefore the correct answer is D.

Which stage of anesthesia is described by the following characteristics? - respiratory paralysis - flaccid muscles - eyes widely dilated - eyes non-reactive - cardiopulmonary collapse A. Stage 1 B. Stage 2 C. Stage 3 D. Stage 4

Correct answer: (D) Stage 4. Explanation: When discussing the depth of anesthesia there four separated stages. Stage 1 is amnesia/analgesia, stage 2 is delirium/excitement, stage 3 is surgical anesthesia and stage 4 is overdosage. Stage 3 is further subdivided into four planes. Stage 4 begins with paralysis of the diaphragm to cardiac arrest. Stage 4 is characterized by respiratory paralysis, as well as flaccid muscles. The eyes are fully dilated and are non-reactive to light. Stage 4 concludes with complete cardiopulmonary collapse and death.

Ten days after a left third space neurectomy, a 41 year old female presents for her second post operative visit. On her initial post operative visit 3 days ear;y she was diagnosed with a post operative hematoma. At the time she agreed utilize crutches and maintain a nonweightbearing status. Additionally a multi-layer compressive dressing was applied. On her visit today, she reports 100% compliance to the weight-bearing restriction. Upon removal of her dressing there is little improvement noted in her hematoma. What would be the next logical step? A. Continue nonweightbearing and apply an unna boot-type dressing B. Empiric antibiotics C. Heat compresses and nonweightbearing D. Suture removal and full expression of hematoma

Correct answer: (D) Suture removal and full expression of hematoma. Explanation: The key to this question is the realization that despite strict adherence to instruction combined with an appropriate first-line therapy for a hematoma post interspace neurectomy, her hematoma persist. For more complicated and persistent hematoma it is paramount that evacuation of the fluid occurs before bacterial proliferation results in a deep space infection. To this extent the most appropriate follow up treatment would be to judiciously remove necessary sutures and perform complete expression of the contents of the interspace with copious lavage. Continuation of the initial therapy is unlikely to yield different results. Furthermore, antibiotics are not justified at this time as they will not aid in eradication of the hematoma.

Which of the following statements regarding fractures of the posterior talar process is correct? Non-operative treatment of this injury is in part accomplished by a below-knee cast held in 5 degrees of dorsiflexion for a 4- to 6-week period. The posteromedial tubercle is avulsed due to the attachment of the posterior tibiotalar ligament upon forceful dorsiflexion of the ankle. Provocation of pain can be elicited upon moving the lesser toes passively or actively. The mechanism of injury can either be by excessive plantarflexion or dorsiflexion.

Correct answer: (D) The mechanism of injury can either be by excessive plantarflexion or dorsiflexion. Explanation: Fracture of the posterolateral process of the talus occurs via significant plantarflexion of the ankle which impinges the tubercle against the tibial plafond. Likewise excessive dorsiflexion can cause an avulsion of the tubercle due to the attachment of the posterior talofibular ligament. This provides the justification for answer (D). In contrast, the non- operative treatment for this injury if a below-knee cast in 5 degrees of plantarflexion for 4-to 6- weeks. Similarly, the second choice is anatomically incorrect. This fracture involves the posterolateral process of the talus and is in part associated with the posterior talofibular ligament. Furthermore, provocation of the pain can be evoked by passive or active motion of the hallux which moves the flexor hallucis longus tendon in the groove between the two tubercles and agitates the fracture.

A medial displacement calcaneal osteotomy was performed as part of the surgical solution for the stage II posterior tibial tendon dysfunctional foot pictured below. Which of the statements below is not correct regarding the medial displacement calcaneal osteotomy? The skin incision is placed parallel to the peroneal tendons and is made directly down to bone. When performing the osteotomy, care must be taken when exiting medial to avoid the medial neuromuscular structures. The osteotomy re-establishes the Achilles tendon as a heel inverter. The medial displacement calcaneal osteotomy produces a tightening effect on the plantar fascia.

Correct answer: (D) The medial displacement calcaneal osteotomy produces a tightening effect on the plantar fascia. Explanation: The medial displacement calcaneal osteotomy is a commonly performed procedure in the surgeon's armamentarium for flatfoot surgery. The skin incision is made parallel to the peroneal tendons just anterior to the achilles tendon attachment on the superior surface of the calcaneus extending to a point distal and inferior to the weight-bearing surface of the calcaneus. It is made directly down to bone where subperiosteal direction ensues. Structures at risk during the procedure include the Achilles superiorly, plantar structures inferiorly and the medial neuromuscular bundle just distal to the medial cortical calcaneal cortex. Furthermore, by shifting the posterior tuber of the calcaneus medially, you are in reality shifting the distal Achilles attachment medially in relation to the subtalar joint axis. This does indeed re-establish the Achilles as an inverter of the subtalar joint. Although once thought to tighten the plantar fascia with medial tuber displacement, studies have shown that in actuality the plantar fascia is more relaxed after the medial calcaneal displacement osteotomy.

A 26 year old male presents complaining of right ankle pain as well as right plantar and posterior heel pain. He states the pain has been present for about 6 months. Clinical and radiographic images are below. A. The radiographic finding and cutaneous findings are probably not related. B. Treatment with anti-inflammatories is not effective. C. Immobilization in a below-knee cast will lay resolves symptoms. D. The patient has had a recent venereal, conjunctival or enteric infection.

Correct answer: (D) The patient has had a recent venereal, conjunctival or enteric infection. Explanation: The scenario described above is a classic example of Reiter's syndrome. The classic Reiter's patient is male between the ages of 13-60 with history of a venereal, conjunctival or enteric infection 1 to 4 weeks prior to the development of musculoskeletal or cutaneous complaints. In the radiograph an erosion is noted on the posterior calcaneus with spurring, as well as a fluffy periosteal reaction on the plantar calcaneus. Knee, ankle and subtalar joints are often commonly affected. The cutaneous manifestation that is shown is keratoderma blennorrhagicum which occurs in Reiter's Syndrome. So, indeed the radiographic and cutaneous findings are related. Furthermore, NSAIDs are the first line of treatment for Reiter's. Conversely, immobilization is contraindicated. Mobilization to maintain joint function is a primary goal of treatment.

It is standard and customary to cleanse the donor site on a skin graft harvest site with alcohol before harvesting with a power dermatome. What is the rationale for this? A. To sterilize the skin graft harvest site B. To cool the skin graft harvest harvest site through evaporative cooling C. To open the pores of the skin graft D. To remove sticky residue from the surgical sterile preparation

Correct answer: (D) To remove sticky residue from the surgical sterile preparation. Explanation: Surgical sterile preparations leave a sticky residue on the skin. The sticky residue would interfere with the harvesting with the power dermatome and interfere with graft. Therefore cleaning the skin graft harvest site with alcohol before graft harvest is of great importance. The alcohol does not provide any greater sterilization than the surgical scrub that was used at the initiation of the surgery. Additionally, alcohol has no effect on the pores and although it would cool the surface skin temperature by evaporative cooling a negligible amount it has no relevance to skin graft harvesting.

What procedure is indicated in the surgical treatment of recurrent plantar fibromatosis following prior local resection? A. Local resection with intra-lesional corticosteroid infiltration B. Wide resection of the plantar fascia C. Sclerosing therapy for the lesions D. Total fasciectomy

Correct answer: (D) Total fasciectomy. Explanation: Total fasciectomy is indicated when there has been a local recurrence of plantar fibromatosis or when there is involvement in multiple sites. (A) Local resection with intra-lesional corticosteroid infiltration is not indicated for surgical correction of recurrent plantar fibromatosis. (B) Wide plantar fascial resection is not indicated for surgical correction of recurrent plantar fibromatosis. (C) Sclerosing therapy is not indicated for treatment of recurrent plantar fibromatosis.

Refer to the fixation involved in the surgical correction pictured below. Which is a true statement regarding the fixation pictured below? Cannulated screws were used for the first metatarsal cuneiform arthrodesis The compression staple on the Evans procedure has caused dorsal migration of the anterior calcaneus Over-drilling is not necessary for the screw used in the first metatarsal cuneiform arthrodesis Two screw fixation as opposed to single screw fixation the calcaneal osteotomy provides resistance to rotational forces

Correct answer: (D) Two screw fixation as opposed to single screw fixation the calcaneal osteotomy provides resistance to rotational forces. Explanation: Pictured above is an excellent example of a flatfoot correction. Two fully threaded non-cannulated screws were used in the fixation of the first metatarsal cuneiform arthrodesis. They were inserted using a lag technique requiring over-drilling of the proximal cortex. A compression staple was utilized on the Evans procedure and is performing well stabilizing the bone graft and providing excellent position. The anterior calcaneus is well aligned. In the osteotomy of the calcaneus two partially threaded screws were used. The major advantage of using two screws as opposed to one is stabilization of rotational forces that aren't stabilized with single screw fixation.

An adult polio patient has a tibialis anterior tendon graded 2/5 muscle strength, both peroneals graded 3/5 muscle strength, and the long toe extensors graded 3/5 muscle strength. The gastrocnemius soleus and tibialis posterior are both 4/5 in strength. The quadriceps are graded 5/5 in strength. The resultant foot structure is equinocavovarus. The best treatment option is: A. Lambrinudi's arthrodesis only B. Watkins tibialis posterior tendon transfer only C. TAL only D. Using all three in conjunction

Correct answer: (D) Using all three in conjunction. Explanation: Muscles are graded 0-5; 0 has no power while 5 is full strength against gravity and resistance. Tendons lose one grade of power upon transfer; thus, only a 4/5 or 5/5 tendon should be transferred. In the above scenario, the tibialis anterior, peroneals, and toe extensors are adversely affected. The gastrocnemius soleus and tibialis posterior are available for transfer. The goal is a stable plantigrade foot free from bracing. As an adult, this is probably a long-standing deformity. Thus, both bony and soft tissue work is necessary for the best long-standing result. The tendon transfer will convert the dynamic deforming force into a corrective force, and prevent recurrence of a deformity following the static deformity correction. The Lambrinudi arthrodesis removes bone from the distal and plantar aspect of the talus, and displaces the navicular dorsally. This maneuver corrects the foot equinus, but leaves the talus in a preferred equinus position. A triple arthrodesis is then completed. The tibialis posterior tendon is detached, brought through the interosseous membrane to the dorsum of the foot. To prevent or correct varus, the tendon is secured to the lateral cuneiform, or even the cuboid. The gastrocnemius soleus remains a deforming force, so a TAL is indicated. Before a TAL is performed, check the quadriceps strength, as a weak quadriceps in conjunction with a TAL can unmask knee instability. A posterior ankle capsulotomy may also be necessary.

On an ankle x-ray, an avulsion fracture is noted from the distal fibula by the anterior inferior tibiofibular ligament. This is also known as: A. Volkmann fracture B. Dupuytren's fracture C. Maisonneuve fracture D. Wagstaffe fracture

Correct answer: (D) Wagstaffe fracture. Explanation: A Wagstaffe fracture is an eponym for a distal fibular avulsion fracture by the anterior inferior tibiofibular ligament. This is commonly seen in SER ankle fractures. (A) A Volkmann fracture is a fracture of the posterolateral aspect of the tibia. (B) A Dupuytren's fracture is of the fibula approximately two inches proximal to its distal aspect. (C) A Maisonneuve fracture is a spiral fracture of the proximal third of the fibula which is often associated with a tear of the distal tibiofibular syndesmosis, fracture or medial malleolus or deep deltoid liagment and is associated with an interosseous membrane tear.

Which of the following are potential complications of tarsal coalitions? A. Pes planus B. Peroneal spasm C. Talar beaking D. Subtalar joint arthrosis E. All of the above

Correct answer: (E) All of the above. Explanation: The potential complications (stigmata) tarsal coalitions include pes planus, peroneal spasm, talar beaking, subtalar joint arthrosis, as well as possible equinus, ball-and-socket ankle joint, and calcaneal fibular remodeling.

Your patient is a 40 year old male with stiffness in the great toe. Exam reveals limited dorsiflexion and plantar flexion at the first MTP. There is swelling around the joint and the patient complains of pain when he pushes off on the toes to walk. Which of the following is not an appropriate option for treatment of arthritis of the first metatarsophalangeal joint? A. Cheilectomy B. Metatarsal osteotomy C. Arthroplasty D. Arthrodesis E. Medial capsulorrhaphy

Correct answer: (E) Medial capsulorrhaphy. Explanation: Medial capsulorrhaphy is a procedure designed to tighten the soft tissue around the first metatarsophalangeal joint to correct a bunion deformity. Pain relief is achieved by straightening the great toe, and the procedure may also improve the cosmetic appearance of the foot. It is not an appropriate treatment for arthritis, although it can be performed in conjunction with other procedures. The other procedures are all used to treat arthritis of the great toe. Cheilectomy removes bone spurs on the top surface of the bones that develop with arthritis. A metatarsal osteotomy is used to cut the bone to shorten the metatarsal to achieve better alignment, usually performed on the lesser toes. A chevron osteotomy consists of resection to the metatarsal to correct misalignment. Arthroplasty is used to treat patients with moderate to severe hallux rigidus by taking away damaged bone and placing a piece of soft tissue from the foot, a tendon or capsule, into the joint space to allow movement. In advanced stages of hallux rigidus, severe joint damage can be treated by removing damaged cartilage and fusing two bones together with screws or plates, which allows them to grow together. This is a permanent correction of the joint, and it results in elimination of arthritis and pain, but movement in the big toe is restricted.

Which of the following is not a feature of Gram-negative diabetic foot osteomyelitis? A. Fetid odor B. Necrotic soft tissue C. Leukocytosis D. Elevated white cell count E. Normal white cell count

Correct answer: (E) Normal white cell count. Explanation: Gram-negative diabetic foot osteomyelitis is characterized by fetid odor, necrotic soft tissue, leukocytosis, elevated glycosylated hemoglobin, and often history of traumatic wound. White cell count is elevated.

When imaging the calcaneus in the axial view, which of the following is correct? A. The plantar surface of the foot should be parallel to the image receptor. B. The central ray should be perpendicular to the midportion of the calcaneus. C. The central ray should be 20 degrees cephalad to the long axis of the foot. D. The central ray should enter the foot at the level of the base of the first metatarsal. E. The central ray should be 40 degrees cephalad to the long axis of the foot.

Correct answer: (E) The central ray should be 40 degrees cephalad to the long axis of the foot. Explanation: When obtaining an axial or plantar dorsal view of the calcaneus, the patient should be in a supine or seated position with leg fully extended. The image receptor should be centered to the ankle with the plantar surface of the foot in perpendicular position to the image receptor. The central ray should be directed 40 degrees cephalad to the long axis of the foot, entering midline at the base of the fifth metatarsal.

Which of the following is consistent with an injury to the Lisfranc joint? A. Negative weight bearing radiographs rule out injury. B. They resolve spontaneously without sequelae in most patients. C. They result in instability of the talo-navicular complex. D. The Lisfranc joint can be divided into two columns. E. The joint stabilizes the midfoot arch.

Correct answer: (E) The joint stabilizes the midfoot arch. Explanation: The Lisfranc joint is the articulation between midfoot and forefoot and is composed of five tarsometatarsal joints that stabilize the midfoot arch. The joint can be divided into three longitudinal columns, including the medial, middle, and lateral columns. The medial column consists of the first ray, the middle column consists of the second and third tarsometatarsal joints, and the lateral column consists of the fourth and fifth tarsometatarsal joints. Lisfranc joint injuries are rare and as many as 20% are missed on initial anteroposterior and oblique radiographs. They carry a high risk of chronic disability if untreated. Signs and symptoms include swelling over the midfoot and pain with palpation along the tarsometatarsal articulations. A negative weight-bearing radiograph does not rule out a grade 1 or 2 injuries.

Which of the following procedures is best indicated for the underlapping fifth toe pictured below? A. Ruiz-Mora Procedure B. Thompson Procedure C. Post Procedure D. Lapidus Procedure

Correct: B Explanation: The Thompson procedure is the only procedure listed above indicated for an underlapping fifth toe. It consists of a Z-type incision dorsally over the proximal phalanx, the resection of the fifth proximal phalanx in its entirety, reefing of the capsule to fill the dead space and corrective closure of the Z incision. The Ruiz-More procedure is indicated for a cock-up fifth toe. It consists of an elliptical plantar incision, resection of the proximal phalanx and suturing of the incision to medial deviate and plantar flex the toe with closure. The Post procedure is for correction of the rectus hammertoe. It consists of resection of the head of the proximal phalanx through either a linear or transverse elliptical incision and reefing of the capsule with closure. The Lapidus procedure is indicated for severely overlapping fifth toes. It consists of a curvilinear incision over the dorsum of the fifth tmp joint and proximal phalanx with a secondary more proximal incision over the extensor tendon. The extensor tendon is transacted proximally and bought through the distal incision. The dorsal, medial and lateral tmp capsule are released. The extensor tendon is wrapped around the phalanx from dorsal medial to plantar lateral and attached to periosteum of the head of the fifth metatarsal, soft tissue or muscle proximal to the fifth metatarsal head.

Which theory describes the relationships of the longitudinal and oblique axis of the midtarsal joint in response to STJ position? A. Root's Theory B. Valmassy Theory C. Lewis Theory D. Elftman Theory

Correct: D Explanation: As the subtalar joint pronates, the two axes of the midtarsal joint become more parallel with each other, allowing more motion of the forefoot on the rearfoot. This "unlocking" allows for much easier motion of the midfoot on the hindfoot, often associated with instability. In a supinated subtalar joint, the two axis of the midtarsal diverge decreasing the motion available at the two joints of the midtarsal joint. This "locking" of the midtarsal joint is believed to improve bipedal ambulation and typically occurs during late midstance and propulsion phases of gait. During this time, the lesser tarsal bones are stabilized by the synergistic action of the posterior tibial and peroneus muscles. Clinically, the locking of the midtarsal can be estimated by placing the STJ in neutral and applying a dorsiflexion force on the 4th and 5th metatarsal heads to resistance, as commonly performed during plaster casting for custom orthotics.

The most common complication in surgical repair of all forms of polydactyly is: A. Infection B. Forefoot imbalance C. Scar contracture D. Selecting wrong toe for deletion

Correct: D Explanation: The majority of polydactyly complications result from faulty judgement and lack of experience on the part of the surgeon. Insufficient pre-operative surgical planning or vascular assessment will often lead to selecting the wrong digit for removal. Infection rates with polydactyly cases are not any different than the rate for any other clean foot procedure. Forefoot imbalance is of no real concern because with the exception of the hallux, the lesser toes play little role in structural balance of the forefoot. The adjacent metatarsal heads are the major players in the balance of the forefoot. Scar contracture would also cause no greater problem in the polydactyly cases as in other foot cases.

You are a resident in the clinic. You would like to take radiographs of a patient's foot. Assuming the patient is positioned correctly, what is the tube head angulation for a dorsoplantar projection? A. 45 degrees from the vertical, directed posteriorly B. 0 degrees from the vertical, directed posteriorly C. 90 degrees from the vertical, directed anteriorly D. 15 degrees from the vertical directed posteriorly

Correct: D Explanation: Tubehead angulation should be 15 degrees. It has been advocated that DP projections be performed with the tubehead at 5, 10, 20 degrees posteriorly, depending on the metatarsal declination, but this should be performed as adjunct, always start with 15 degrees. On the average, met declination is 15 degrees relative to the plane of support.

Full thickness skin grafts: (Select all that apply) A. Have a higher failure rate B. Include the epidermis and dermis without any appendages C. Common donor sites are the sinus tarsi and gluteal fold D. Has a better cosmetic appearance than the split thickness skin graft (STSG)

D - Has a better cosmetic appearance than the split thickness skin graft (STSG),C - Common donor sites are the sinus tarsi and gluteal fold,A - Have a higher failure rate. Correct answer: (A) (C) (D) Have a higher failure rate, common donor sites are the sinus tarsi and gluteal fold and has a better cosmetic appearance than the split thickness skin graft (STSG). Explanation: FTSG's have a higher failure rate because there are more structures attached to the graft requiring an intact blood supply to successfully incorporate. In addition to the dermis and epidermis, most dermal appendages are included in the graft. In addition to the sinus tarsi and gluteal fold, flexor creases such as the popliteal fossa and inguinal area are potential donor sites. STSG's have a higher degree of contracture, are less durable, and may display hyperpigmentation.

Recurrent hindfoot valgus can occur in rheumatoid patients even after successful reduction and fusion of the subtalar joint. What is the manner in which this deformity recurs even after successful reduction and fusion of the subtalar joint? (Select all that apply) A. Failure of fixation of the subtalar joint B. Residual equinus C. Subluxation of the tibiotalar joint D. Subluxation of the naviculocuneiform joint

D - Subluxation of the naviculocuneiform joint,C - Subluxation of the tibiotalar joint. Correct answer: (C) (D) Subluxation of the tibiotalar joint and subluxation of the naviculocuneiform joint. Explanation: Valgus deformity of the hindfoot can recur after fusion in the rheumatoid patient due to subluxation of the tibiotalar joint and failure of the deltoid ligaments. Valgus deformity of the hindfoot can recur after fusion in the rheumatoid patient due to subluxation of the naviculocuneiform joint. (A) As stated above there is a successful fusion of the subtalar joint so failure of the fixation is not possible. (B) Even if there was residual equinus the solid union of the subtalar joint, would not allow any movement of the hindfoot.

The phenomenon pictured below is commonly associated with which disease(s)? (select all that apply) A. Alcoholism B. Diabetes mellitus C. Gout D. Syringomyelia

D - Syringomyelia,B - Diabetes mellitus,A - Alcoholism. Correct answer: (A) (B) (D) Alcoholism, diabetes mellitus and syringomyelia. Explanation: The phenomenon pictured above is acute Charcot neuroarthropathy. Charcot neuroarthropathy is associated with the following conditions: diabetes mellitus with peripheral neuropathy, tabes dorsalis, leprosy, syringomyelia, meningomyelcoele, congenital insensitivity to pain, chronic alcoholism, spinal cord injury and compression and peripheral nerve injuries. In the western world diabetes mellitus is the most common cause. The average age of onset is 57- years-old, the average of length of time that an individual was diagnosed as diabetic until the onset of Charcot is 15 years. Alcoholism, diabetes and syringomyelia, as in all the predisposing conditions, have an obvious neural commonality. Gout does not have a neural component, but rather is an inflammatory state caused by dysfunctional purine metabolism.

Empiric therapy for diabetic foot infections directed at Pseudomonas aeruginosa is unnecessary except for patients with the following risk factors: A. Elevated glycosylated hemoglobin level B. High local prevalence of Pseudomonas infection C. Frequent exposure of foot to water D. Warm climate E. Tobacco abuse

D - Warm climate,C - Frequent exposure of foot to water,B - High local prevalence of Pseudomonas infection. Correct answer: (B) (C) (D) High local prevalence of Pseudomonas infection, frequent exposure of foot to water and warm climate. Explanation: These risk factors are the only ones recognized by the Infectious Disease Society of America in its guidelines. Glycosylated hemoglobin level and tobacco abuse are not associated with increased risk for Pseudomonas infections.

Which statement(s) is/are true regarding pediatric subtalar coalitions? (Select all that apply) A. The talocalcaneal coalition is most common. B. The C-sign, although a reliable indicator of T-C coalition, is not pathognomonic. C. The best imaging study in diagnosing a T-C coalition is the CT scan. D. A CT scan should be ordered prior to a calcaneonavicular coalition resection, to rule out a second T-C coalition and clearly define the extent of the coalition. E. 90% of coalitions are either talocalcaneal or calcaneonavicular. F. The EDB interposition is utilized in T-C coalition resection.

E - 90% of coalitions are either talocalcaneal or calcaneonavicular.,D - A CT scan should be ordered prior to a calcaneonavicular coalition resection, to rule out a second T-C coalition and clearly define the extent of the coalition.,C - The best imaging study in diagnosing a T-C coalition is the CT scan.,B - The C-sign, although a reliable indicator of T-C coalition, is not pathognomonic.. Correct answer: (B) (C) (D) (E) Explanation: Historically, the calcaneonavicular coalition was considered the most common, perhaps because it was easier to diagnose with plain films. However, with advanced imaging with CT (the gold standard) or MRI, the middle facet talocalcaneal coalition, and anterior process of the calcaneus with the navicular coalition, occur in fairly equal frequency. The C-sign has been found to have low sensitivity for T-C coalition; absence of the middle facet may be a more accurate sign. The T-C coalition may be suggested on the lateral or axial x-ray, but CT remains the gold standard for imaging of this coalition. The calcaneonavicular coalition may be more extensive under the talar head and not appreciated on plain films. Also, a second T-C coalition is not that rare, and may be missed on plain films. Thus, the indication for a CT scan. Other midfoot/rearfoot coalitions, such as calcaneocuboid or naviculocuboid, are not commonly seen. The EDB interposition (Badgley) is utilized in C-N coalition resection.

Which of the following would not be a contraindication to spinal anesthesia? A. Anticoagulant therapy B. Hypovolemic state C. Infection at the injection site D. History of malignant hyperthermia

Explanation: Contraindications to neuraxial anesthesia include bleeding disorders and anticoagulation, infection near the site of insertion, septicemia, shock or hypovolemia, certain spinal deformities and highly anxious patients. Patients on anticoagulation therapy have a higher risk for hematoma after spinal anesthesia so it is a contraindication as the hematoma can compress the spinal cord. Hypovolemia and shock are contraindications due to the anesthetized sympathetic nervous system with spinal anesthesia. The induced hypotension and bradycardia could worsen the patient's condition. Malignant hyperthermia is an emergent condition induced by general anesthesia but is not a condition associated with spinal or regional anesthesia alone.

A 92-year-old female patient is transferred to the emergency department for evaluation of an infected foot wound with likely osteomyelitis and sepsis. Her past medical history is significant for hypertension, diabetes mellitus, and myocardial infarction. She was diagnosed with metastatic squamous cell lung cancer a month prior to admission. On arrival, her temperature is 101°F, blood pressure is 85/40 mmHg, heart rate is 120/minute, and respirations are 38/minute. Pulse oximeter reveals an SaO2 of 84. The patient will probably require intubation to survive, but she has a living will signed 3 weeks ago that states she does not want any extraordinary measures taken to prolong life. Her daughter just arrived and states she wants "everything done." What is the most appropriate course of action? Transfer patient back to nursing home. Intubate patient and transfer to ICU. Inform daughter that her mother has a living will and that you will not intubate her to prolong her life. Meet with daughter and any other family members to discuss their rationale for ignoring patient's wishes about terminal care. Politely inform daughter that she will have to prove her mother was incompetent when she signed her living will if care is to continue.

Main learning point: Family meetings are an important aspect of end of life care. Correct answer: (D) Meet with daughter and any other family members to discuss their rationale for ignoring patient's wishes about terminal care. Explanation: Although it is never possible to predict the outcome of a calm discussion about terminal care when family members are faced with the imminent death of a loved one, it is still the best course of action. The choices made by the patient in a living will should be honored in most cases, but there may be extenuating circumstances at times. It is most appropriate to treat the patient's family with respect and to give them the opportunity to discuss why they feel the patient's wishes should be honored or overruled. Ignoring the daughter or refusing to discuss the situation is never an appropriate choice. Telling the daughter to embark on a legal course of action to override her mother's living will can contribute to her stress and is not appropriate.

Immediately after a needle stick, you are tested and are HIV positive. You were negative when last tested 9 months ago. You have, over the past several months, performed invasive procedures on several patients and plan on continuing to do so. Are you required to tell your patients (past and present) this information? Yes, all surgical patients should be aware if their surgeon has a potentially communicable disease. No, continue to use universal precautions, but you are not required to disclose your status to patients. Yes, inform both your hospital and your patients of your HIV status. You are only required to inform your patients after you have been diagnosed with an AIDS-defining illness. You are obligated to disclose your status to your hospital, and they have the right to terminate you based on your increased risk to patients.

Main learning point: In the case of a healthcare worker with a blood-borne illness, they are ethically obligated to tell the hospital/employer but the employee cannot be terminated based upon this new information. Correct answer: (B) No, continue to use universal precautions, but you are not required to disclose your status to patients. Explanation: Healthcare workers should use universal precautions at all time. Whether an infected physician should disclose the condition depends upon the likelihood of risk to the patient and the relevant regulations, but if it becomes clear that the risk to a patient associated with contact or with a procedure is high despite appropriate preventive measures, physicians should remove themselves from care. If a clinically significant exposure occurs, the physician is obligated to disclose the condition after the fact. In the case of a healthcare worker with a blood- borne illness, there is an ethical obligation to notify the hospital or employer, but this information cannot be used to terminate your employment.


Conjuntos de estudio relacionados

PrepU ch.19 Assessing Lungs and Thorax

View Set

Costs in the short run Study Plan

View Set

English 10H Logical Fallacy Quiz

View Set

Securities Industry Essentials Exam

View Set

EVALUATE VARIABLE EXPRESSIONS (LEVEL C)

View Set

Gen Chem Class 4 Amplifire 1/2 and 2/2

View Set

Chapter 10: Sternum and Ribs (Bontrager)

View Set

SmartBook Assignment Chapter 16/17: Disorders in Immunity

View Set

Section 6, Unit 2: Deed of Trust and Mortgage Instruments

View Set